面白い問題おしえて〜な 十七問目

このエントリーをはてなブックマークに追加
1132人目の素数さん
2132人目の素数さん:2010/09/13(月) 04:44:56
183 名前:132人目の素数さん[sage] 投稿日:2010/05/20(木) 02:40:03
3つの実数x,y,z ( x+y+z > 0 , y < 0 )を (x,y,z) → (x+y,−y,z+y)
とする変換を考えます。変換後、負の数のどれか(存在するとき)をyとして残りをx、zと
して同じ変換を繰り返します。このとき変換のときにどのように負の数を選んでも
必ず有限回の変換後に3つの数すべてが非負実数になることを証明せよ。

前スレ183の問題だけど
f(x,y,z)=|x+y|+|y+z|+|z+x|+|x|+|y|+|z|とおけば x+y+z>0 y<0 である限りは
f(x,y,z)-f(x+y,-y,z+y)=|x+z|-|x+2y+z|=|x+y+z|+|y|-|(x+y+z)+y|>0 となるのを利用するんだっけ

x,y,zに変換を行う度にf(x,y,z)を記録していったら
x,y,zが整数なら記録されたf(x,y,z)の値達は自然数の減少数列になるから
いずれはx,y,zとも非負にならざるを得ない
でもx,y,zが実数なら同じ手法が使えないし、いつまでも変換を行える反例あるんじゃね
3132人目の素数さん:2010/09/13(月) 10:16:38
問題
以下の数列の解を求めよ。
f(1) = 0
f(n) = Σ(k=2, n)[log(k-1)/log2)]+1, (n >= 2)
4132人目の素数さん:2010/09/13(月) 22:56:45
以下の条件をみたすように球面を5つの領域に分割する方法を1つ答えよ。
(1) 5つの領域はすべて合同である。
(2) 同一の点を共有しない2つの領域が存在する。
5132人目の素数さん::2010/09/14(火) 11:59:58
ある数列は、1と2からなる無限数列で、次の条件をみたす。

T 数列の最初の要素は、1である。
U 3っつの連続した1はなく、さらに、2つの連続した1も無い。
V 数列に中の連続した2つの1を単独の(つまり一個の)2に置き換え、
 単独の1はそのまま残し、
 数列にもとにある2を消去すると、
 この操作によって得られる数列は、元の数列と同じ。

 (Uによって、Vに述べられた操作で数列を書き換えることができる)

(1) 具体的に、この数列を表示してみよ。
  (5項以上で)

(2)この数列の最初のn個の要素の中にいくつの2があるか。

(3)数列を出力するプログラムを作成してみてください。
  (おまけ)
6132人目の素数さん:2010/09/14(火) 17:05:55
問題文これでいいの?
7132人目の素数さん:2010/09/14(火) 18:04:19
2つの連続した1が無いならば、1は常に単独。よって

>V 数列に中の連続した2つの1を単独の(つまり一個の)2に置き換え、

この操作は無意味。しかも「数列に中の」は日本語でおk。
8132人目の素数さん:2010/09/14(火) 19:30:43
>>3 の問題を訂正
以下の数列の解を求めよ。
f(1) = 0
f(n) = Σ(k=2, n)([log(k-1)/log(2)]+1), (n >= 2)
9132人目の素数さん:2010/09/14(火) 20:33:49
>>8
もう解になってるじゃん。
Σを使わない形で表現しろってこと?
10132人目の素数さん:2010/09/14(火) 20:49:48
>>9
そうだけど。
11132人目の素数さん:2010/09/14(火) 21:12:13
>>8
どこに面白さがあるの?
問題そのもの?解く過程?解いた結果?2進数が面白いの?
12132人目の素数さん:2010/09/14(火) 21:32:23
>>11
数列の問題でlogがやガウス記号が有るから風変りな問題で
解く過程も変わっていると思う。
13132人目の素数さん:2010/09/14(火) 21:46:28

(1) u(l+m+n)膿{0}_{i=-2}k_i を展開しなさい。
(2) (1)の展開式の中に異物が混じってます。それを答えなさい。
14132人目の素数さん:2010/09/14(火) 22:45:58
>>4
正20面体の絵を見ながらやってみたができなさそう。
60面に分割してみたが工作しないと無理とわかったw
15132人目の素数さん:2010/09/15(水) 01:07:02
>>8

略解:

n≧2のときを考える。
D1:= { (x,y)∈R^2|1≦x≦n−1 , 0≦y≦log_[2]x }
D2:= { (x,y)∈R^2|0≦y≦log_[2](n−1) , 0≦x≦2^y }
と置く。D1に含まれる格子点の個数がf(n)である。
D2に含まれる格子点の個数をg(n)と置けば、
D1∪D2 = { (x,y)∈R^2|0≦x≦n−1 , 0≦y≦log_[2](n−1) } (長方形)
となるから、f(n)+g(n)=n*(1+[log_[2](n−1)]) となる。g(n)は簡単に
計算できてるので詳細は省略する。最終的に、

f(n) = n*(1+[log_[2](n−1)]) + 1−2^*(1+[log_[2](n−1)]) (n≧2)

となる。
16132人目の素数さん:2010/09/15(水) 01:08:30
オウ、ミス…

誤: D2:= { (x,y)∈R^2|0≦y≦log_[2](n−1) , 0≦x≦2^y }
正: D2:= { (x,y)∈R^2|0≦y≦log_[2](n−1) , 0≦x < 2^y }
17132人目の素数さん:2010/09/15(水) 03:54:24
>>14
裏返したものも合同としていいなら、
20面体でできるっぽい。
18132人目の素数さん:2010/09/15(水) 04:03:35
20面体の展開図イメージ

△_▲_△_△_△
▽△▼▲▽△▼▲▼▲
 ̄▽ ̄▼ ̄▽ ̄▽ ̄▽

黒の2つの領域が互いに共有点を持たない。
(他の部分の分割の仕方は察してくれ)
19132人目の素数さん:2010/09/15(水) 04:20:40
こっちの方がわかりやすかったか。

▲_▲_△_△_△
▽△▼▲▽△▼▲▽△
 ̄▽ ̄▽ ̄▽ ̄▼ ̄▼

この展開図上で縦に5分割して
左の2つ、右の2つでそれぞれちょっと組み替えればおk
20132人目の素数さん:2010/09/15(水) 06:49:28
>>19
おぉ!ちょっとまだちゃんと理解してないが、
この20面体に外接する球面を考えて、
球の中心と領域の境界上の点を結んだ半直線と球面との交点によって
球面上での境界を作ればおkかな?
21132人目の素数さん::2010/09/15(水) 11:41:37
>>5

訂正です。改めて、問題を書きます。

ある数列は、1と2からなる無限数列で、次の条件をみたす。

T 数列の最初の要素は、1である。
U 3っつの連続した1はなく、さらに、2つの連続した2も無い。

V 数列の中の連続した2つの1を単独の(つまり一個の)2に置き換え、
 単独の1はそのまま残し、
 数列にもとにある2を消去すると、
 この操作によって得られる数列は、元の数列と同じ。
 (Uによって、Vに述べられた操作で数列を書き換えることができる)
(1) 具体的に、この数列を表示してみよ。  (5項以上で)
(2)この数列の最初のn項の要素の中にいくつの2があるか。
(3)数列を出力するプログラムを作成してみてください。(おまけ)
--------- 問題 ここまで ---------
22132人目の素数さん:2010/09/15(水) 12:43:07
おまけだけ。

#include <stdio.h>
#define MAX_TABLE 100000
char table[MAX_TABLE];

void Q21(void)
{
  int read = 0, write=1;
  table[0]=1; putchar('1');
  if(table[read++]==2){
    table[write++]=1; putchar('1'); if(write>=MAX_TABLE) return;
    table[write++]=1; putchar('1'); if(write>=MAX_TABLE) return;
  }
  for(;;){
    table[write++]=2; putchar('2'); if(write>=MAX_TABLE) return;
        if(table[read++]==2){
          table[write++]=1; putchar('1'); if(write>=MAX_TABLE) return;
        }
    table[write++]=1; putchar('1'); if(write>=MAX_TABLE) return;
  }
}

int main(void)
{
        int n;
        Q21();
        puts("");
        return 0;
}
23132人目の素数さん:2010/09/15(水) 18:57:49
>>15
f(n) = n*(1+[log_[2](n)]) + 1−2^*(1+[log_[2](n)])
も解となり、2つ答えがあるのも変わっていると思います。
24132人目の素数さん:2010/09/16(木) 00:47:50
分からない問題スレより転載。これどうやって解くんだろ?

図のような、一辺の長さが 1 の立方体4つから構成されるブロックが
充分たくさんある。このブロックを隙間無く組み合わせて作ることの
できる直方体の3辺の長さの組(a, b, c) をすべて答えよ。
ただし、a, b, cは2010≦a≦b≦c≦2012 をみたす正整数であるとする。

  / \
  |\ /|
  |  |  |
/ \|/ \
|\/|\/|
| | | |  |
\|/\|/
25132人目の素数さん:2010/09/16(木) 03:00:34
どうやら、マスコンとやらの問題が出回ってるらしいな。
http://yuzuru.2ch.net/test/read.cgi/venture/1245341407/
http://www.goodfind.jp/2012/special/mathcon.html
26132人目の素数さん:2010/09/16(木) 04:20:08
>>24
>2010≦a≦b≦c≦2012

ワロタ

結局5C2の10通り
2010 2010 2010
2010 2010 2011
2010 2010 2012
2010 2011 2011
2010 2011 2012
2010 2012 2012
2011 2011 2011
2011 2011 2012
2011 2012 2012
2012 2012 2012

のうちどれが可かということでしょ
全偶数なら可能なのは簡単にわかるけど
2011はどうすればいいんだろう。どうやら来年の西暦は素数っぽいね
27132人目の素数さん:2010/09/16(木) 11:13:04
とりあえず体積が4の倍数にならないものはできねぇ
28132人目の素数さん:2010/09/16(木) 13:41:44
>>15

>>8

> 略解:

> n≧2のときを考える。
> D1:= { (x,y)∈R^2|1≦x≦n−1 , 0≦y≦log_[2]x }
> D2:= { (x,y)∈R^2|0≦y≦log_[2](n−1) , 0≦x≦2^y }
> と置く。D1に含まれる格子点の個数がf(n)である。
> D2に含まれる格子点の個数をg(n)と置けば、
> D1∪D2 = { (x,y)∈R^2|0≦x≦n−1 , 0≦y≦log_[2](n−1) } (長方形)
> となるから、f(n)+g(n)=n*(1+[log_[2](n−1)]) となる。g(n)は簡単に
> 計算できてるので詳細は省略する。最終的に、

> f(n) = n*(1+[log_[2](n−1)]) + 1−2^*(1+[log_[2](n−1)]) (n≧2)

> となる。
2928:2010/09/16(木) 13:49:30
誤爆
30132人目の素数さん:2010/09/16(木) 19:11:39
結局、宿題、レポート丸投げスレなわけか。
31132人目の素数さん:2010/09/16(木) 23:41:52
>>2 で反例ありそうって書いたけどやっぱり反例なかった
いつまでも変換を行える 3 つの実数があったと仮定する
このとき T : (x,y,z)→(x+y,-y,z+y) と変換を定義し、要素3の集合上の
置換σ[0],σ[1],...を用意すれば、ある X[n]=(x[n],y[n],z[n]) があって
∀n X[n+1]=σ[n](TX[n]), x[n]+y[n]+z[n]=a>0, y[n]<0, z[n]=max(x[n],y[n],z[n])>0
を満たすように出来る。ここで a=x[0]+y[0]+z[0] とする

f(X[n])=|x+y|+|y+z|+|z+x|+|x|+|y|+|z|とおけば
f(X[n])-f(X[n+1])=f(X[n])-f(TX[n])=2min(a,-y[n]) であり f(X[n]),f(X[n+1])>0 だから
f(X[0]) = f(X[n])+Σ[k<n]{f(X[k])-f(X[k+1])} > 2Σ[k<n]min(a,-y[k]) となるので
n>N → 0<-y[n]<a/3 となる N が存在しなければならない

n>N なら z[n]+y[n]≧(z[n]+y[n]+x[n]+y[n])/2=a/2+y[n]/2 > a/3 > -y[n] であり
z[n]+y[n]=max(x[n]+y[n],-y[n],z[n]+y[n]) > 0 , -y[n]>0 となるので
y[n+1]=x[n]+y[n]<0 , z[n+1]=z[n]+y[n] , x[n+1]=-y[n] となる
よって S : (x,y,z)→(-y,x+y,z+y) とおけば、 n>N なら X[n]=S^(n-N) X[N] となる
しかし S^6 は恒等変換になるので f(X[N]) > f(X[N+6])=f(X[N]) となり矛盾する
以上よりいつまでも変換は行えないことがわかる
32132人目の素数さん:2010/09/16(木) 23:44:22
>>26>>27
2010 2010 2010
2010 2010 2011
2010 2010 2012
2010 2011 2012
2010 2012 2012
2011 2011 2012
2011 2012 2012
2012 2012 2012

が可能性あり

そのうち
2010 2010 2010
2010 2010 2012
2010 2012 2012
2012 2012 2012
は確実に可能、

残るは
2010 2010 2011
2010 2011 2012
2011 2011 2012
2011 2012 2012
これらの吟味というわけですか
33132人目の素数さん:2010/09/18(土) 21:05:36
正六面体を赤青黄緑黒白のうちの何色かで塗り分けるパターンは何通りか。ただし、隣り合う面は異なる色で塗り分けるものとする。
34132人目の素数さん:2010/09/18(土) 22:16:03
>>33
3色 6C3=20通り
4色 6C2*4C2=90通り
5色 6*5*3=90通り
6色 5*3!=30通り
計230通り
35132人目の素数さん:2010/09/18(土) 22:21:04
>>34
正解
36132人目の素数さん:2010/09/18(土) 23:38:40
>>33隣り合う面が同じ色でも良い場合は何通り?
37132人目の素数さん:2010/09/19(日) 02:37:45
>>8 >>15
 1 + [log_2 (k-1)] = (k-1を2進表示したときの桁数)
nの2進表示がm桁だったとする。
 1 + [log_2 (n)] = m,
 2^(m-1) ≦ n < 2^m,

2進表示がj桁の数は 2^(j-1) 個ある。 (← 最高位は1, 下位は任意)
 j*2^(j-1) = (j-1)*2^j - (j-2)*2^(j-1),
∴ (k=2,2^(m-1)) ・・・・・・ = (j=1,m-1) j*2^(j-1) = (m-2)*2^(m-1) + 1,
一方、
  (k=1+2^(m-1),n) m = (n-2^(m-1))*m,
∴ f(n) = n*m + 1 - 2^m,          >>23
も解となり、2つ答えがあるのも変わっていると思います。
38132人目の素数さん:2010/09/19(日) 06:01:00
>>36
6+120+600+1020+630+30=2406になった。正しいかどうかは知らぬ。
しかしその内訳を記すには、3行では足りない
39132人目の素数さん:2010/09/19(日) 09:12:49
>>24
直方体の体積は8の倍数になることが示せた。
残念ながら、辺の長さの偶奇までは出なかった。

とりあえず、奇数辺を含むものは
(2010,2011,2012) (2011,2012,2012)
の2種類に絞れたことになる。
このあとは分からん(^o^)
40132人目の素数さん:2010/09/19(日) 11:34:23
>8の倍数になることが示せた。

どうやって?
41132人目の素数さん:2010/09/19(日) 12:06:02
>>40
39じゃないが横から
空間を市松模様(チェス盤)の立体空間バージョンのように塗り分ける
そこにおいたブロック1個はどうしても白1黒3かまたはその逆の色となる
ブロックが奇数個だとどうしてもそれらが占める空間の色は
白と黒の数が2+4n個分食い違ってしまう
42132人目の素数さん:2010/09/19(日) 12:13:01
なるほど
ソーマキューブの全回答を考えるときにそういう考え方を使ったことがあったわ
43132人目の素数さん:2010/09/19(日) 14:32:53
そもそも奇数辺はありえるのかな。
(2010,奇数,2012) (奇数<2009)
ができればそこから(2010,2011,2012)ができるな。
(2011,2012,2012)も同じく。
44132人目の素数さん:2010/09/19(日) 14:34:04
奇数<2009じゃなくて奇数≦2009だった。
45132人目の素数さん:2010/09/19(日) 17:40:28
ということは例えば奇数辺が素数じゃなければ可能で素数なら不可能といったことはないわけだ。
ひとつできればそれ以上の奇数辺はすべて可能と。
46132人目の素数さん:2010/09/20(月) 16:07:12
>>17-20

cis型
 △
 ▽△▽

trans型(d)
 ▽△
  ▽△

−−−−−−− 鏡

  △▽
 △▽
trans型(L)
47132人目の素数さん:2010/09/24(金) 01:15:54
age
48132人目の素数さん:2010/09/24(金) 01:16:38
age
49132人目の素数さん:2010/09/24(金) 01:36:51
面積の等しい多面体A, Bがある.
Aを鋏で直線的に何回か切り, それを組み合わせて, Bと合同な多角形を作ることはできるか?
50132人目の素数さん:2010/09/24(金) 06:59:07
底辺と高さがそれぞれ等しい二つの三角形は直線分割合同である
面積の等しい二つの三角形は直線分割合同である
いくつかの三角形とそれらの面積の和と等しい面積を持つ三角形は直線分割合同である
任意の多角形とそれと面積の等しい三角形は直線分割合同である
51132人目の素数さん:2010/09/24(金) 22:34:05
a^3+b^3+c^3=d^3を満たす正の整数a,b,c,dの解が無限にあることを証明せよ。
ただし、a,b,cは互いに素であること。
52132人目の素数さん:2010/09/24(金) 23:17:20
n^3 + (3n^2+2n+1)^3 + (3n^3+3n^2+2n)^3 = (3n^3+3n^2+2n+1)^3

(3n^2)^3 + (6n^2+3n+1)^3 + (9n^3+6n^2+3n)^3 = (9n^3+6n^2+3n+1)^3

b^3(a^3+b^3)^3+a^3(a^3-2b^3)^3+b^3(2a^3-b^3)^3 = a^3(a^3+b^3)^3

a^3(a^3-b^3)^3+b^3(a^3-b^3)^3+b^3(2a^3+b^3)^3 = a^3(a^3+2b^3)^3

(3x^2+5xy-5y^2)^3+(4x^2-4xy+6y^2)^3+(5x^2-5xy-3y^2)^3 = (6x^2-4xy+4y^2)^3,

(p^2+16pq-21q^2)^3 + (-p^2+16pq+21q^2)^3 + (2p^2-4pq+42q^2)^3 = (2p^2+4pq+42q^2)^3
53132人目の素数さん:2010/09/26(日) 00:37:44
>>24そろそろ解きたいな。

2*3*nが作れないことは分かった。
(2*3の三次元テトリスを想像して一段目から作っていこうとすればすぐ分かる)

あと2次元版を考えてみた。

■■
の形を敷き詰める問題で
■□□
■■□
の2*3形を作って敷き詰める以外の方法で長方形ができるか試したところ
6*9のときにできることが分かった。
ということは3次元でも2*2*2の敷き詰め以外の方法で直方体を作れるのかな?
54132人目の素数さん:2010/09/26(日) 15:19:23
>>53
>2*2*2の敷き詰め以外の方法で直方体を作れるのかな?
一応できたが、残念ながら4×4×4なので、問題の解決には寄与しない。

一一二二
一五五二
三五六四
三三四四

一七七二
八五七九
八八六六
三十六四

火日七水
日日九九
八十月九
木十十金

火火水水
火日月水
木月月金
木木金金

ところで、
>■
>■■
>の形を敷き詰める問題
で、奇数×奇数が無理なのはどうやって説明すればいいんだ?
単純に偶奇では説明がつかないのだが...。
55132人目の素数さん:2010/09/26(日) 20:00:00
□□■■■
□◇■■■
■◇◇□□
■■◆□□
  ◆◆□□

□□□■■
□□□■■
■■◇■■
■□◇◇◆
  □□◆◆

□□■■■
□□■■■
□□◇◇◇
■■◇◇◇
■       

□□■■■
□◇■■■
■◇◇□□
■■◆◆□
    ◆   

□□■■■
□□■■■
□□◇□□
■◇◇□□
■■  □□
56132人目の素数さん:2010/09/26(日) 21:31:06
>>54
なるほど、上2段下2段は対称にできてるな。
これができるとなると奇数辺もできる気がしてくる。

> で、奇数×奇数が無理なのはどうやって説明すればいいんだ?

これは>>41と同じ説明で・・・て、奇数マスだから白マス黒マスの数が一緒じゃないから駄目なのか。
5*9あたり実はできたりするのかな。

>>55
これはなに?説明きぼん
57132人目の素数さん:2010/09/26(日) 22:37:43
>>55は2次元版の5*5を試行錯誤したものか?
マスの数が3の倍数じゃない時点で無理なのは確定なんだが。
58132人目の素数さん:2010/09/26(日) 23:23:48
二つくっつければ5*9ができる
59132人目の素数さん:2010/09/27(月) 00:07:15
あ、なるほどね
60132人目の素数さん:2010/09/27(月) 06:05:31
>>55
5×9できるのか。自分の根気が足りなかっただけだなorz
61132人目の素数さん:2010/10/01(金) 00:10:51
age
62132人目の素数さん:2010/10/01(金) 03:50:51
ドゥーン!!  -=・=-  -=・=-

ようこそ、呪いのスレへ。
実は今君に呪いをかけたんだ。
このレスをみてしまうと君はもう一生、異性を拝めなくなる。そんな呪いだ。
もちろん童貞なら一生童貞のまま人生を終える。処女もしかり。
災難だと思って諦めてくれたまえ。
仏の顔もって言うしね、謝って許してもらおうとも思っていない。
だけど一つだけ呪いを解く方法があるんだ、それは・・・

「 男湯に女性を入れてる浴場名を報告スレ 」
でgoogle検索してこのスレに行って

「 >>1はくそすれたてんなキチガイ 」
って書き込むんだ。

では、健闘を祈るよ
63132人目の素数さん:2010/10/01(金) 06:37:26
 □
□□□
 □

十字型のペントミノだけを組み合わせて長方形作れるかな
64132人目の素数さん:2010/10/01(金) 07:15:37
長方形の角に置けない。終了。
65132人目の素数さん:2010/10/01(金) 07:41:17
すごい!
66132人目の素数さん:2010/10/01(金) 22:29:10
>>24
http://www.goodfind.jp/2012/special/img/math-con6.jpg
この出題者コメント見ると
(2010,2011,2012) (2011,2012,2012)
もできないくさいな。
67132人目の素数さん:2010/10/02(土) 10:14:10
正方形の各辺と辺の中点に8個の点A〜Hを置きます。
AHG
B□F
CDE

各点から2点を選びそれを結ぶ直線を引きます。
それを繰り返し、正方形内に4本の直線が引かれたとき、一筆書きできる形になるのは何種類あるでしょうか。
68132人目の素数さん:2010/10/02(土) 10:19:02
>>67
繰り返しの過程で
例えばAB、BCなどと続いた場合は?
またABが2回以上繰り返された場合は線が二本あると看做すのか?

と思ったが、それ以前に「直線」4本なら一筆書き自体無理だな
69132人目の素数さん:2010/10/02(土) 18:20:11
「正方形内に」とあるんだが、それでも無理か?


70132人目の素数さん:2010/10/02(土) 18:21:13
> 各辺と辺の中点に8個の点

正方形の 角4つと、辺の中点4つの、計8点という意味なのかな?
71132人目の素数さん:2010/10/02(土) 20:47:21

>>68

逆に訊きたいんですが、ユークリッド平面上で、異なる位置にある2点間を線分として厳密に直線を引いた場合、複数の直線として識別することが可能な直線は引けるんでしょうか?

>>70

> 各辺と辺の中点に8個の点
正方形の 角4つと、辺の中点4つの、計8点という意味なのかな?

その通りです。>>68,70さんご指摘ありがとうございます。訂正し再度。



正方形を描き、この4角と4辺の中点に8個の点A〜Hを置きます。
AHG
B□F
CDE

各点から2点を選びそれを結ぶ直線を引きます。
それを繰り返し、正方形内に4本の直線が引かれたとき、一筆書きが可能な図形になるのは何通りあるでしょうか。

ただし、正方形の辺および、一度引かれた直線上には重複して線をひいてはならない(直線が交差するのは良い)。また、一筆書きは既に描かれている正方形を含めて行うこと。
72132人目の素数さん:2010/10/02(土) 20:56:29
直線が通っていい点が重複しないと無理じゃね?
っていうか,線分だろ?無限遠ワープとかありなんか?
73132人目の素数さん:2010/10/02(土) 20:58:57
たぶん「直線」だと無限長で端がないからだと思う
74132人目の素数さん:2010/10/02(土) 21:17:44
はぁ?無限長で端がないからだと思う?
そんな「直線」が2本あったら一筆書きどころの話じゃねーんだけど
7573:2010/10/02(土) 21:22:02
>>74
すまん、>>73は出題者に対してなんだ
説明不足だったな
76132人目の素数さん:2010/10/02(土) 21:55:14
あぁ、そうい事ですか。
「直線」じゃなくて「線分」ですね。

77132人目の素数さん:2010/10/03(日) 01:07:45
>>69
もちろん。
正方形内に4本だからこそ、辺上に何本も引かれても操作が繰り返されるだろう。
そのことを言っているんだけどね。

>>71
ユークリッドだの非ユークリッドだの持ちだす前に
直線と線分の区別くらいつけないと。
78132人目の素数さん:2010/10/03(日) 17:48:35
自作問題。あまり面白くはない。


一辺がnの正三角形Dがある。これを、一辺が1の
小正三角形に分割する(n^2個の小正三角形になる)。

各小正三角形の頂点(全部で(n+1)(n+2)/2個の頂点がある)を
黒と白で塗り分ける。ただし、どの小正三角形に対しても、
その3つの頂点のうち黒で塗られている頂点が1個だけである状態にする。

以下、nは3の倍数とする。

(1)Dの3つの頂点を全て黒、または全て白で塗った場合、
残りの頂点を上手く塗れば、題意の塗り分けが可能であることを示せ。

(2)Dの3つの頂点のうち、黒で塗った頂点が1個または2個の場合は、
残りの頂点をどのように塗っても、題意の塗り分けが不可能であることを示せ。
79132人目の素数さん:2010/10/03(日) 18:47:35
鏡像関係の2パターンしか塗り分ける方法がないな…
80132人目の素数さん:2010/10/03(日) 19:25:05
>>77
> 正方形内に4本だからこそ、辺上に何本も引かれても操作が繰り返される

操作とは何のこと?

81132人目の素数さん:2010/10/03(日) 19:41:47
ああ違う、てっぺんの小三角形の頂点を決めれば、だった
3通りあるじゃん…
82:2010/10/03(日) 23:57:26
非負整数からなる数列a[n]が、
a[n+2]=|2a[n+1]-a[n]|(n=1,2,・・・)
を満たしているとする。
a[m]=a[m+1]となるmが存在するa[1],a[2]の条件を求めよ。
83132人目の素数さん:2010/10/04(月) 15:29:41

12個の金貨のうちに、贋金が1個あり、本物とは重さが微妙に違う。

上皿天秤を3回だけ使い、贋金を見つけ出し尚かつそれが本物とは
軽いのか重いのかを判定する方法がある。どんな方法か?
84132人目の素数さん:2010/10/04(月) 15:50:24
頻出問題となにか違うのか?
85132人目の素数さん:2010/10/04(月) 16:00:07
ただし
・天秤測定は左右どちらの皿にどの金貨を乗せるかをあらかじめ指定しなくてはならない。
・測定の結果は3回の測定が終わってから左右どちらが下がったか(または釣りあったか)が
 「1度目右、2度目左、3度目釣りあった」のように3回分まとめて報告される。
(つまり直前の結果によって乗せる金貨を変えたりはできない。)

てのを付け加えたら多少難易度は上がるが、それでも頻出問題だな。

金貨13枚の場合は、3回の測定ではできないことの証明。   とか。

それも頻出か。
86132人目の素数さん:2010/10/04(月) 17:07:08
>>85
変えた条件で3回だけで軽いか重いかまでの判定が済むのか?
87132人目の素数さん:2010/10/04(月) 17:15:09
すむんじゃないかな。


すくなくとも
天秤に載せる金貨の組み合わせは有限なのだから
すべてについて調べれば必ず正解は見つかる。(不可能な場合も含めて)
そういう意味ではあまりおもしろくない問題だ。
コンピュータの登場はこういったパズルをある意味面白くなくしてしまったな。

88132人目の素数さん:2010/10/04(月) 17:19:26
直前に釣りあったなどして、本物だとわかっている金貨を基準に
重い軽いを決定するというテクニックが使えなくなるので
難易度が上がるというわけだ。
89132人目の素数さん:2010/10/04(月) 17:50:25
あらかじめ指定するんなら無理じゃないのか?
90132人目の素数さん:2010/10/04(月) 17:53:06
無理だと思うなら、無理なことを証明すればいい。
91132人目の素数さん:2010/10/04(月) 17:55:05
無理すぎてつまらんな。シラケた。
92132人目の素数さん:2010/10/04(月) 17:58:14
え? できるだろ? 
93132人目の素数さん:2010/10/04(月) 17:59:02
もしできないとしたら、そのできないことの証明が無理だって言ってるんじゃないのか?
94132人目の素数さん:2010/10/04(月) 18:21:41
できることを保障してもらえないとやる気が起きない
というとうな意味なんじゃないかな。
「こんなに考えたのに、できないってのはないだろうよ」
という感じの
95132人目の素数さん:2010/10/04(月) 18:22:37
つまり、方法を見つけたときにはカタルシスを感じるが
できないことの証明にはそれを感じないとでもいうのか
96132人目の素数さん:2010/10/04(月) 18:52:46
とりあえず、14個ではできない証明。

・全ての金貨は少なくとも1度天秤に乗せる必要がある。
 (量っていない金貨が贋物であった場合、重い軽いが特定できない)
・全ての計測で天秤がつりあってしまうことは許されない。 
 (全てが本物になってしまう)
・1回の天秤測定では3通り「右下がり、左下がり、つりあう」の結果が得られる。
 従ってn回の測定で区別できる事象の種類は最大でも3^n-1通り。 3回ならば26通り。

・14枚の金貨のどれか1枚が重いまたは軽いのだから、区別しなくてはならない事象は28通り

 26 < 28 なので 14枚は不可能。


97132人目の素数さん:2010/10/04(月) 18:55:19
続いて 13枚が不可能な証明。

・13枚の金貨のどれか1枚が重いまたは軽いのだから、区別しなくてはならない事象は26通り
 (これは 3回の測定での上限26通りには合致)
・秤の両腕には、同じ枚数の金貨を乗せなくてはならない。
  (贋物が重い場合、どの程度重いかは不明なため、少ない枚数のほうに贋物があったとき、どちらに傾くかが不定になる。)
・2回の計測で区別できる事象は、最大で3^2の9通りを超えない。
・1回目の測定には、 片腕には1〜6枚のうちいずれかの枚数を載せることになる。

1回目の計測について
片腕に1枚乗せて、釣りあってしまった時、 可能性が残る事象は、11枚のうちどれかが重いまたは軽いの22通り。 
片腕に2枚乗せて、釣りあってしまった時、 可能性が残る事象は、9枚のうちどれかが重いまたは軽いの18通り。 
片腕に3枚乗せて、釣りあってしまった時、 可能性が残る事象は、7枚のうちどれかが重いまたは軽いの14通り。
片腕に4枚乗せて、釣りあってしまった時、 可能性が残る事象は、5枚のうちどれかが重いまたは軽いの10通り。
片腕に5枚乗せて、右に傾いてしまった時、 可能性が残る事象は、右5枚のどれかが重いまたは左5枚のどれかが軽いの10通り。 
片腕に6枚乗せて、右に傾いてしまった時、 可能性が残る事象は、右6枚のどれかが重いまたは左6枚のどれかが軽いの12通り。
いずれにせよ、残り2回の計測で区別できる上限を超えてしまう。
また、1回目の計測では、どの金貨についても真贋の情報が全くないので、ぜったいに釣り合わない様な組み合わせや
絶対に右に傾かないような組み合わせの計測をすることはできない。

以上により。 13枚では不可能。
98132人目の素数さん:2010/10/05(火) 02:10:19
>>95
>>87が結論でしょ

結局は工夫の余地が少ない作業にしかならないわけで
それを嬉々としてやれるかどうかだけ
楽しもうと思えば十分楽しめる
99132人目の素数さん:2010/10/05(火) 15:35:15
>>98
いや、>>89を受けていっているので、そいう話ではない。
100132人目の素数さん:2010/10/05(火) 15:38:47
> 結局は工夫の余地が少ない作業にしかならないわけで 

ここはすこし違うと思う、
過去に比べて、かなり大きな量のものでも
工夫することなく、作業のみでも回答や証明ができてしまうようになっただけで
工夫してそれを簡素にする余地はまだいくらでものこっている。


101132人目の素数さん:2010/10/05(火) 15:44:04
12枚でできたと思うんだけど。 解答いる?
それとも、まだ考えたい人がいる?
102132人目の素数さん:2010/10/05(火) 16:13:12
>>101
あらかじめ指定しても出来る?
出来ないと思うのだが。

3回とも釣り合う場合があったらダメ。
2回釣り合うことがあり得たらダメ。
……
103132人目の素数さん:2010/10/05(火) 16:15:22
> 2回釣り合うことがあり得たらダメ。 

んなことはない。
104132人目の素数さん:2010/10/05(火) 16:27:39
>>103
じゃあ、2回釣り合って、1回釣り合わなかったとき、どうやって判別するのか教えてくれ。
105132人目の素数さん:2010/10/05(火) 16:36:59
答えを書かずにそこだけを取り出して説明などできないが

たとえばA〜Iの金貨で
DG-HI 釣りあった
BC-EF 釣りあった
AB-CD が右に傾いた  

これだと Aが軽いことが特定できる。
AB-CD が左に傾いたなら Aが重いことが特定できる

もちろん、これは2回つりあって1度傾いたときに特定できることを
示しているだけで、他の傾き方をした場合に他の金貨を特定できる
ような例にはなっていない。
106132人目の素数さん:2010/10/05(火) 16:39:52
>>104
上の例で納得してもらえないなら、あとは答を検証してもらうしか
説明する方法が思いつかない。

というか、逆に、なぜ2度つりあってしまったら
特定できないと考えているのかを聞かせてくれ。
それを否定することはできると思う。
107132人目の素数さん:2010/10/05(火) 17:55:19
私も、解になりそうな組合わせをいくつか見つけて
そのうちの1つは実際に上手くいくことが確かめられた。

一応、完全ではないがある程度の方針はある
(その方針以外の方法で見つけられるかどうかは不明である)のだが、
実際に検証することなく
見つけたそれらの組合せが解になっていること
を示す手立てが今のところ見つかってないので
解になっているかどうかは、いちいち検証しなければならず
非常に面倒で時間がかかる。

その為、他の組合せが解になってるかどうかは確かめてない。
108132人目の素数さん:2010/10/05(火) 18:04:28
>>83
こういうのって、3進法で表現して云々〜ってやると
判別方法が簡単に見つかるんだよね確か
>>85の条件で考えると、上手く行くか分からないけど
109132人目の素数さん:2010/10/05(火) 19:10:06
>>106
3枚ずつ以下を乗せることがある場合、それが釣り合ってしまうと、
それ以外の6枚以上に贋物があることになり、他の2回では判別出来ない。
5枚ずつ以上乗せることがある場合、それが釣り合わないと、
この10枚以上に贋物があることになり、他の2回で判別出来ない。
従って、3回とも4枚ずつ乗せることになる。
釣り合わないときが1回だけしかない場合、どうやって判別するの?
110132人目の素数さん:2010/10/05(火) 19:30:21
>>106じゃないが
3回とも、片腕には4枚ずつ乗せることになる。
これはおk。

例えば1回目、2回目が釣り合って、3回目だけ釣り合わないとき
1回目と2回目に乗せた金貨は全て本物だとわかる。
この時点で、11枚が本物だとわかるのであれば
残りの1枚が贋物だとわかり、3回目の結果から
贋物が軽いのか、重いのかが判別できる。
(これは答えを見つけるためのヒントでもある)
111薄氷の湖 ◆ZJwTrwL.xg :2010/10/05(火) 22:05:06
1〜43までの中から6個を選ぶロト6で、6個の当たり数字に連続した数字が含まれている(例えば、2,14,15,27,31,38は、14,15が連続している)確率を求めよ。
112132人目の素数さん:2010/10/05(火) 22:06:04
見たことある気がするなあ
113132人目の素数さん:2010/10/05(火) 22:08:55
1-(38C6/43C6)か?
114132人目の素数さん:2010/10/05(火) 22:10:49
○○○○○○○○○○○○○○○○○○○○○○○○○○○○○○○○○○○○○
115132人目の素数さん:2010/10/06(水) 02:03:56
>>109
> 釣り合わないときが1回しない場合、どうやって判別するの?

判別ってのは、何の判別なのかがわからないのだが   
こちらで勝手に推測すると、
1) 贋物の金貨がどれなのか。 
2) 贋物が重いのか軽いのか。

1) について
釣りあわなかった1回の測定だけに秤に乗っていて、 それ以外の測定では乗っていない金貨が贋物。
つまりそのような金貨が2枚あってはならないので、そうならないように計画する必要がある。

2) について
一見すると、下がった側に思い贋物があるのか、上がった側に軽い贋物があるのかわからないようにみえるかもしれない。
が、じかし1)の条件を満たしていれば、そのようなことはおこらない。

ほとんど解答を言っているような感じになってきたな。



116132人目の素数さん:2010/10/06(水) 02:05:26
× 下がった側に思い贋物があるのか
○ 下がった側に重い贋物があるのか
117132人目の素数さん:2010/10/06(水) 05:59:26
42C5/43C6
118132人目の素数さん:2010/10/06(水) 23:35:07
>>83

知る人ぞ知る ---- アインシュタインが1時間かけて解いたと言う、有名なクイズだぞ。

皆んな、気バレ! (^o^)
119132人目の素数さん:2010/10/07(木) 00:30:25
どうしてこの問題に限って
やたらかまってオーラがすごいんだろう?
120132人目の素数さん:2010/10/07(木) 02:17:29
理論がないから?
いわゆる情報量というやつとの関連で説明される試行回数の下限と発見的手法というのが気持ち悪いンだろうな。
121132人目の素数さん:2010/10/07(木) 02:28:42
122132人目の素数さん:2010/10/07(木) 03:17:29
>>121
パズル系じゃね?
123132人目の素数さん:2010/10/07(木) 07:53:13
>>120
理論的に出来ると思うよ。
124132人目の素数さん:2010/10/07(木) 08:51:48
>>122
確かに。
125M_SHIRAISHI:2010/10/08(金) 12:32:09
>>83

件(くだん)の問題には、微妙に異なる「正解」が 少なくとも4つ在る。
126132人目の素数さん:2010/10/08(金) 12:35:29
もっとむちゃくちゃいっぱいあるだろ
127132人目の素数さん:2010/10/08(金) 12:36:58
載せ方を先に指定するという限定バージョンの方が考えやすいような気もする。
128132人目の素数さん:2010/10/08(金) 17:55:18
可能なパターンの組み合わせが、ずいぶん減るからね。
129132人目の素数さん:2010/10/08(金) 20:15:38
4回で39枚の場合、5回で120枚の場合、6回で363枚の場合
一般にn回で(3^n -3)/2枚の場合は
たぶんできた!(少なくとも1つの解は見つけられる)
(面倒なんで検証はしてない。一応理論もあるけど、未完成なので正解かどうかは厳密には保証できない)

一般の枚数の場合はどうなんだろ?
例えば
3回で4枚〜12枚のどの場合でも判別できるなら
一般にm枚のときn回で判別可能な可能性は高いと思う。
ただし、nは次を満たす自然数;
(3^(n-1) -3)/2 < m ≦ (3^n -3)/2
130132人目の素数さん:2010/10/08(金) 21:40:20
未検証でもいいから証明を書くなりしないとただのチラ裏になってるぞ。
間違ってたっていいじゃん。
131132人目の素数さん:2010/10/08(金) 22:47:00
取り敢えず
t回でn枚の判別できるとき(解の1つがわかっているとき)に
t+1回で2n枚、3n枚を判別する方法(これは正しいことが保証できる)

a1,a2,…,an,b1,b2,…,bnの2n枚の場合:
{ai,bi}を一塊Aiとみなして
A1,…,Anをt回でn枚の判別する方法で測定すれば
贋物が含まれている組{ak,bk}の軽重(すなわち贋物の軽重)が判明する。
また
・左[a1,a2,…,an]_右[b1,b2,…,bn]
と測定すれば
{a1,a2,…,an}と{b1,b2,…,bn}の軽重が判明する。



a1,…,an,b1,…,bn,c1,…,cnの3n枚の場合:
{ai,bi,ci}を一塊Aiとみなして
A1,…,Anをt回でn枚の判別する方法で測定すれば
贋物が含まれている組{ak,bk,ck}の軽重(すなわち贋物の軽重)が判明する。
また
・左[a1,a2,…,an]_右[b1,b2,…,bn]
と測定すれば
{a1,…,an}と{b1,…,bn}の軽重が判明する。
(つり合う場合は{c1,…,cn}の中に贋物があることが判明する)
132132人目の素数さん:2010/10/08(金) 22:52:47
残念なのは 2n枚以下、3n枚以下 ではなく 2n枚、3n枚ぴったりというところだなと思った。

3n-1枚のときできるかどうかはわからない。
133べ ◆GMPNeMrEog :2010/10/09(土) 00:48:34
>>82に書いたが、

非負整数からなる数列a[n]が、
a[n+2]=|2a[n+1]-a[n]|(n=1,2,・・・)
を満たしているとする。
a[m]=a[m+1]となるmが存在するa[1],a[2]の条件を求めよ。
134132人目の素数さん:2010/10/09(土) 01:01:06
問題が人気がないのは、興味の方向やレベルの違いや既知の問題かも含めて
それを楽しめる人がいないということ

135べ ◆GMPNeMrEog :2010/10/09(土) 01:02:52
んなことはどうでもいい。出題しているだけだ。
136132人目の素数さん:2010/10/09(土) 01:05:56
なるほど、答えないのも自由だしな。
137132人目の素数さん:2010/10/09(土) 01:29:06
自演で答える自由もある
138132人目の素数さん:2010/10/09(土) 02:00:43
その問題は解けたには解けたが、答えが
たくさんありすぎて面白くない。
139132人目の素数さん:2010/10/09(土) 03:16:39
>>131の続き
各金貨を載せる回数とそのパターンに注目する。
例えば、いくつかの金貨a,b,c,…に対して、3回の測定で贋物とその軽重が判別できるとする。
このとき、各測定において、秤の両腕には同じ枚数の金貨を載せなくてはならない。

金貨aが
1回目:右の載せる、2回目:左に載せる、3回目:載せない
となっているとする。以後このようなことを
金貨aのパターンは(右,左,×)であると表現する。

もし、金貨bがaと同じパターン(右,左,×)であるとすると
aかbが贋物であるとき、どちらが贋物であるか判断できない。

また、別の金貨cがaと対称なパターン(左,右,×)であるとすると
aかcが贋物であるとき、どちらが贋物であるか判断できない。

aと同じパターンでも、対称なパターンでもないパターン
つまり(右,左,×),(左,右,×)以外のパターンを、aと独立なパターンと呼ぶとする。

以上から、次が導ける:
いくつかの金貨a,b,c,…に対して、贋物とその軽重が判別できるとき
各測定において秤の両腕には同じ枚数の金貨が載っていて、
それぞれの金貨のパターンは互いに独立でなければならない。


もしその逆、各測定において秤の両腕には同じ枚数の金貨が載っていて、
それぞれの金貨のパターンは互いに独立ならば贋物とその軽重が判別可能である …(※)
ということが証明されれば、一般の枚数の場合の解を見つけやすくなるし
与えられた測定法が解になっているかどうかの判定が非常に簡単になる。
140132人目の素数さん:2010/10/09(土) 03:20:58
(※)が正しいなら
3回で12枚の場合の解の1つから、4回で39枚の場合の解の1つを構成できる。

3回で12枚の場合の解で、測定回数が3回のうち
1回載せる金貨は3枚(3パターン)
2回載せる金貨は6枚(6パターン)
3回載せる金貨は3枚(3パターン)
であった(とする)。
>>131の方法で、4回で36枚の場合の解を構成すると
1回載せる金貨は 3枚( 3パターン) =3
2回載せる金貨は12枚(12パターン) =3+3+6
3回載せる金貨は15枚(15パターン) ==== 6+6+3
4回載せる金貨は 6枚( 6パターン) ======== 3+3
…(1)
で、秤の両腕の枚数は等しい。

測定回数が4回のとき
1回載せるパターンで互いに独立となるのは高々 4(=C[4,1]*1)パターン
2回載せるパターンで互いに独立となるのは高々12(=C[4,2]*2)パターン
3回載せるパターンで互いに独立となるのは高々16(=C[4,3]*4)パターン
4回載せるパターンで互いに独立となるのは高々 8(=C[4,4]*8)パターン
…(2)

(2)のうち、(1)のどのパターンとも独立であるパターン3つ
1回載せるパターン、3回載せるパターン、4回載せるパターン
を1つずつ選び、秤の両腕の枚数が等しくなるように調整して4回で36枚の場合の解と合わせれば
4回で39枚の場合の解が構成できる。
(同様に5回で120(=39*3+3)枚の場合の解も構成できる)
141M_SHIRAISHI:2010/10/09(土) 08:27:23
>>138 

微妙に、ほんの少しw「微妙」に異なる「正解」が4つ or 5つもあるのだ。
余の従兄(いとこ)の一人(彼は、今、琉球大学で数学を教えている)が発見した。
これには、驚いた。ヽ(^。^)ノ 余は、正解は余が解いたものたった1つと
想っていたからだった。

* 話は変わるが、諸君は、どうして、余のように、「実名とE-mailの宛先」を書かぬのだ?
恐いのか? 卑劣かつ臆病ではないか! 恥を知れ!!!!!!!

http://www.age.ne.jp/x/eurms/Ronri_Kaikaku.html
142132人目の素数さん:2010/10/09(土) 08:31:39
だから、微妙に違うのを区別したら無茶苦茶いっぱいあるっつうの
143132人目の素数さん:2010/10/09(土) 10:13:42
何を持ってして、「異なる」といっているのかがよくわからん。
144132人目の素数さん:2010/10/09(土) 10:26:34
>>141
揉め事を嗅ぎつけて飛んでくる蝿が
145132人目の素数さん:2010/10/09(土) 10:34:10
>>141
エムシラ、黙れ。
シンゴは怒っているぞ。
146べ ◆GMPNeMrEog :2010/10/10(日) 00:56:34
>>138
とりあえず興味を持ってくれる人を見つけるため、単に「条件」と書いた。
答えがたくさん出てきた時に、「一般的な式で表せないだろうか」というときめきに似たものを感じなかっただろうか?
これからもその「ときめき」みたいなものを持ち続けていて欲しい。
では、注文を聞こう。

この問題で聞きたいのは「条件が一般的な式で表せるか?」という事。
表せないならその理由を証明して欲しい。
ついでに数学者?が解けなかった問題ね。
147132人目の素数さん:2010/10/10(日) 00:56:45
>>135
答えが知りたいのなら東京出版に聞けばいいんじゃないのw
148べ ◆GMPNeMrEog :2010/10/10(日) 00:59:19
>>147
なぜ東京出版に聞けばわかると言い切れるのか。
149132人目の素数さん:2010/10/10(日) 01:28:08
>>146
逆算するだけ。
各項が1と-1から成る任意の数列{λ[n]}に対して

x[1]=x[2]=1
x[n+2]=2x[n+1]+λ[n]*x[n]

と定義した数列{x[n]}の任意の隣り合った二項x[k],x[k+1]と
任意の非負整数cに対して

a[1]=c*x[k+1]
a[2]=c*x[k]

と置いたとき、かつそのときのみ題意が成り立つ。
(数列{x[n]}は広義単調増加になることが
簡単な帰納法から示せるので、上のように置いた
a[1],a[2]は非負になっている。)
150132人目の素数さん:2010/10/10(日) 01:44:35
>>146
やっぱこの問題(というより出題者か)
かまって度が異様に高すぎる
151132人目の素数さん:2010/10/10(日) 01:53:37
ベは糞
152132人目の素数さん:2010/10/10(日) 13:34:17
31 名前: 名無しステーション [sage] 投稿日: 2010/10/10(日) 13:29:18.05 ID:hM5lzrGb
Q アタック25でn問正解したのに最終獲得パネルが0枚だった。nの最大値を求めよ
153132人目の素数さん:2010/10/10(日) 13:54:47
ルール確認しとかないと解けん

5×5のオセロ状なのは憶えてるが
それとは別に途中で他人のマス目を奪える特別ルールがあった気がする
・そのルールの発動の回数やタイミングは?
・取るマス目は25個全ての中から自由に選べるのか?
154132人目の素数さん:2010/10/10(日) 14:10:14
155132人目の素数さん:2010/10/10(日) 14:17:07
ついで
アタック25シミュレータ
ttp://kim-my.hp.infoseek.co.jp/java/attack25/a25.html

「21問」(四隅以外とって四隅だけ他の奴に取られる)では
残ってしまうパネルがあるのでだめ
156132人目の素数さん:2010/10/10(日) 14:27:37
思った以上に複雑なルールだった
戦略性抜きで、nの最大値を実現するために
参加者がありえない愚かな選択をするのもありなのね
157132人目の素数さん:2010/10/10(日) 14:45:36
>>155
アタックチャンスを使っても21問じゃダメ?
例えば
赤が1,5,6,21,25以外のパネルを得て、アタックチャンス問題になる。
緑が正解して、25を得て、13をアタックチャンスの狙い目に指定する。
赤が正解して6を得た後、緑が1,5,21を得て、最後に緑が13を得れば
赤は21問正解したけどパネルは全て緑になるはず。
158132人目の素数さん:2010/10/10(日) 16:15:15
アタックチャンスで自らのパネルを指定することも出来たんだっけ?
159132人目の素数さん:2010/10/10(日) 16:30:54
なんだろう、真面目にアタック25で数学してる
このシチュエーションが若干シュールに感じられるw
160132人目の素数さん:2010/10/10(日) 16:37:07
一般化できるかな

アタックn^2 (n:自然数)でk問正解したのに獲得枚数が0だった。
k=n^2-4を証明できるか?
161132人目の素数さん:2010/10/10(日) 16:38:07
違った
kの最大値はn^2-4
162132人目の素数さん:2010/10/10(日) 20:45:44
それは明らかに無理だろw
163132人目の素数さん:2010/10/10(日) 21:41:59
どのあたりで明らか?
164132人目の素数さん:2010/10/10(日) 23:33:41
nが大きかったらどう考えても無理
165132人目の素数さん:2010/10/10(日) 23:38:21
たとえばどう考えたら無理だってわかる?
166132人目の素数さん:2010/10/11(月) 00:03:01
あきらかに
167132人目の素数さん:2010/10/11(月) 00:04:32
晃蟹
168132人目の素数さん:2010/10/11(月) 00:07:13
つか、k=1のときも2のときもおかしいじゃないか
169132人目の素数さん:2010/10/11(月) 00:12:33
よし、これからは自明と書く代わりに「晃蟹」と書こう!w
170132人目の素数さん:2010/10/11(月) 00:31:26
nの偶奇の方が問題になりそうな気が

アタック25だとパネルの「真ん中」の13を
アタックチャンスでとれば(とられれば)21問
正解でパネル0枚というのがありえるんだよね。

でもアタック36の場合「パネルの真ん中」がないから
そこで取りこぼしが生じるような気がする

線形代数とか群論とか数論で誰か鮮やかに証明できないのかねw?
171132人目の素数さん:2010/10/11(月) 00:31:52
やってみな
172べ ◆GMPNeMrEog :2010/10/11(月) 01:36:56
>>133は、条件を簡単な式で表せるか?
が問題ね。
それが出題した数学者?でも解けなかった。

>>149は簡明とは言えないだろう。
173132人目の素数さん:2010/10/11(月) 01:43:14
>>172
「簡単な式」の数学的な定義は?

>>149は簡明とは言えないだろう。
これはお前の単なる感想に過ぎない。
174べ ◆GMPNeMrEog :2010/10/11(月) 01:55:19
>>173
ただ問題文を変えただけにしか見えないんだがw
175132人目の素数さん:2010/10/11(月) 02:14:29
>>174
質問の答えになってない。

「簡単な式」の数学的な定義は?

>>149は簡明とは言えないだろう。
これはお前の単なる感想に過ぎない。
176132人目の素数さん:2010/10/11(月) 14:51:43
この大数の宿題の最後をベは自分で解いたのかよ
177132人目の素数さん:2010/10/11(月) 17:21:16
正三角形に3つの円を重ならないように入れて、3つの円の面積の和を最大にしなさい。
178132人目の素数さん:2010/10/11(月) 19:12:20
正三角形の1辺の長さを1として
3つの円の面積の和の上限はpi/4
179べ ◆GMPNeMrEog :2010/10/12(火) 00:53:43
>>175
x+xより2xの方が簡明だよな?
こういう規則をいちいち書いていけないという事w
それで簡明だと思ってるなら、もっと簡明にする事ができない
証明を頼むわ。

>>176
この問題は宿題ではないがな
180132人目の素数さん:2010/10/12(火) 00:56:23
お、「べ」さんだ。
こんばんは。そういえば年齢はいくつですか?
181132人目の素数さん:2010/10/12(火) 00:58:11
149は x[n+1]+x[n+1] ではなく 2x[n+1]と書いてある。
182132人目の素数さん:2010/10/12(火) 01:10:00
>>177
正三角形の1辺の長さを1として

内接円(半径1/√12)を描き、隙間にその1/2サイズの円を描くと
 (π/12)(1 + 1/4 + 1/4) = π/8,
となる。(下限)
183132人目の素数さん:2010/10/12(火) 02:22:16
>>179
話が平行線だな。
それが簡明かどうかは、見方によって変わるだろ。
使用する「項」の種類で見れば「x+x」の方が簡明と言える。

・「x+x」は和の演算を使っており、使用する項は「x」の1種類。
・「2x」は積の演算を使っており、使用する項は「x」と定数項「2」の2種類。

従って、この見方においては「x+x」の方が簡明。
あと、演算の複雑さで見ても、積よりも和の方が簡明だと見れば「x+x」の方が簡明。

アセンブラでは乗算より加算の方が速いから、「2x」より「x+x」で
計算した方が得することもいっぱいある。


もういいから、「簡明にせよ」じゃなくて、
「nの多項式で表せ」とか具体的に指定しろよ。
184132人目の素数さん:2010/10/12(火) 15:04:33
見方によって変わることが理解できない人にとって
具体的でないという指摘はやはり理解できないことがある
本人はまったく明示的にたった一つのことを示していると考えているからだ

185132人目の素数さん:2010/10/12(火) 21:35:35
大数はこの問題の解答掲載するかどうか分からないから、「べ」の素晴らしく「簡明」な解答に期待するわw
どんな解答か楽しみだな
186132人目の素数さん:2010/10/12(火) 22:20:12
正三角錐に3つの球を重ならないように入れて、3つの球の表面積の和を最大にしなさい。
187132人目の素数さん:2010/10/12(火) 22:34:53
したらえーやんw
188べ ◆GMPNeMrEog :2010/10/12(火) 23:38:58
>>183
x+y+x+yを簡単にせよ
とかいう問題を見たことないか?

アセンブラなんて取り出してきたらどんな答えでも
言い訳できてしまうだろ。

ならnの多項式で表せるかどうか答えよ。
表せない場合その理由も。
189132人目の素数さん:2010/10/12(火) 23:53:35
>>188
>x+y+x+yを簡単にせよ
>とかいう問題を見たことないか?
お話にならない。
そういう問題は中学・高校で見たことがあるが、
いい加減なもんだよ。「簡単」を厳密に定義しても
中・高では理解されないし、計算力に重点を置いた問題だから、
「簡単」の定義を明記せずにお茶を濁してるだけ。

例えば、「xy+y」と「(x+1)y」のどっちがより簡明なのか。
俺が高校生だった頃は、どっちもマルだったけどな。

>アセンブラなんて取り出してきたらどんな答えでも
>言い訳できてしまうだろ。
題意を満たす任意のa[1],a[2]が求められるアルゴリズムが
記載されていれば、何でも答えになる。
「簡明な答え」を要求したかったら、何を以って「簡明」とするのか
出題者がキチンと定義しなければ話にならない。「アルゴリズムの複雑さ」
を定義するのと同じようなもんだな。しかもお前は

「これ以上簡明に出来なければ、そのことを 証 明 せ よ 」

と言っているわけだ。「簡明」の定義が無いのに
証明なんかできっこない。こちらが勝手に「簡明」を
定義しても、お前は不服に思うだろうしなww
190183:2010/10/12(火) 23:53:53
>>188
私が間違っておりました。申し訳ございません。
β様に極めて失礼な態度をとってしまったことを猛省致します。
191べ ◆GMPNeMrEog :2010/10/12(火) 23:59:30
>>189
まぁ分からないならという事で問題として作っているだろう。

nの多項式で表せるかどうか答えよ。
表せない場合その理由も。


>>190
許す
192132人目の素数さん:2010/10/13(水) 00:01:15
「べ」ってやつ自演ひどすぎるだろ頭悪すぐるw
193183:2010/10/13(水) 00:01:51
>>191
ありがとうございます。
β様の心の広さはあまりに広大で、容易に私の心を内包してしまうようです。
今後ともよろしくお願致します。
194132人目の素数さん:2010/10/13(水) 00:21:41
>>191
>nの多項式で表せるかどうか答えよ。
そうだよ、こういう風に具体的に指定しないと
証明もクソもないよ。

解答:
nの多項式では表せない。以下で、このことを証明する。

題意を満たすa[1],a[2]がnの多項式で表せるとする。
すなわち、xの多項式f(x),g(x)が存在して、

(a[1],a[2])=(f(n),g(n)) (n∈N)

だけが題意を満たすa[1],a[2]だとする。任意の非負整数cに対して
(a[1],a[2])=(c,c)は題意を満たすから、各cに対して、
(f(n),g(n))=(c,c) を満たすn∈Nが存在することになる。
このようなnをcごとに1つずつ取り出してn_cと書くことにする。
f(n_c)=c だから、c≠c' ならばn_c≠n_c'である。
すなわち、n_0,n_1,n_2,…は全て異なる。
一方で、f(n_c)=c=g(n_c) だから、xについての方程式
f(x)−g(x)=0 はx=n_c (c=0,1,2,…)を解に持つことになる。
n_cは全て異なるのだから、f(x)−g(x)=0は無限個の解を
持つことになり、よってxの多項式としてf(x)とg(x)は完全に
一致しなければならない。このとき

(a[1],a[2])=(f(n),f(n)) (n∈N) …(*)

となるが、(a[1],a[2])=(3,1)もまた題意を満たすのに(*)では
表せないので矛盾する。
195べ ◆GMPNeMrEog :2010/10/13(水) 00:27:36
数学者の関君(前数学板で見たあの人と同姓同名?)が解けなかったんだから、
何か怪しいな。
196132人目の素数さん:2010/10/13(水) 00:36:22
>>195
素直になろうョ
そやないと恥をかくのは貴方だョ
197べ ◆GMPNeMrEog :2010/10/13(水) 00:39:05
いや、しかし数学者で解けなかったのだから、
多項式以外の簡明な表し方ができるかどうかで迷ったのか・・・?
198132人目の素数さん:2010/10/13(水) 00:41:13
そういう書き込みをしていると、そのうち某おっさんが絡んでくるョ
199べ ◆GMPNeMrEog :2010/10/13(水) 00:45:03
(・ョ・ )
200132人目の素数さん:2010/10/13(水) 16:34:26
自作問題。簡単です…


n≧3とする。n次対称群をSnと置く。奇置換τ∈Snを
1つ取って固定する。また、σ_i ∈Sn を

σ_i = ( i i+1 i+2 ) ( i=1,2,3,…,n-2 )

と置く(σ_i は長さ3の巡回置換)。
Snの任意の元は、τ及びσ_1,σ_2,…,σ_{n-2}の積で
表せることを示せ。
201132人目の素数さん:2010/10/13(水) 19:56:06
√S[n]=a[n],a[1]=1の一般項を求めよ
昔の大数の宿題らしいが俺はギブ
202132人目の素数さん:2010/10/13(水) 22:22:11
Sn-Sn-1=an=Sn^.5
(1-^.5)Sn=Sn-1
(1-^.5)^(n-1)Sn=S1=1
an=(1-(1-^.5)^(n-1))Sn
Sn=(1-^.5)^-1Sn-1=(1-^.5)^-(n-1)a1
an=(1-(1-^.5)^(n-1))(1-^.5)^-(n-1)a1
=Sn-a1=an^2-a1
an=(1+/-(1+4a1)^.5)/2
203132人目の素数さん:2010/10/13(水) 22:36:53
>>202
S[n] - S[n]^.5 = (1-^.5)S[n]

??
204132人目の素数さん:2010/10/13(水) 22:39:10
Sn-Sn-1=an=Sn^.5
(1-^.5)Sn=Sn-1
(1-^.5)^(n-1)Sn=S1=1
an=(1-^.5)Sn=(1-^.5)(1-^.5)^-(n-1)S1=(1-^.5)^-(n-2)S1
b-b^.5=a
(a-b)^2=b
b^2-(2a+1)b+a^2=0
b(a)=((2a+1)+/-(4a+1)^.5)/2
an=b^(n-2)(1)

205132人目の素数さん:2010/10/13(水) 22:52:25
Sn-Sn-1=an=Sn^.5
(1-^.5)Sn=Sn-1
(1-^.5)^(n-1)Sn=S1=1
an=(1-(1-^.5))Sn=(1-(1-^.5))(1-^.5)^-(n-1)S1
=((1-^.5)^-(n-1)-(1-^.5)^-(n-2))S1
b-b^.5=a
(a-b)^2=b
b^2-(2a+1)b+a^2=0
b(a)=((2a+1)+/-(4a+1)^.5)/2
an=(b^(n-1)-b^(n-2))(1)


206薄氷の湖 ◆ZJwTrwL.xg :2010/10/13(水) 23:35:46
これは中学生向けだけど。
一辺aの正方形ABCDにおいて、A,B,C,Dのそれぞれを中心とした半径aの4つの円のすべてが重なる部分の面積を求めよ。
207132人目の素数さん:2010/10/14(木) 01:27:20
その問題10回は見たな・・・
208132人目の素数さん:2010/10/14(木) 15:52:11
>>201
条件たらなくね?
209132人目の素数さん:2010/10/14(木) 18:17:37
>>177
途中まで計算をしてみた。
三角形の一辺の長さをlとし、
三角形の3点をABCをA(0, √3*l/2), B(-l/2, 0), C(l/2, 0)とする。
y軸上に円C1の中心O1があるものとし、その半径をaとする。
x軸と辺ACと円C1に接する最大の円をC2とし、その中心をO2、半径をbとすと
O1(0, √3*l/2-2*a)、O2(l/2-√3*b, b)となり、線分O1O2の距離はa+bであるから
三平方の定理により
3*a^2 + 2*a*b + 3*b^2 - 2√3*l*a - 2√3*l*b + l^2 = 0 …@
が成立する。
@を媒介変数θで表示すると
a = l/4*(cos(θ) + √2*sin(θ) + √3)
b = l/4*(cos(θ) - √2*sin(θ) + √3) …A
3つの同じ大きさの円となる場合をaの最小値とし、三角形の内接円になる場合を
aの最大値とすると、
l/4*(1 + √3) <= a <= √3*l/6 …B
となるから、Aのθの定義域は
α <= θ <= π …C
ただしαは
cos(α) = -5*√3/9
sin(α) = √6/9
を満たす角度。3つの円の面積の和をSとすると
S = π(a^2 + 2*b^2) …D
= π*l^2/16*(3*cos(θ)^2 - 2*√2*sin(θ)*cos(θ) + 6*sin(θ)^2 + 6*√3*cos(θ) - 2*√6*sin(θ) + 9)
S(α) = 11*π*l^2/108
S(π) = 3*π*l^2*(2-√3)/6
よって
S(α) < S(π)
210209:2010/10/14(木) 20:52:48
訂正
S(α) ≒ 0.320
S(π) ≒ 0.316
から
S(α) > S(π)
211211:2010/10/14(木) 20:57:32
2=1+1
212209:2010/10/14(木) 21:30:52
dS/dθ = π*(2*√2*(sin(θ)^2 - cos(θ)^2) + 6*sin(θ)*cos(θ) - 6*√3*sin(θ) - 2*√6*cos(θ))/16
dS/dθ(α) = -5*√2*π/54
dS/dθ(π) = π*(√6-√2)/8
dS/dθ(α) < 0、dS/dθ(π) > 0からCの範囲で極小値が存在する。
213132人目の素数さん:2010/10/15(金) 22:18:52
関数y=x2乗のグラフに点(2、-1)から引いた接線の方程式を求めよ
お願いしますm(__)m
214132人目の素数さん:2010/10/15(金) 22:26:20
そんなクソ程も面白くない問題をここに書き込めるお前の方が面白いよ
215132人目の素数さん:2010/10/15(金) 22:27:03
関数y=x2乗の
グラフに
点(2、-1)
から
引いた
接線の
方程式を
求めよ

(2、ー1)を通って、
y=x^2と交わって
そのとき交差しない(yより上か下にある)

y=x^2は上はコンケーブな曲率なので、直線はしたにしかない。

だから

y+1=a(x-2)
y=a(x-2)-1=<x^2
x^2-ax+2a+1>=0
(x-a/2)^2+(2a+1-a^2/4)>=0
x=>a/2+/-(2a+1-a^2/4)^.5
a^2-8a-4=0
a=4+/-2(5)^.5
x=2+/-5^.5,x^2=9+/-4*5^.5
y=2(2+/-5^.5)(2+/-5^.5-2)-1=+/-4*5^.5+10-1=9+/-4*5^.5
y=(4+/-2*5^.5)(x-2)-1
216132人目の素数さん:2010/10/15(金) 22:35:45
>>214
どのあたりがどう面白いんだ?
217132人目の素数さん:2010/10/15(金) 22:41:07
そんなの一々つっかかる所じゃないだろうが
サラッと流せ、そんなもん
お前コミュ障か?
218132人目の素数さん:2010/10/15(金) 22:50:36
あちこちのスレにまき散らしてるな
219132人目の素数さん:2010/10/15(金) 22:57:26
マンデルブロート関数のグラフに点(2、-1)から引いた接線の方程式をすべて求めよ
220132人目の素数さん:2010/10/16(土) 01:08:26
>>217
価値観の共通な相手としか話ができないのを
コミュニケーション障害とは言わないのか?
221132人目の素数さん:2010/10/16(土) 01:08:48
>マンデルブロート関数のグラフに
この辺の投げやり感どうにかしろ
222221:2010/10/16(土) 01:12:50
>>220
くっそお前クソ面白んないレスを間に挟むなやハゲが
俺の面白いツッコミがちょっと立ち位置悪い感じになってるやんけ
223132人目の素数さん:2010/10/16(土) 01:25:34
>>222
東京03のコントかよw
224132人目の素数さん:2010/10/16(土) 01:35:06
>>220
言わない。 
通常コミュニケーション障害では価値観が共通かどうかと関係なく障害がある。

価値観が違う人とだけコミュニケーションがとれないのは
基本的なコミュニケーション能力そのものではなく、
相手が知らないことを、やさしく順を追って話すとか
自分が知らないことを話されても、その説明を求めまた注意深く聞くなどの
論理的な会話の組み立てがうまくできない場合が多い。
225132人目の素数さん:2010/10/16(土) 01:45:43
たとえば>>224である

>論理的な会話の組み立てがうまくできない場合が多い。
226132人目の素数さん:2010/10/16(土) 01:49:27
>>225
それを受け取れないのも含むんだよ。
227132人目の素数さん:2010/10/16(土) 01:51:20
>>224
> 価値観が違う人とだけコミュニケーションがとれないのは

これは日本語ではない。
228132人目の素数さん:2010/10/16(土) 01:59:31
> これは日本語ではない。 

これも日本語ではない。
229132人目の素数さん:2010/10/16(土) 02:00:56
そもそも日本語であることの定義は?
230132人目の素数さん:2010/10/16(土) 13:10:37
この文は日本語ではない
231132人目の素数さん:2010/10/16(土) 13:28:53
Ceci n'est pas une langue Japonaise.
232132人目の素数さん:2010/10/16(土) 13:48:46
What is the definition of being Japanese language at all?
233132人目の素数さん:2010/10/16(土) 13:53:00
Qual e la definizione dei giapponesi?
234132人目の素数さん:2010/10/17(日) 06:50:22
>>219
亡くなられたぞ
235132人目の素数さん:2010/10/20(水) 22:00:26
箱入り娘の状態数は全部で幾つか
二状態の最短経路の最大値は幾つか
236132人目の素数さん:2010/10/20(水) 23:59:29
暇人がシラミつぶしを面白がる問題ねw
237209:2010/10/23(土) 18:09:41
Fの範囲でdS/dθ(t) = 0となるtは以下の4次方程式の1つの解で
√2*(√3 - 1)*t^4 - 6*(√3 + 1)*t^3 + 6*√2*t^2 - 6*(√3 - 1)*t - √3 - 1 = 0
t = (√((-(214*√2+42*√3+123*√6+52)/2+√((764662*√2+1540836*√3+441768*√6
+2674719)/54))^(1/3)-((214*√2+42*√3+123*√6+52)/2+√((764662*√2+1540836
*√3+441768*√6+2674719)/54))^(1/3)+(177+96*√3)/6)+√(-((-(214*√2+42*√3
+123*√6+52)/2+√((764662*√2+1540836*√3+441768*√6+2674719)/54))^(1/3)
-((214*√2+42*√3+123*√6+52)/2+√((764662*√2+1540836*√3+441768*√6+2674719)
/54))^(1/3)+(177+96*√3)/6)-2*(-(177+96*√3)/4-(√2*(351*√3/4+156))/√((
-(214*√2+42*√3+123*√6+52)/2+√((764662*√2+1540836*√3+441768*√6+2674719)
/54))^(1/3)-((214*√2+42*√3+123*√6+52)/2+√((764662*√2+1540836*√3+44176
8*√6+2674719)/54))^(1/3)+(177+96*√3)/6))))/2+(√6+2*√2)*3/4
t ≒ 15.317482703001
tがこの値をとるときのθをβとおくと
β ≒ 3.01120792441003
S(β) ≒ 0.289476604944306*l^2
以上からθがCの範囲内にあるとき、三角形の面積の和Sは
θ = βのとき最小値、S(β) ≒ 0.289476604944306*l^2
θ = αのとき最大値、S(α) = 11*π*l^2/108 ≒ 0.319977029532293*l^2
をとる。
238209:2010/10/23(土) 18:19:57
>>237の前に以下を追加
t = tan(θ/2)とおくと
cos(θ) = (1 - t^2)/(1 + t^2)
sin(θ) = 2*t/(1 + t^2) …E
(5*√2 + 3*√6)/2 <= t < +∞ …F
となるから
dS/dθ(t) = π*l^2*(2*√2*(√3 - 1)*t^4 - 12*(√3 + 1)*t^3 + 12*√2*t^2 - 12*(√3 - 1)*t - 2*(√3 + 1))/(16*(1 + t^2)^2)
239132人目の素数さん:2010/10/24(日) 20:15:31
ABCDEという五人の男がいる。
彼らは皆等しく論理的で頭がよい。
100枚の金貨がある。
Aから順に「誰々に何枚誰々に何枚…」という具合に金貨の分配法の提案をする。
過半数の同意を得られればその分配法に決まり終了する。
過半数の同意を得られない場合は提案者は殺され、次の提案者の番になる。

問題.
Aはどのように提案すれば1枚でも多くの金貨を得られるでしょうか。

質問があったらしてね。
240132人目の素数さん:2010/10/24(日) 20:16:16
あげます
241132人目の素数さん:2010/10/24(日) 20:28:26
ABCDE
五人がいる。
100枚の
金貨がある。
Aから
順に
「誰々に何枚誰々に何枚…」
という具合に
金貨の分配法
の提案をする。
過半数の同意を得られればその分配法に決まり
過半数の同意を得られない場合は
提案者は殺され、
次の提案者の
番になる。

問題.
Aは
どのように
提案すれば
1枚でも多くの
金貨を
得られるで
しょうか。
242132人目の素数さん:2010/10/24(日) 20:32:24
98/3=32 A
34 BC
243132人目の素数さん:2010/10/24(日) 20:38:21
>>241
有名問題「海賊の多数決」「海賊ゲーム」みたいだが
色々条件が不足し過ぎて解けない
244132人目の素数さん:2010/10/24(日) 20:42:10
3人で山分けを提案すればいい。この手は最初じゃないとできない。
3人目からは2人で山分け。
245132人目の素数さん:2010/10/24(日) 20:51:27
でも賢いCは3人目になるまで拒絶する。だからAが生き残ることはない。
DEはCになるまでどちにしても拒絶する。DEはDになると過半数はないので
Cで合意するしかない。
246132人目の素数さん:2010/10/24(日) 20:53:27
DEで2人が合意すれば過半数だけど、Eが裏切ればEのひとりじめ。だからDはCに合意する。
247132人目の素数さん:2010/10/24(日) 21:07:27
だからどのみちもらえないBEに33やりAは34もらう。
それかBEに1、Aは98でもいい。
248132人目の素数さん:2010/10/24(日) 21:16:50
Bにあげる提案をするとBは合意しないんだよ
4人になってから自分の好きなように提案できるし
249132人目の素数さん:2010/10/24(日) 22:11:33
>>248
DE2人の場合Eは反対し成立しない
3人の場合Dは合意されないと0なので1で賛成、よってC99、D1、E0
4人の場合Cは反対、Eは合意されないと0なので1で賛成、Dは合意されないと1なので2で賛成。よってB97、C0、D2、E1
5人の場合Bは反対、Cは合意されないと0なので1で賛成、Eは合意されないと1なので2で賛成、よってA97、B0、C1、D0、E2

これでどうだろうか
250132人目の素数さん:2010/10/24(日) 23:23:48
>>239
「論理的で頭が良い」だけだと
倫理観だとか金貨入手は彼らにとってのぞましいことなのかなどは分からないが、

半数は過半数に入るのか(超過はしてないから入らないだろう)
同意を得るとなると自分は頭数に入るのか(入らないだろう)となると
Aは自分を除く4人中3人以上の同意を得ない限り殺されるわけだな。

Aは殺される可能性が高いが、
分配条件にもAの死が言及されていてパラドックスが成立したらどうするんだろう
251132人目の素数さん:2010/10/25(月) 11:22:24
DE二人になったら、Eが反対すれば過半数を得られないからDが殺されてEの独り占め。
なので、CDE3人になったら、C100、D0、E0でもDは同意せざるを得ない。
つまり、3人になったら、D、Eは0になると予想されるので、Bの提案で1個でももらえれば同意することになる。
従って、BCDE4人になったらB98、C0、D1、E1で過半数の3票が得られる。
つまり、4人になったらC0、D1、E1になると予想されるので、
AはCDEの3人のうち2人にC0、D1、E1よりも1枚多く分配すれば過半数の3票を得られるが、
このとき1枚多く分配した2人以外は0でよい。
よって、A97、B0、C1、D2、E0か、A97、B0、C1、D0、E2のどちらかで3票得られる。
252132人目の素数さん:2010/10/25(月) 11:33:20
>>251は分配者も1票持つ場合。持たない場合は、
CDE3人になったら、Eは反対すれば独り占め出来ることになるので、
BCDE4人の時点でCDは1枚でももらえれば同意することになる。
従って4人になるとB98、C1、D1、E0が予想されるので、A95、B0、C2、D2、E1で3票得られる。
253132人目の素数さん:2010/10/25(月) 15:17:54
結局のところ、提案する順番でもめることになり、成立しないな。
254132人目の素数さん:2010/10/25(月) 22:01:23
自分の得より自分が生きることを優先するとする 
また無益な殺生はしないとする(反対しても賛成しても自分の利益が変わらないときは賛成する)…@
提案順序はABCDEとする

Eにとっては反対してもその案が可決されたらその通りになるのだから
提案者が死んで人数が死んだほうが自分のもらえる量が増える
Eは常に反対する
だからEに対する分け前は基本みんな0で提案
100にしたら多分Eは賛成するがその他が否決
D的には
DE状態になってしまったらEが反対→D死亡
になるから
DはC状態を反対したくない→DはCを賛成する
それを知るCはどのような提案をしても(例えばC100D0E0)
可決されると知ってるからABを反対(Cに提案権を来させる)
そうなると自部の分け前が0になって困るDは
極力Cに提案権を与えたくない
よってDはA、Bを賛成
Bはこれらの推測から
Dは必ず賛成しC、Eは普通なら反対するが
これでは自分は死んでしまうので
C、Eのどちらかを賛成させたい
C、Eに100与えれば@より可決してもらえるので
Bはどちらかを100にする
(B0C100D0E0)か(B0C0D0E100)
を提案する
つまりBに提案権が来てもBは自分の利益が0になる提案しかできない
これと@からBは絶対Aに賛成する
よってAは絶対B、Dから賛成されるため必ず過半数
よって(A100B0C0D0E0)でおk
255132人目の素数さん:2010/10/25(月) 22:23:13
>>254
> C、Eに100与えれば@より可決してもらえるので
> Bはどちらかを100にする
> (B0C100D0E0)か(B0C0D0E100)
> を提案する
おかしい。
Eはここで可決しないと0になるから(君も前段でそう言っている)、1個でももらえれば賛成する。
256132人目の素数さん:2010/10/25(月) 22:27:46
× Eは
○ D、Eは
257132人目の素数さん:2010/10/25(月) 22:36:35
> Eにとっては反対してもその案が可決されたらその通りになるのだから 
> 提案者が死んで人数が死んだほうが自分のもらえる量が増える 

これ 単純にそう結論付けていいのか?
258254:2010/10/25(月) 22:39:52
>>255
ほんとだ スマン

>(前略)
>それを知るCはどのような提案をしても(例えばC100D0E0)
>可決されると知ってるからABを反対(Cに提案権を来させる)
この時点でEが絶対反対するていう仮定が@より崩されてるんだなorz
259254:2010/10/25(月) 22:41:41
>>257
最終的にEに提案権が来るまで反対し続けると仮定したつもりだったんだけど
なんやかんやで矛盾しましたorz
260254:2010/10/25(月) 23:07:04
要は
Eは絶対反対と仮定すると
Dが絶対賛成
Cが絶対反対→Eが絶対賛成で矛盾して
Eが絶対賛成→Dが絶対反対→Cが絶対反対→Dが絶対賛成ってなって
どっちにしろ矛盾するから場合分けが必要な感じ
261べたっち:2010/10/26(火) 01:00:16
これと同じ問題。前スレッド立ってて、自力で解いた記憶あるわ・・・w
(98,0,1,0,1)になるんだよね。
262132人目の素数さん:2010/10/26(火) 01:00:50
無益な殺生はしない場合

もし、Eが提案する番まで来たなら、Eは100枚独り占めになる

もしDの番になったら、E100D0と提案する時、またその時のみ可決される
(Eは無益な殺生をしないから)

Cの番。もしCが否決されたらDは獲得数0枚。よって無益な〜の精神よりDは0でも賛成するので
E0D0C100と提案すれば、反対はEだけで可決

Bの番。もしBが否決されたらCが可決される。Cの案ではEもDも獲得数0枚なので
Bの提案で0枚だとしても、無益〜より賛成する故
E0D0C0B100と提案すれば、反対はCのみで可決


Aの番。Aが否決されたら上のBの案が可決。
EDCは0枚でも無〜より賛成してくれるので、
E0D0C0B0A100と提案すれば、反対はBのみで可決
263べたっち:2010/10/26(火) 01:31:38
>>262
Eが提案する番まで来たなら、Eは100枚独り占めになる
なら、Eは全部反対するでしょ。
264べたっち:2010/10/26(火) 01:32:44
というかこれ前解いたから、
問題ば微妙違ってなければ>>261が正解なんだが
265132人目の素数さん:2010/10/26(火) 01:51:58
>>264
似たような問題で、海賊が多数決する問題があるが、
条件が違うので解が異なる(考え方は同じ)

>>263
Eは「自分の番まで来たら、独り占めできる」と知ることができるが、同時に
「Cの番まで来たら、その案は可決される」
「Bの番まで来たら、その案は可決される」
等「自分の番が来ないこと」も知ることになる。
それら全ての情報を元に賛否を判断すると考えれば、全部に反対するとは限らない。
266べたっち:2010/10/26(火) 01:56:43
>>265
いや、だからEは反対しなければいずれにせよ0なんだから、
全部反対するでしょ。
267132人目の素数さん:2010/10/26(火) 02:03:22
>>266
無益な殺生をしないという条件(設定)の下では、正に
>いずれにせよ0なんだから
反対しないんだよ
268132人目の素数さん:2010/10/26(火) 02:14:13
>>266
>>254
> また無益な殺生はしないとする(反対しても賛成しても自分の利益が変わらないときは賛成する)…@
の仮定ででいずれにしろ0なら賛成

条件が同じ時に否定する場合の解は>>251でしょ
269べたっち:2010/10/26(火) 02:17:06
>>267-268
反対しなければ、いずれにせよ0。
反対した場合、「100になる可能性がある」から反対するという意味で言ってるんだが。
270132人目の素数さん:2010/10/26(火) 02:22:40
>>269
3人の場合C100,D0,E0でC、Dの賛成で可決されるから、E100の可能性は0だよ
271べたっち:2010/10/26(火) 02:31:01
>>270
そう。それをまず言うべき。

この設定だと、これが答えだな。
272132人目の素数さん:2010/10/26(火) 02:37:26
262に書いてあることだろ…
273べたっち:2010/10/26(火) 02:40:08
>>272
同じ問題だと勘違いしていた「べたっち」さんに対して、
どのような点が違うのか?を説明するべきだったと、
「べたっち」さんは言ってるんじゃないかな?
274べたっち:2010/10/26(火) 02:42:06
まぁ潔く言い訳はやめて、きちんと読んでなかったのを認めるわ。




β崩壊!
275ワシは山猫軒 ◆MuKUnGPXAY :2010/10/26(火) 02:44:32
>>274
β崩壊って何や? ちゃんと説明してみ! 媒介する力とその基本粒子は何や?


276132人目の素数さん:2010/10/26(火) 05:36:54
β崩壊 
べーたあほうかい
べーた阿呆かい
277132人目の素数さん:2010/10/26(火) 09:36:13
そうで スカ ? ツリー
そうです イカ釣り
278132人目の素数さん:2010/10/26(火) 13:00:16
イカとスカ一緒にすんなや
279132人目の素数さん:2010/10/27(水) 02:28:50
猫に小判、まで読んだ。
280132人目の素数さん:2010/10/27(水) 03:10:39
これは ひどい
281132人目の素数さん:2010/10/27(水) 05:42:24
○にイを書いたら?に…
282ウザい猫 ◆MuKUnGPXAY :2010/10/31(日) 21:43:50
>>279

283132人目の素数さん:2010/11/01(月) 16:10:50
問題
F(n,x,y)=F(n,F(n,x,y+1),y+1) (y<nのとき)
F(n,x,y)=x+y (y≧nのとき)
となる関数F(n,x,y)を定義します。
F(n,0,0)をnの式で表してください。
284132人目の素数さん:2010/11/01(月) 21:54:26
(2^n)n 一睨み
285132人目の素数さん:2010/11/02(火) 18:05:30
nを自然数とする。
nを3つの1以上の整数の和で
表す場合の数は何通りあるか。
286132人目の素数さん:2010/11/02(火) 18:23:30
>>285
3つの数字の並び順は区別すんの?
287132人目の素数さん:2010/11/02(火) 18:37:43
しません
288132人目の素数さん:2010/11/02(火) 18:48:00
区別しないのは面倒なんじゃなかったっけ?
289132人目の素数さん:2010/11/03(水) 00:50:44
区別するならコンビネーションですぐ

しない場合は2数が同じ時と3数が同じ時に場合分けして重複度で割ることになるのかな
290132人目の素数さん:2010/11/03(水) 10:27:49
区別しない場合、Σ[k=1, |n / 3|]|(n - k) / 2|
291132人目の素数さん:2010/11/03(水) 19:08:30
>>285-287
 (1/12)n^2 + (1/2){n/2} - (3|n を除いて 1/3),

(略解)
nをk個の自然数の和で表わす方法の数を「制限付き分割数」とか云うらしい・・・・・
 q_k (n)     (1≦k≦n)
 「1」を含むもの …… q_(k-1) (n-1)
 「1」を含まないもの …… 各項を1減らしたものと同数なので q_k (n-k)
∴ q_k (n) = q_(k-1) (n-1) + q_k (n-k)
      = 納L=1, min(n-k,k)] q_L (n-k),

 q_1(n) = 1,
 q_2(n) = [n/2],
 q_3(n) = (1/12)n^2 + (1/2){n/2} - (3|n を除いて 1/3),
ただし {x} = x - [x],
292290:2010/11/03(水) 23:03:40
訂正
Σ(k=1, [n/3])([(n-k)/2]-k+1)
293132人目の素数さん:2010/11/04(木) 00:09:02
nを2以上の整数とする.
袋の中に赤球1個,白球n個の計n+1個の球が入っており,赤球に0が,白球にそれぞれ0,1,2,…,n-1が書かれている.
この袋から無作為に1個の球を取り出し,その球に書かれている数字がk(k=0,1,2,…,n-1)のとき,
その球を袋に戻さずにさらにk個の球を袋から無作為に取り出す.
袋の中に赤球が残らない確率を求めよ.
294132人目の素数さん:2010/11/04(木) 00:27:21
>>274
 弱い相互作用
 ウィーク・ボゾン(W^±, Z^0)


>>283
F(n,x,y) = x + 2^(n-y)・n,  (y≦n)
F(n,x,y) = x + y,     (y≧n)
295132人目の素数さん:2010/11/04(木) 00:49:24
>>285
区別しない場合、場合の数をf(n)とすると
n = 6*m+3 のとき、f(n) = 3*m^2 + 3*m + 1
n = 6*m+4 のとき、f(n) = 3*m^2 + 4*m + 1
n = 6*m+5 のとき、f(n) = 3*m^2 + 5*m + 2
n = 6*m+6 のとき、f(n) = 3*m^2 + 6*m + 3
n = 6*m+7 のとき、f(n) = 3*m^2 + 7*m + 4
n = 6*m+8 のとき、f(n) = 3*m^2 + 8*m + 5
ただし m >= 0
296132人目の素数さん:2010/11/04(木) 01:34:57
映画「コンタクト」で恒星ベガから送られてきた素数のパルス信号。
映画では「知的生物に違いない」といい感じに話が進んで行ったが、
素数ってやっぱり基本性質だから、物理現象で偶然、ということもあるかと思う。
ということで…

110111011111011111110…
のように、素数回連続した1の間に0が挟まる数列に関して
なにか面白い一般式が見つからないだろうか。
297132人目の素数さん:2010/11/04(木) 01:38:15
もうすでに
298132人目の素数さん:2010/11/04(木) 02:23:43
>>293
1/2
299132人目の素数さん:2010/11/04(木) 02:24:57
>>298
すげー
どうやったの?
天才!!
300132人目の素数さん:2010/11/04(木) 02:45:03
n+1個の玉から1個の玉を引く確率は1/(n+1)
n個の玉からk個の玉を引いたときに赤玉がなくなる確率はk/n
求める確率をp(n)とおく

p(n)
=1/(n+1)+Σ(k=0,n-1)[k/n(n+1)]
=1/(n+1)+(n-1)/{2(n+1)}
=1/2
301132人目の素数さん:2010/11/04(木) 02:46:59
>>300
あなんだそれか
残念
302132人目の素数さん:2010/11/04(木) 03:24:39
デレツンww
303132人目の素数さん:2010/11/04(木) 03:25:13
残念とかいうなw
304132人目の素数さん:2010/11/04(木) 17:31:15
赤い玉がなくならない場合を□で、
赤い玉がなくなる場合■で表す。
□の個数は赤い玉がなくならない場合の数、
■の個数は赤い玉がなくなる場合の
数を表すことになる。

□□□□□□□(最初に0と書かれた玉を引いた場合)
□□□□□□■(最初に1と書かれた玉を引いた場合)
□□□□□■■
□□□□■■■
□□□■■■■
□□■■■■■(最初に(n-2)と書かれた玉を引いた場合)
□■■■■■■(最初に(n-1)と書かれた玉を引いた場合)
■■■■■■■(最初に赤い玉を引いた場合)

■は全体の半分を占めているので
赤球がなくなる確率は1/2
305132人目の素数さん:2010/11/04(木) 18:58:34
1/(n+1)+Σ(k=0,n-1)[k/n(n+1)] 
とあんまり変わらん。
306132人目の素数さん:2010/11/04(木) 23:53:12
Σ(k=0,n)[k・nCk]を求めよ。
307132人目の素数さん:2010/11/05(金) 00:24:23
実質はシグマ使って(n+1)約分して終わりだが
>>304のように絵で見せると面白い上に小中学生にも分かってもらいやすくていいね
308132人目の素数さん:2010/11/05(金) 01:53:57
それはあれだ、ツルカメ算みたいなもんだ。
連立一次方程式を長方形で図示して計算するようなカンジ。
やってることの本質は一緒。


同じ図形を使うのでも、
ttp://aozoragakuen.sakura.ne.jp/suuron/node45.html (相互法則の証明 のところ)
こういうのは本当に面白い。「図形で言い換えただけ」以上のものを感じる。
309132人目の素数さん:2010/11/05(金) 02:07:05
でもやってることの本質は一緒

整数問題は認識しにくいことと
図形が規則性や周期性を明示的に使ってるように見えないせいだな
310132人目の素数さん:2010/11/05(金) 03:01:50
>>307
まあそれは逆に言うと、特定のマス数では図示しやすいが
一般にnで表すようなことは図では難しく、式のほうが理解しやすい
(もちろん式を扱いなれていることが前提で)ということだわな。
311132人目の素数さん:2010/11/08(月) 01:14:54
>>306
 k・C[n,k] = n!/{(k-1)!(n-k)!}
      = n・C[n-1,k-1]    (k>0)
を代入する。
 n・2^(n-1)
312132人目の素数さん:2010/11/09(火) 16:56:42
xyz空間中に半球V:{(x,y,z)|x^2+y^2+z^2≦1,z≧0}があり、
点光源Pがz軸上の点(0,0,1)にある。
VとPがxy平面上につくりだす影を求めよ。
313132人目の素数さん:2010/11/09(火) 16:59:00
<<312
訂正
「半球V:{(x,y,z)|x^2+y^2+z^2≦1,z≧0}」は
「半球V:{(x,y,z)|(x-1)^2+y^2+z^2≦1,z≧0}」
の間違いでした。
すみません
314132人目の素数さん:2010/11/09(火) 17:49:35
a・cos(θ)+b・sin(θ)=cos(2θ)、a・sin(θ)-b・cos(θ)=2・sin(2θ)のとき
(a+b)^(2/3) + (a-b)^(2/3) はθに無関係な一定の値をとることを示し、その値を求めよ。
315132人目の素数さん:2010/11/09(火) 19:40:07
>>313
哲学的な問題だなぁと思ったら
訂正で一気に宿題臭くなった件
316132人目の素数さん:2010/11/10(水) 03:03:39
>>314

 a = cosθ・cos(2θ) + 2sinθ・sin(2θ) = (cosθ)^3 + 3cosθ・(sinθ)^2,
 b = sinθ・cos(2θ) - 2cosθ・sin(2θ) = -3sinθ・(cosθ)^2 -(sinθ)^3,

∴ a±b = (cosθ干 sinθ)^3 = {(√2)cos(θ ± π/4)}^3,

∴ 2
317132人目の素数さん:2010/11/10(水) 03:17:24
>>313
x = y^2
318132人目の素数さん:2010/11/10(水) 22:04:23
>>317
おしい
319132人目の素数さん:2010/11/12(金) 14:46:41
>>312
は東大の過去問
320132人目の素数さん:2010/11/13(土) 08:07:16
log7を計算しなさい
321132人目の素数さん:2010/11/13(土) 08:37:16

山櫻桃(さんおうとう)
log2 = 0.3 0 10

死なない
log3 = 0.4 7 7 1

(住所不明・「jk」さん)


蜜を入れ
log2 = 0.3 0 1 0

死なない
log3 = 0.4 7 7 1

毒はふたある
log4 = 0.6 0 2 1

毒くれ
log5 = 0.699 0

梯子一つ
log7 = 0.845 1

(住所不明・「Harry」さん)
322132人目の素数さん:2010/11/13(土) 08:38:02
0点よくないよ桑名さん
logπ = 0 . 4 9 7 1 4 98 7 3

0点はよ来い
log7 = 0 . 8 4 5 1

0点泣くわい!泣くわな!
log2π = 0 . 7 9 8 1 7 9 8 7
323132人目の素数さん:2010/11/13(土) 08:38:49
オッ サンを 縛 ろうとしたのは、何でも 納屋に 運んで 食おうと 苦心する 人。
log1=0 log2=(0.)30 log3=(0.)48 log4=(0.)60 log5=(0.)70 log6=(0.)78 log7=(0.)85 log8=(0.)90 log9=(0.)94 log10=1

324132人目の素数さん:2010/11/13(土) 08:50:07
* sin(α+β)= sinαcosβ + cosαsinβ 咲いたコスモスコスモス咲いた
* cos(α+β)= cosαcosβ - sinαsinβ コスモスコスモス咲いた咲いた
1マイナスたんたん分のたーんたん
325132人目の素数さん:2010/11/13(土) 08:52:03
tan(α+β)=(tana+tanb)/(1-tanatanb) 1マイナスたんたん分のたーんたん
326132人目の素数さん:2010/11/13(土) 08:55:23
If a + b – c = d, and if a – b + c = e, then a =

(A) .5(d + e)
(B) d – e
(C) 2d + e
(D) d + e
(E) 2(d + e)
327132人目の素数さん:2010/11/13(土) 08:56:29
What is the value of a ?

(1) (3/2)a + b = 6
(2) (2/3)b + a = 4
328132人目の素数さん:2010/11/13(土) 08:57:44
Question 3

If eight pounds of macadamia nuts, priced at $6.00 per pound, are combined with twelve pounds of brazil nuts, priced at $5.00 per pound, what is the per-pound price of the resulting mixture?

(A) $5.25
(B) $5.40
(C) $5.50
(D) $5.75
(E) $5.80

Question 4

If A, B, C, and D are all positive numbers, is the value of A – B greater than the value of C – D ?

(1) A + D = B + C
(2) A and B are each greater in value than either C or D.

(A) Statement (1) ALONE is sufficient to answer the question, but statement (2) alone is NOT sufficient.
(B) Statement (2) ALONE is sufficient to answer the question, but statement (1) alone is NOT sufficient.
(C) BOTH statements (1) and (2) TOGETHER are sufficient to answer the question, but NEITHER statement ALONE is sufficient.
(D) Each statement ALONE is sufficient to answer the question.
(E) Statements (1) and (2) TOGETHER are NOT sufficient to answer the question.
329132人目の素数さん:2010/11/13(土) 08:58:25
Question 5

If x^2 + 6x = –9, how many values of x are possible?

(A) none
(B) one
(C) two
(D) three
(E) infinitely many
330132人目の素数さん:2010/11/13(土) 16:41:16
ここは算数の問題を英語で書くスレじゃない。
331132人目の素数さん:2010/11/13(土) 20:05:17
問五

若 x^2+6x=-9 、 (二)幾 何 (一レ)値 (レ)x (レ)満 是 。

(甲) 無
(乙) 一 耳
(丙) 二
(丁) 三
(戊) (レ)無 限
332132人目の素数さん:2010/11/13(土) 22:18:43
>>318
2x >= y^2 ∧ x^2 + y^2 >= 1
333132人目の素数さん:2010/11/24(水) 12:12:08
f(n, m) = Σ[k=0, n]k^mを求めよ。
334132人目の素数さん:2010/11/24(水) 17:24:42
>>333

nのm+1次式であり
 f(n, 0) = n+1,
 f(n, m) = Σ[i=1,m+1] A(m+1,i) n^i, (m>0)
ここに
 A(m+1,i) = (-1)^δ(i,m) {1/(m+1)} C(m+1,i) B(m+1-i),
 δ(i,m) はクロネッカーのデルタ,
 C(m+1,i) は2項係数,
 B(j) はベルヌーイ数.

http://mathworld.wolfram.com/PowerSum.html
http://mathworld.wolfram.com/FaulhabersFormula.html
335132人目の素数さん:2010/11/25(木) 00:12:54
g(n, m) = Σ[k=1, n] k(k+1)…(k+m-1) を求めよ。

336132人目の素数さん:2010/11/25(木) 00:17:46
>>335

 k(k+1)…(k+m-1) = {k(k+1)…(k+m) - (k-1)k(k+1)…(k+m-1)}/(m+1),
より
 g(n, m) = n(n+1)…(n+m)/(m+1),

注) Pochhammer の記号
337132人目の素数さん:2010/11/25(木) 17:35:24
フィボナッチ数列の第n項をa[n]とおく
lim[n→∞](a[n]/a[n+1] + a[n+1]/a[n])を求めよ
338132人目の素数さん:2010/11/25(木) 17:47:27
√5
339132人目の素数さん:2010/11/26(金) 21:15:27
a,bを正の整数とする
1*1の正方形のタイルを隙間なく並べてa*bの長方形ABCDを作る。
Aにあるタイルを最初に出発し、この長方形の全ての
タイルを一回ずつ通ってCにあるタイルへ最後に到達する。
この場合の数は何通りあるか求めよ。
タイルからは辺を共有するタイルへのみ移動
できるものとする。
340132人目の素数さん:2010/11/26(金) 21:26:08
>>339
0通りの場合もあるんで、まともにはできそうにないが。
341132人目の素数さん:2010/11/26(金) 21:48:26
まともでないとは、どのような意味だ?
342132人目の素数さん:2010/11/26(金) 22:21:52
0通りに関しては偶奇分け程度でできそうな気もする
できない気もする
343132人目の素数さん:2010/11/26(金) 22:31:01
a,bが両方偶数のときだけ0通りかな
344132人目の素数さん:2010/11/28(日) 22:22:20
既出だろうけど、問題。
cを正の実数、dを(0<d<c)を満たす実数とする。
x軸上のdと、y軸上の(c-d)を通過する直線をdの範囲内で連続的に動かしたとき、
直線が通過する領域と通過しない領域の境界となる曲線を方程式で表せ。
345132人目の素数さん:2010/11/29(月) 01:31:59
xy平面上にx=m,y=n(m=0,1,2,3,・・・,n=0,1,2,3,・・・)で表される網目状の道がある。
原点(0,0)を出発し、点(4,5)へ秒速1で遠回りせずに向かうことを考える。
ただし、途中サルがおり、これと遭遇してはならない。
サルは最初点(4,5)におり、格子点で移動の向きを変えながら秒速1で常に移動する。
サルと遭遇せずに無事点(4,5)にたどり着くことのできる確率を求めよ。
346132人目の素数さん:2010/11/29(月) 05:45:49
サルの移動は「格子点で移動の向きを変える」「常に移動」から直進・停止はありえないことは分かるが
あとはランダムなのか?
347132人目の素数さん:2010/11/29(月) 08:12:05
格子点上では出会わない。
出会うとしたら、格子点と格子点の間で、4.5後、6.5後、8.5後の何れか。
それぞれについて、確率を計算し、和を求める。(重複の考慮も必要)
と言う問題なのか?
348132人目の素数さん:2010/11/29(月) 11:04:45
>>344
2点を通る直線の方程式は、0 < d < cのとき
y = -d-c*x/d+x+c
xを固定してyをdの関数としてdで微分すると
y'(d) = -1+c*x/d^2
yの最大値は0 < d < cの範囲でd = √(c*x)のときで
y = -2*√(c*x)+x+c
349132人目の素数さん:2010/11/30(火) 00:19:39
√(2+4+6+8+…)−√(1+3+5+7+…)=?
350132人目の素数さん:2010/11/30(火) 01:10:05
>>349
2
351132人目の素数さん:2010/11/30(火) 10:12:15
>>349
不定
352132人目の素数さん:2010/11/30(火) 11:56:27
>>349
lim[n→∞](√(n^2+n)-n)
353132人目の素数さん:2010/11/30(火) 12:15:23
lim[n→∞]n*(1+1/(2*n)-1)
= 1/2
354132人目の素数さん:2010/11/30(火) 13:04:23
353は、√(1+x)=1+(1/2)x+...の展開を使ったようだが、分子の有利化の方が簡明だとおもわれる
(√(n^2+n)-n) =(n^2+n-n^2)/(√(n^2+n)+n)=n/(√(n^2+n)+n) → 1/2 (n→∞)
355132人目の素数さん:2010/11/30(火) 13:17:04
>>252
まず前提がおかしい
√(2+4+6+8+…)-√(1+3+5+7…)=lim[m→∞](√(m(m+1)))-lim[n→∞](n)≠lim[n→∞](√(n^2+n)-n)
356132人目の素数さん:2010/11/30(火) 14:16:50
>>355
349が丁寧に√の中に4項づつ整数を書いている事を
考えれば、m = nと推測される。
357132人目の素数さん:2010/11/30(火) 14:36:45
>>355>>352宛だった

>>356
> 349が丁寧に√の中に4項づつ整数を書いている事を
> 考えれば、m = nと推測される。
それは絶対にしてはいけない
√(2+4+6+8+…)はただ極限lim[n→∞](Σ[k=1,n](2n))を表しているだけであって、√(1+3+5+7+…)も同様で無関係

lim[n→∞](√(n^2+n)-n)を表したければ、
lim[n→∞](√(2+4+6+8+…+2n)-√(1+3+5+7+…+(2nー1))
とでもしなければいけない
358132人目の素数さん:2010/11/30(火) 20:54:30
>>345
0.676202425986312...
359132人目の素数さん:2010/11/30(火) 23:36:11
>>348
模範解答thx 正解です。
やっぱり簡単すぎたか・・・
個人的に、この曲線をはじめて定式化したときは感動した。
360132人目の素数さん:2010/12/01(水) 03:26:12
>>357
√(2+4+6+8+…)÷√(1+3+5+7+…) の場合も同じ?
361358:2010/12/01(水) 08:24:43
訂正
>>345
0.109068645363001...
362132人目の素数さん:2010/12/01(水) 11:36:47
>>360
同じ、減算と同様に不定
363132人目の素数さん:2010/12/01(水) 15:00:57
>>362

よく教科書なんかに

2*2*4*4*6*6*8*8*…
-------------------- = π/2
1*3*3*5*5*7*7*9*…

という公式が載っているが、不適切な記述?
364132人目の素数さん:2010/12/01(水) 15:26:34
>>363
どんな教科書やねんw ダメに決まってる。
((2/1)*(2/3))*((4/3)*(4/5))*((6/5)*(6/7))*((8/7)*(8/9))*…
って書けばいいだけのこと。
365358:2010/12/01(水) 15:43:24
再訂正
>>345
192421442772185427049260473 / 238490541610172532400324608
= 0.806830499327349...
366132人目の素数さん:2010/12/01(水) 16:00:00
岩波数学辞典第3版28円周率
367132人目の素数さん:2010/12/01(水) 17:22:17
17世紀にはそれでよかったんだろう >ウォリスの公式
368358:2010/12/02(木) 14:52:20
>>345
再々訂正
113320301 / 143327232 = 0.790640406702335
369132人目の素数さん:2010/12/02(木) 22:28:33
>>368
見苦しいから、もう止めなよ。
370132人目の素数さん:2010/12/03(金) 00:06:49
a[1]=2
a[2]=4
a[n+2]=a[n+1]^a[n]
のとき
a[n]を求めよ
371132人目の素数さん:2010/12/03(金) 01:47:42
a[n]=2^b[n]と表せる
以下略
372358:2010/12/03(金) 03:31:40
>>369
それが正解だからもう書かないけどな
プログラムでしか解決できない問題
373358:2010/12/03(金) 03:34:18
>>369
それが正解だからもう書かないけどな
プログラムでしか解決できない問題
374132人目の素数さん:2010/12/03(金) 05:47:55
(0,0)から(4,5)へ行く道順はC(9,5)通りあるが、それぞれ同じ確率なのか、それとも、
分岐に来たときに等確率で、いずれかの道が選ばれるか、それが問題では指定されていない。

また、もしかすると、「遭遇せず無事に」等という記述から、遠方に猿が見えるとき、
(可能ならば)猿と出会わないようなルートへ切り替える事が可能としているかも知れない。

いずれにせよ、問題を解くには、その問題がきちんと確定していなければならない。それが
なされないまま、自分の勝手な解釈で問題を解き、答えを何度も訂正したあげく、最後に書
いたのが正解だ等と言い放ち、さらに、プログラムでしか解けないなどと断言するような輩
の言葉が信用されると思われるか?
375358:2010/12/03(金) 06:50:29
>>374
「点(4,5)へ秒速1で遠回りせずに向かうことを考える」ということから、
(0, 1), (1, 0)方向にしか移動しない。両方とも確率1/2で移動すると仮定した。
また、サルは
0 <= x <= 4, 0 <= y <= 5
の領域内を移動すると仮定した。

「サルは最初点(4,5)におり、格子点で移動の向きを変えながら秒速1で常に移動する。」
これは
1. サルが格子点で移動可能な方向全てに等確率で移動する
2. サルが格子点で必ず方向転換する
の2つが考えられると思われるが、1を採用した。

>遠方に猿が見えるとき、(可能ならば)猿と出会わないようなルートへ切り替える事
それが可能であれば、問題の意味がない。
376358:2010/12/03(金) 06:51:14
>>374
「(0,0)から(4,5)へ行く道順はC(9,5)通りあるが、それぞれ同じ確率」
「分岐に来たときに等確率」
は同値だな。
言葉が信用されようが、信用されまいがそんな事はどうでもいい。
377132人目の素数さん:2010/12/03(金) 08:41:59
C(9,5)通りの道全てが同確率の場合、それぞれの道順は1/126で選択される
一方、分岐路毎に等確率で移動する場合、例えば、→→→→↑↑↑↑↑という
道順は、最初の4通りのみ選択肢があるので、1/16で選択される。
明らかに、異なり、「同値」ではない。

> また、サルは
> 0 <= x <= 4, 0 <= y <= 5
> の領域内を移動すると仮定した。

問題を読む限り、サルは(5,6)へ移動してから、接触可能ゾーンに復帰する事も可能
問題を正しく読み取れていない。

>それが可能であれば、問題の意味がない。
意味がないわけなはない。別のより複雑な問題になるだけ。
378132人目の素数さん:2010/12/03(金) 09:03:03
別にどのように設定してといたっていいじゃん
試験じゃないんだし
379358:2010/12/03(金) 09:22:57
>>377
「分岐に来たときに等確率」
は、サルでない方は行き方を制限されて、結果的にC(9, 5)と等しくなる
と言いたかった。題意から、そのように制限されるからね。

問題を正しく読み取るって何?。上記の仮定をした場合についての
計算結果を示しているだけ。
380132人目の素数さん:2010/12/03(金) 09:58:05
> と言いたかった。題意から、そのように制限されるからね。

いずれの解釈であろうと、ルートの数がC(9,5)通りなのは、当たり前。
違うのは、それぞれのルートがもつ確率。

> 問題を正しく読み取るって何?。上記の仮定をした場合についての
> 計算結果を示しているだけ。

勝手に仮定を設けて答えを出したのなら、答えに、「この様な仮定を設けた」と
一言触れておかないと、全く評価されない。
381132人目の素数さん:2010/12/03(金) 14:42:38
面白い問題よりも他人の問題点のほうが好きなやつが混じっているようだな。
382132人目の素数さん:2010/12/03(金) 23:33:40
そうでもないだろ
そこそこ面白いかもしれないが粗がある問題と
それへの指摘が存在してるだけだ
383132人目の素数さん:2010/12/03(金) 23:59:06
382 132人目の素数さん[sage]:2010/12/03(金) 23:33:40
そうでもないだろ
そこそこ面白いかもしれないが粗がある問題と
それへの指摘が存在してるだけだ
384132人目の素数さん:2010/12/04(土) 02:59:11
>>381
条件が違えば解答が違うのに指摘しないほうが問題だろ
385132人目の素数さん:2010/12/04(土) 06:27:29
384 132人目の素数さん[sage]:2010/12/04(土) 02:59:11
>>381
条件が違えば解答が違うのに指摘しないほうが問題だろ
386132人目の素数さん:2010/12/04(土) 06:28:19
374 132人目の素数さん[sage]:2010/12/03(金) 05:47:55
(0,0)から(4,5)へ行く道順はC(9,5)通りあるが、それぞれ同じ確率なのか、それとも、
分岐に来たときに等確率で、いずれかの道が選ばれるか、それが問題では指定されていない。

また、もしかすると、「遭遇せず無事に」等という記述から、遠方に猿が見えるとき、
(可能ならば)猿と出会わないようなルートへ切り替える事が可能としているかも知れない。

いずれにせよ、問題を解くには、その問題がきちんと確定していなければならない。それが
なされないまま、自分の勝手な解釈で問題を解き、答えを何度も訂正したあげく、最後に書
いたのが正解だ等と言い放ち、さらに、プログラムでしか解けないなどと断言するような輩
の言葉が信用されると思われるか?
387358:2010/12/04(土) 07:39:11
>>374
>さらに、プログラムでしか解けないなどと断言するような輩
>の言葉が信用されると思われるか?

そうであれば、プログラム以外の方法でこの問題が解決できるという事を
示さなければ、そのような輩の言葉も信用されない。
388132人目の素数さん:2010/12/04(土) 08:49:00
>>359
候補の曲線を与える方程式として見当つけるだけなら
判別式からすぐ出ると思うんだが、
どこに感動したん?
389132人目の素数さん:2010/12/04(土) 13:04:35
原点からスタートした物は時刻4に(4,0),(3,1),(2,2),(1,3),(0,4)の何れかにいる。
いずれの場所にいるかは、問題の設定で異なるが、分岐では1/2の確率で何れかが選ばれるとして
考える事にすると、それぞれ、1/16,4/16,6/16,4/16,1/16の確率で、それぞれの位置にいる。
(4,0)から残りの経路は1通り、(3,1)および(0,4)から残りの経路は5通り、(2,2)および(1,3)から
残りの経路は10通り、合計31通りの経路がある。
例えば、(3,1)から、(3,2)-(4,2)-(4,3)-(4,4)-(4,5)という経路を取る場合を考える。
時刻4にサルが(3,2)にいる確率は(4/4^4)
この経路を通り、時刻4.5に遭遇する確率は、(4/16)*(1/2)^2*(4/4^4)*(1/4)
時刻5にサルが(4,2)にいる確率は(10/4^5)
この経路を通り、時刻5.5に遭遇する確率は、(4/16)*(1/2)^2*(10/4^5)*(1/4)
時刻6にサルが(4,3)にいる確率は(225/4^6)
この経路を通り、時刻6.5に遭遇する確率は、(4/16)*(1/2)^2*(225/4^6)*(1/4)
時刻7にサルが(4,4)にいる確率は(1225/4^7)
この経路を通り、時刻7.5に遭遇する確率は、(4/16)*(1/2)^2*(1225/4^7)*(1/4)
時刻8にサルが(4,5)にいる確率は(4900/4^8)
この経路を通り、時刻8.5に遭遇する確率は、(4/16)*(1/2)^2*(4900/4^8)*(1/4)
この和がこの経路を取ったときに遭遇する確率。高々31通り(対称性を利用すれば、実質はもっと少ない)について、
同様の計算を行えば、プログラムでなくても計算可能。ただ、面倒くさいだけ。もしかすると、どこかに見落としが
あるかも知れないが、「プログラムでしか解決できない問題」に変質するとはとうてい考えられない。
なお、サルが、時刻tに(4-x,5-y)にいる確率はΣ[a+b+c+d=t,a-b=x,c-d=y] t!/(a!*b!*c!*d!*4^t)で与えられる。
390132人目の素数さん:2010/12/04(土) 14:32:32
>>389
サルが人間と同じ時刻で遭遇し、その後同じ経路をだどる場合も重複して
確率を足してしまうから、それでは合っていない。
391132人目の素数さん:2010/12/04(土) 15:06:17
サルおよび、人間が格子点上にいるとき、
サルの位置のX座標+サルの位置のY座標+時刻 は常に奇数
人間の位置のX座標+人間の位置のY座標+時刻 は常に偶数
だから、格子点上でサルと人間が遭遇する事はない。
遭遇するのは常に、道路の中間で、一度遭遇すると、時間0.5前の時と位置が入れ替わる状態になり、距離は1となる。
その後、サルがどのような歩みを行おうとも、人間を追い抜く事はもちろん、出会う事もない。
従って遭遇は一度きり。
392358:2010/12/04(土) 15:32:10
>>390は間違えた。

>>389
>>375で設定した問題で、プログラムでしか解決できないと言っているので
話のすり替えだと思う。
393132人目の素数さん:2010/12/04(土) 15:52:46
プログラムでしか解決できないって言ったのはお前だぞ。

>> 373 :358:2010/12/03(金) 03:34:18
>> >>369
>> それが正解だからもう書かないけどな
>> プログラムでしか解決できない問題


俺が見る限り、そこまで膨大な内容ではない問題なので、374では、そんなことを断言
するのはおかしいと指摘したところ、387で、だったら非プログラム的にやってみろと
言われたので、389で手計算での手法の方針を示した。

392の「言っているので」 とか、「話のすり替えだと思う。」とか、意味不明。

それから、ちょっと確認するが、390の書き込みもお前(=358)でいいんんだよな
394132人目の素数さん:2010/12/04(土) 15:54:57
また低能者のグダグダ合戦か
395132人目の素数さん:2010/12/04(土) 15:59:51
どっちも消えろようぜぇ
396132人目の素数さん:2010/12/04(土) 16:19:54
>>393
>>372では、>>375で示した条件を加えた問題において、プログラムでしか解決できないと
言っている。

そちらは勝手に、自分でサルは拘束条件がないという問題を仮定してそれを手計算で
しているだけ。分からないのこの違いが。
397132人目の素数さん:2010/12/04(土) 16:44:11
お前は未来に書かれる内容についてコメントしたと言うのか。
傑作だな。ここまでトンチンカンないい訳を恥知らずにも堂々と書いてくるとは。

>>345のオリジナル文面では、サルが非衝突領域への移動を制限する拘束条件など最初からない
非制限が問題の設定であり、お前が勝手に制限を設けた。
398132人目の素数さん:2010/12/04(土) 16:54:17
>>397
未来がどうたら、頭大丈夫?
勝手に設けた制限のもとではプログラムでしか解決できないと言っている。
私が設けた制限の下で計算できるならその方法を示していただきたい。
399132人目の素数さん:2010/12/04(土) 18:52:31
>>388
問題の難易度の話じゃなく。
原型は、「方眼紙でx軸とy軸の目盛りを足してNとなる直線を引くと、
綺麗な曲線が浮かび上がる。」という話から。
中学生レベルで簡単に書ける身近な曲線だが、
定式化には高校数学が分かる程度の数学の慣れが必要って意味で、
思い入れが深いって意味。

どっかの試験問題にならんかなー。
400132人目の素数さん:2010/12/04(土) 18:57:10
>>398 ほれ。
4×5という制限が設けられた領域でのランダムウォーク。本質的な差は何らない。
ただし、壁があるため、1/2、1/3、1/4の何れかで移動する。
角からは6、辺からは4、内部からは3の重みをかけて変移する分布表を作ればよい。

左:時刻1(分母2)     中央:時刻2(分母6)     右:時刻3(分母72)
0000---0000---0000---0001---0000    0000---0000---0001---0000---0002    0000---0004---0000---0022---0000
0000---0000---0000---0000---0001    0000---0000---0000---0002---0000    0000---0000---0010---0000---0022
0000---0000---0000---0000---0000    0000---0000---0000---0000---0001    0000---0000---0000---0010---0000
0000---0000---0000---0000---0000    0000---0000---0000---0000---0000    0000---0000---0000---0000---0004
0000---0000---0000---0000---0000    0000---0000---0000---0000---0000    0000---0000---0000---0000---0000
0000---0000---0000---0000---0000    0000---0000---0000---0000---0000    0000---0000---0000---0000---0000

左:時刻4(分母864)      中央:時刻5(分母10368)      右:時刻6(分母124416)
0016---0000---0134---0000---0176    0000---0770---0000---2300---0000    4016---0000--16966---0000--18400
0000---0046---0000---0236---0000    0234---0000---1562---0000---2300    0000---9656---0000--27772---0000
0000---0000---0060---0000---0134    0000---0318---0000---1562---0000    1890---0000--11280---0000--16838
0000---0000---0000---0046---0000    0000---0000---0318---0000---0738    0000---1908---0000---9198---0000
0000---0000---0000---0000---0016    0000---0000---0000---0202---0000    0000---0000---1560---0000---3942
0000---0000---0000---0000---0000    0000---0000---0000---0000---0064    0000---0000---0000---0990---0000
401132人目の素数さん:2010/12/04(土) 21:50:40
俺は財布を持つようになってこのかた、
つまり20年くらいか、財布を失ったことがない。
これからもずっと失わずにいるかもしれない。
これはつまり、自分の一年あたりの財布を失う確率が低いことを
毎年証明して行ってるようなものだ。

[質問]
20年間財布を失ったことがない俺の
一年あたりに財布を失う確率はいくらなのか?
また、ちょうど20年目に初めて財布を失ったとした場合、
それはいくらになるのだろうか。

[雑談]
あなたは今まで、何年間の間に何回くらい財布を失いましたか?
402132人目の素数さん:2010/12/04(土) 23:23:51
>>400
続きは?
403132人目の素数さん:2010/12/04(土) 23:52:34
>>401
この手の核心を逸れた例示は見覚えがあるな
404132人目の素数さん:2010/12/04(土) 23:54:44
>>399
>定式化には高校数学が分かる程度の数学の慣れが必要って意味で、
>思い入れが深いって意味。

個人的な問題というわけだな。
このように一般論と個人的な問題を区別して論じる態度は大切だ
405132人目の素数さん:2010/12/05(日) 02:47:22
俺は週に5日間、毎週
午前6時から午前8時の間と、午後6時から午後8時の間電車に乗る。
一生に一度だけ大地震にあうとしたら、
電車の中でその大地震にあう確率は何パーセント?
406132人目の素数さん:2010/12/05(日) 02:58:05
>>405
0
407132人目の素数さん:2010/12/05(日) 03:17:34
>>405
規則正しい生活でいいね。
4/24=1/6 だね。サイコロ並のクソな確率でした
408401:2010/12/05(日) 03:47:20
自分で解決してみた。
2つ目だけ答えが出た。
1年間に財布を失う確率を p、「ちょうどX年目に初めて財布を失った」の事象がおこる確率を q とすると
q = (1-p)^(X-1)×p^1
r = logq = (X-1)ln(1-p)+ln(p)
dr/dp = 0 とすると -(X-1)/(1-p)+1/p=0, (X-1)p=1-p, Xp=1, p=1/X
よって、p=1/X、つまり 1/20 である確率(尤度?)が一番高く、
「20年に1回起こったんだから 1/20」というのは数学的にも正しかったことが分かった。
ちなみにその場合、その確率は 1.886 %。 かなり低いがこれが最高の確率でしかない。不思議。

また、1つめの問題は、同じように考えると
「俺は財布を失うことが無い」が一番確からしいことになってしまった。
不思議。
409132人目の素数さん:2010/12/05(日) 07:11:39
>>402 本質的な差はないと書いただろう。

389で
> なお、サルが、時刻tに(4-x,5-y)にいる確率はΣ[a+b+c+d=t,a-b=x,c-d=y] t!/(a!*b!*c!*d!*4^t)で与えられる。
と書いた物が、400の内容に変更されるだけ

もし、時刻7と時刻8の数値が必要というのなら、
時刻7 (4,4)および、(5,3)に 16966*4+27772*3+18400*6=261580 
時刻8 (5,4)に 261580*4=1046320 
をあげておく。元々表全体は必要ない。必要な数字はこれだけだから。

> 勝手に設けた制限のもとではプログラムでしか解決できないと言っている。
> 私が設けた制限の下で計算できるならその方法を示していただきたい。
と358がコメントしたことから、358はオリジナル版での手計算の可能性を認めるものの、
自分で課した矩形制限内での、サルの動きとなると手計算では追えないと思っていた
のだろう。だから、その部分のみ抽出して示したまで。

もともと、「プログラムでしか解決できない」を否定したいだけなので、既に十分な材料は示した。
410132人目の素数さん:2010/12/05(日) 07:27:11
>>409
それで?
411132人目の素数さん:2010/12/05(日) 07:46:29
> 勝手に設けた制限のもとではプログラムでしか解決できないと言っている。
> 私が設けた制限の下で計算できるならその方法を示していただきたい。

に応え、方法を示しただけ。もともとは、

>> 387 :358:2010/12/04(土) 07:39:11
>> >>374
>> >さらに、プログラムでしか解けないなどと断言するような輩
>> >の言葉が信用されると思われるか?
>>
>> そうであれば、プログラム以外の方法でこの問題が解決できるという事を
>> 示さなければ、そのような輩の言葉も信用されない。

から始まっている。

>>372 >>373の「プログラムでしか解決できない問題」および、「それが正解」(←自ら、余計な
条件を課した元での計算である事を認めた為、オリジナル版の正解でない事は明白)を撤回するのが、
いいのではないでしょうかね。
412132人目の素数さん:2010/12/05(日) 07:47:04
早く確率求めろや池沼
413132人目の素数さん:2010/12/05(日) 07:47:49
>>411
死ねゴミ
414358:2010/12/05(日) 09:02:33
>>411
何でもいいが、プログラムでしか計算できないと言ったのを
正確に言い直させていただくならば、手計算では計算量が膨大で
計算するのが面倒であるという事。
数学的に解答があるとするならば、ゴールを(m, n)として
ゴールに到達する確率P(m, n)を求める事ではないだろうか?
また、より一般的な問題としてサルの初期位置を(p, q)としてP'(m, n, p, q)を
求める問題もあると考えられる。
これらが計算できないで、手計算で問題が解けると強弁されてもね。
415132人目の素数さん:2010/12/05(日) 09:04:05
やはり面白い問題よりも他人の問題点のほうが好きなやつが混じっているようだな。 
416132人目の素数さん:2010/12/05(日) 09:19:32
>>407
>週に5日間
この部分を見逃してますよ
417132人目の素数さん:2010/12/05(日) 09:32:00
>>414
> より一般的な問題としてサルの初期位置を(p, q)としてP'(m, n, p, q)を
> 求める問題もあると考えられる。
一般式を求めてください。プログラムで。
418132人目の素数さん:2010/12/05(日) 09:41:55
417 132人目の素数さん[sage]:2010/12/05(日) 09:32:00
>>414
> より一般的な問題としてサルの初期位置を(p, q)としてP'(m, n, p, q)を
> 求める問題もあると考えられる。
一般式を求めてください。プログラムで。
419132人目の素数さん:2010/12/05(日) 12:48:17
ようやく

>>373
>> プログラムでしか解決できない問題

と書いていたのを、

>>414
>> 何でもいいが、プログラムでしか計算できないと言ったのを
>> 正確に言い直させていただくならば、手計算では計算量が膨大で
>> 計算するのが面倒であるという事。

と訂正させるに至った。
顔の見える世界では別だが、この様な場でこれを為すのは困難。相手が逃げてしまうからだ。
きちんと自分の言に責任を持った事は賞賛してあげよう。
420358:2010/12/05(日) 12:56:08
>>417
m + nが奇数の場合
時刻tに(x, y)に存在する確率をp(x, y, t)
時刻tにサルが(x, y)に存在する確率をq(x, y, t)
(x, y)での進行方向の数をr(x, y)
(x, y)にいて一秒後に(X, Y)に移動するとすると
p(0, 0, 0) = 1
q(m, n, 0) = 1
P(X, Y, t + 1) = p(x, y, t) * (1 - q(X, Y, t)) / (4 * r(x, y))
P(m, n) = Σ[全てのルート]P(m, n, m + n)
この一般解は分からない。
421132人目の素数さん:2010/12/05(日) 12:58:18
>>419
死ねゴミ
422358:2010/12/05(日) 12:59:31
>>419
結局解き方を示しただけで、結局結果が得られていないが。
423132人目の素数さん:2010/12/05(日) 12:59:37
419 132人目の素数さん[sage]:2010/12/05(日) 12:48:17
ようやく

>>373
>> プログラムでしか解決できない問題

と書いていたのを、

>>414
>> 何でもいいが、プログラムでしか計算できないと言ったのを
>> 正確に言い直させていただくならば、手計算では計算量が膨大で
>> 計算するのが面倒であるという事。

と訂正させるに至った。
顔の見える世界では別だが、この様な場でこれを為すのは困難。相手が逃げてしまうからだ。
きちんと自分の言に責任を持った事は賞賛してあげよう。
424132人目の素数さん:2010/12/05(日) 13:24:09
>>422
「プログラムでしか解決できない問題」という認識を
「手計算では計算量が膨大で計算するのが面倒」
という、遠回しではあるが可能であるという認識に変更させる事ができただろ。
これが結果だ。

もし、確率を計算しろと言うのなら、お前が言うように「面倒」だから計算するつもりはない。
実際、この問題の確率を計算した形跡があるのは、お前だけのようであり、俺も他の
大部分と同じで、この様な問題で腕力を振るうつもりはない。
ただし、興味があるとすれば、それは答えを導くための方針捜索であり、計算の結果でるで
あろう確率自体には全く興味はない。そして、なにより問題投稿者へ、問題の設定説明を求
めても、なんの回答も無かったため、最初少しはあったトライしてみようかという気が一気
に失せた。
425132人目の素数さん:2010/12/05(日) 13:27:45
>>424
死ねゴミ
426132人目の素数さん:2010/12/05(日) 13:30:25
>>424
プログラミングがどういうものが知らないの?
427358:2010/12/05(日) 13:31:04
>>424
計算するのが面倒な問題なので、そういう風に言えば
お前のように、その問題解決の方針を示す奇特な人間が出てくるかと
考えてそういう風に言ったまで。
428132人目の素数さん:2010/12/05(日) 13:32:12
>>424
早く求めろド低脳
429132人目の素数さん:2010/12/05(日) 17:17:11
面白い問題の話をするスレであって
心の問題をさらけ出すスレではありませんよ
430132人目の素数さん:2010/12/05(日) 17:41:05
まったくだ
431132人目の素数さん:2010/12/05(日) 17:43:28
>>400
>>409
それで何を求めるつもりなの?
432132人目の素数さん:2010/12/05(日) 19:41:40
>>429
どこに心の問題があるのか指摘してくれ。
433132人目の素数さん:2010/12/05(日) 22:42:58
>>345
この問題は面白いだろうか?
だけど、「遭遇してはならない」が「格子点で遭遇してはならない」だったら、面白い問題だと思う。
434358:2010/12/05(日) 23:35:53
問題を計算し易い様に、点(1, 2)に向かう場合を考える。
(1, 0)→(1, 1)→(1, 2)と移動する場合の確率は、7/16となるが、
>>389の方法では計算できない。
435132人目の素数さん:2010/12/06(月) 01:09:54
>>416
じゃあ適当に 5/7 かけといてくれ
436132人目の素数さん:2010/12/06(月) 07:19:09
>>432
心の問題は一般的には心にあるよ。
脳にある。という意見もある。
437358:2010/12/06(月) 09:10:23
>>409
手計算でその方法では>>434のように計算できないと思われる。
そのため>>420は正しい漸化式となっていない。

私が勝手に設定したサルの移動制限を外し、サルは無制限に
等確率で格子点において4方向に移動するとした場合の>>345の解は
プログラムで計算すると以下の通り。
1959399/2097152 = 0.934314250946044921875...

そちらが主張するプログラム以外での計算方法を示してくれ。
438132人目の素数さん:2010/12/06(月) 11:43:42
脳は使うためにあるよ。
439132人目の素数さん:2010/12/06(月) 11:49:05
>>438
死ねゴミくす
440132人目の素数さん:2010/12/06(月) 12:39:23
>>438
脳を使って、その結果を披露していただきたい
441132人目の素数さん:2010/12/06(月) 13:05:08
心当たりがあると
ちょっとした言葉でも皮肉として受け取ることが出来るといういい見本
442132人目の素数さん:2010/12/06(月) 13:43:40
>>441
心当たりがないがそう書かれたから、問題の解決方法があると思って
聞いてみた

そう言い返す事しかできないようで、お気の毒
443132人目の素数さん:2010/12/06(月) 14:00:30
またあさはかな口喧嘩
いつものことだなw
444132人目の素数さん:2010/12/06(月) 15:07:32
そういう問題のほうが好きなんだから
しかたがない
445132人目の素数さん:2010/12/06(月) 15:35:25
>>437 389の方法で、ルートについての和を考えると、結局は以下の方法に帰着できるようである。
つまり、遭遇する時刻での、場所について、存在確率を求めればよい

時刻t(t=4.5,5.5,6.5,7.5,8.5)に、道路の中間 (0,t),(1/2,t-1/2),(1,t-1),...,(4,t-4)
に人間がいる確率を順に、列挙したベクトルH(t)を考えると
H(4.5)=(1,1,4,4,6,6,4,4,2)/2^5≡H4
H(5.5)=(0,2,5,5,10,10,10,10,12)/2^6≡H5
H(6.5)=(0,0,0,14,15,15,20,20,44)/2^7≡H6
H(7.5)=(0,0,0,0,0,58,35,35,128)/2^8≡H7
H(8.5)=(0,0,0,0,0,0,0,186,326)/2^9≡H8
同様にサルの存在確率をベクトルにすると(矩形外は考えなくて良いので、0とした)
S(4.5)=(1,4,4,6,6,4,4,1,1)/4^5≡S4
S(5.5)=(0,25,25,50,50,50,50,25,25)/4^6≡S5
S(6.5)=(0,0,0,225,225,300,300,225,225)/4^7≡S6
S(7.5)=(0,0,0,0,0,1225,1225,1225,1225)/4^8≡S7
S(8.5)=(0,0,0,0,0,0,0,4900,4900)/4^9≡S8

内積を取り、和を取って1から引くと
1-{(H4,S4)+(H5,S5)+(H6,S6)+(H7,S7)+(H8,S8)}=1959399/2097152 を得る
電卓類は使用したが、手計算で行った。
446132人目の素数さん:2010/12/06(月) 17:31:25
> 電卓類は使用したが、手計算で行った
447132人目の素数さん:2010/12/06(月) 19:31:19

人間の意識モデル、重力や環境により収束エリアに関数結果を
おさめる関数を探せます?

たとえば

●行列データ(外部刺激と、記憶)
A系 : A1,A2・・・・・・An
  ・
Z系 : Z1,Z2・・・・・・Zn

●さらに、各データは時間tと互いの関数(学習と、ニューロン結合)
New A1=a1(t、A1,A2・・・・・・An、・・・、Z1,Z2・・・・・・Zn)
New A2=a2(t、A1、・・・・・
   ・・・
● 時間凾狽ェ進む事で上記は永遠に書き換えられる

● 意識フィールド円エリア半径R (Rは意識のスポットエリア) 
  R= w (t、X,、Y)
  X= x(t、A1,A2・・・・・・An、・・・、Z1,Z2・・・・・・Zn)
  Y= y(t、A1,A2・・・・・・An、・・・、Z1,Z2・・・・・・Zn)
  ※ 0<R<K

− A1〜Zn は環境により変化する変数
− a1〜an 関数は意識関数Wにより変化

これを満たす、関数 an、w、x、y はどのようなモノになるか?

448132人目の素数さん:2010/12/07(火) 01:54:24
>>447
もっと頭の中を整理したあとで
書き込む必要があるかどうか判断してから書き込もうね。
449132人目の素数さん:2010/12/07(火) 02:32:09
448 132人目の素数さん[sage]:2010/12/07(火) 01:54:24
>>447
もっと頭の中を整理したあとで
書き込む必要があるかどうか判断してから書き込もうね。
450132人目の素数さん:2010/12/07(火) 09:41:54
x, y, X, YをX>=0, Y>=0, 0<=x<=X, 0<=y<=Yを満たす整数とし
関数α, βを以下のように定義する。
α(x, X) = 1/2 (x < X)
α(x, X) = 1 (x = X)
β(y, Y) = 1/2 (y < Y)
β(y, Y) = 1 (y = Y)
以下の条件が成立するとき、関数P(y, x, Y, X)を求めよ。
P(0, x) = 1/2^x
P(y, 0) = 1/2^y
P(y, x, Y, X) = α(x, X)*P(y - 1, x) + β(y, Y)*P(y, x - 1)
451132人目の素数さん:2010/12/07(火) 09:54:44
>>450
以下の条件を追加
1<=x<=X, 1<=y<=Yのとき
P(y, x, Y, X) = α(x, X)*P(y - 1, x) + β(y, Y)*P(y, x - 1)
452132人目の素数さん:2010/12/07(火) 10:55:37
>>445に追加
サルの移動を[0,0]×[4,5]に限った場合についても、非プログラム的方法で>>368と同じ結果を得た。

K4=(16*6,46*3,46*3,60*3,60*3,46*3,46*3,16*4,16*4)/(6*12^3)
K5=(0,770*4,770*4,1562*3,1562*3,1562*3,1562*3,738*4,738*4)/(6*12^4)
K6=(0,0,0,16966*4,16966*4,27772*3,27772*3,16838*4,16838*4)/(6*12^5)
K7=(0,0,0,0,0,261580*4,261580*4,261068*4,261068*4)/(6*12^6)
K8=(0,0,0,0,0,0,0,2090592*6,2090592*6)/(6*12^7)
ここで使われている数字は下を除いて>>400に載せた物
(>>409に書いた数字は、時刻6の16966と16838が同じと勘違いして書いた物で間違いです)
K7の261580は16966*4+18400*6+27772*3
K7の261068は16838*4+18400*6+27772*3
K8の2090592は261580*4+261068*4

あとは、>>445で記したHを再利用し、
1-{(H4,K4)+(H5,K5)+(H6,K6)+(H7,K7)+(H8,K8)}=113320301/143327232 を得る
453132人目の素数さん:2010/12/07(火) 11:02:09
そろそろプログラムでは無理な解法が出てもいいはず
454132人目の素数さん:2010/12/07(火) 16:08:00
>>450
途中まで自己レス
0<x<X, 0<y<Yの範囲で
P(y, x) = x+yCx/2^(x+y)
455132人目の素数さん:2010/12/07(火) 17:22:07
P(X,y)=Σ[i=0,y]C(X+y,y-i)/2^(X+y)
P(x,Y)=Σ[i=0,x]C(x+Y,x-i)/2^(x+Y)
P(X,Y)=1
456132人目の素数さん:2010/12/08(水) 00:52:29
>>452
努力は買う
457132人目の素数さん:2010/12/08(水) 08:10:08
ageます、確率の問題で混乱してる人は下記の様な主張をしたりもします。

>交換すると期待値は下がる事は封筒の値を確認する前から分ってる
金額の組が{100,10000}という条件での、交換前後の期待値の大小関係
金額の組が{10000,1000000}という条件での、交換前後の期待値の大小関係
が、封筒の値を確認する前から分かる(決まっている・計算できる)のと同様に
交換前の金額が10000であるという条件の下での、交換前後の期待値の大小関係も分かる。

もっと言えば、確率分布が既知ならば
いかなる時点(例えばホストが封筒を準備する前(封筒の金額組を決める前)の時点)で、
交換前の金額が10000であるという条件の下での交換前後の期待値や
金額の組が{100,10000}という条件での交換前後の期待値等は分かる。

どれかの条件の下で計算したものが真に正しい期待値になる
というわけではなく、それぞれが各々の条件の下での(別の)期待値になるだけである。
別の条件の下での期待値は、別の期待値である為
(条件Aの下での交換前の期待値)≠(条件Bの下での交換前の期待値)となったり
(条件Aの下での交換前の期待値)>(条件Aの下での交換後の期待値)かつ
(条件Bの下での交換前の期待値)<(条件Bの下での交換後の期待値)となることもあるが
これは矛盾でもなんでもない。

どれかが真に正しい期待値というわけではないが、
「交換前の金額が10000であるということを知る人にとっての期待値」等は
「交換前の金額が10000であるという条件の下での期待値」
と解釈するのが自然で普通。それ以外の期待値について言いたい場合は、
「〜の条件の下での期待値」などと断るか、E[X|{X,Y}={100,10000}]等の
記号を用いて、区別できるようにする。

通常、封筒組を固定した期待値(金額の組が〜という条件の下での期待値)
のみを特別扱いすることはない。
(数学で用いられる論理は時制を扱わない。"時系列"は重要でない)
458132人目の素数さん:2010/12/08(水) 08:31:52
馬鹿な人の規制が解除でもされたのだろうか
459132人目の素数さん:2010/12/08(水) 17:47:52
>>457
おまえはまだ隔離スレから出てこれるレベルじゃない。帰れ。
460132人目の素数さん:2010/12/08(水) 18:22:17
P(Y, x) = P(Y, x-1) + P(Y-1, x)/2
= Σ[k=0, x]C(Y+k-1, k)/2^(Y+k)
= 1/(2^(Y+x))*Σ[k=0, x]2^k*C(Y+x-k-1, x-k)
Σ[k=0, x]C(Y+x, k) = C(Y+x-1, x) + 2*Σ[k=0, x-1]C(Y+x-1, k)
= Σ[k=0, x]2^k*C(Y+x-k-1, x-k)
∴P(Y, x) = Σ[k=0, x]C(Y+x, k)/2^(Y+x)
461132人目の素数さん:2010/12/09(木) 07:09:07
Aが向かうゴールを(X, Y)、ただしX+Yは奇数
Aが(y, x)に存在する確率をq(y, x)、q(y, x) = 0(y<0またはx<0)、>>454-455参照
時刻tのとき(y, x)にサルが存在する確率をr(t, y, x)
時刻tのとき(y, x)に向かおうとしていたAが(y, x-0.5)か
(y-0.5, x)でサルと遭遇して(y, x)に到達できなかった確率をp(t, x, y)とすると
p(t, y, x) = r(t-1, y, x)/4*[α(y)*q(t-1, y, x-1)+β(x)*q(t-1, y-1, x)]
α(y) = 1/2 y<=Y
α(y) = 1 y=Y
β(x) = 1/2 x<=X
β(x) = 1 x=X
∴P = 1-Σ[i=0, Y]Σ[j=0, X]p(i+j, i, j)
462>>461 訂正:2010/12/09(木) 17:29:37
α(y) = 1/2 y<Y
α(y) = 1 y=Y
β(x) = 1/2 x<X
β(x) = 1 x=X
463132人目の素数さん:2011/01/02(日) 04:26:01
f(x) = sinx + cosx , g(x) =1+x-x^2
の大小関係を調べよ
464132人目の素数さん:2011/01/02(日) 04:27:38
f(x)≧g(x)
等号はx=0のときのみ
465132人目の素数さん:2011/01/02(日) 06:13:28
>>463
  f "(x) = -sin(x) - cos(x) = -(√2)sin(x + π/4) ≧ -√2,
より
∴ f '(x) - f '(0) = f '(x) - 1 > -(√2)x, (x>0)
  f '(x) - f '(0) = f '(x) - 1 < -(√2)x, (x<0)
より
∴ f(x) - f(0) -x・f '(0) = f(x) -1 -x ≧ -(1/√2)x^2,   >>464
466132人目の素数さん:2011/01/03(月) 00:08:05
>>463
 f '(x) = cos(x) -sin(x) = (√2)sin(x +3π/4),
 x = 0, π/4, π/2 のとき f '(x) = 1 - (4/π)x が成り立つ。

 x < 0 では f '(x) < 1-(4/π)x,
 0 < x < π/4 では f '(x)は上に凸、f '(x) > 1-(4/π)x > 0,
 π/4 < x < π/2 では f '(x)は下に凸、f '(x) < 1-(4/π)x < 0,
 π/2 < x では f '(x) > 1-(4/π)x,
これらを積分して
 f(0) = f(π/2) = 1,
を用いれば
 f(x) ≧ 1 +x -(2/π)x^2,
等号成立は x=0, π/2 のとき。
467132人目の素数さん:2011/01/03(月) 00:13:01
> 等号成立は x=0, π/2 のとき。
468132人目の素数さん:2011/01/03(月) 00:22:22
>>467
日本語勉強しろ。
469猫は作業 ◆MuKUnGPXAY :2011/01/04(火) 15:58:24
もうそろそろエエか。

■■■■■■■■■■■■■■■■■■■■■■■■■■■■■■■■■■■■■■■
■■■■■■■■■■■■■■■■■■■■■■■■■■■■■■■■■■■■■■■
■■■■■■■ このスレは他板・他スレ運営妨害の非常に悪質糞スレの為に ■■■■■■
■■■■■■■反感を買って終了しました。 皆様のご愛顧有難う御座いました■■■■■■
■■■■■■■■■■■■■■■■■■■■■■■■■■■■■■■■■■■■■■■
■■■■■■■■■■■■■■■■■■■■■■■■■■■■■■■■■■■■■■■


470132人目の素数さん:2011/01/04(火) 18:34:15
>>468
数学勉強しろ。
471132人目の素数さん:2011/01/04(火) 21:13:25
>>467=>>470
>>466であってるが。
まさか1+x-(2/π)x^2が1+x-x^2と書いてあると思い込んでるんじゃあるまいな?
472猫は作業 ◆MuKUnGPXAY :2011/01/04(火) 21:23:15
■■■■■■■■■■■■■■■■■■■■■■■■■■■■■■■■■■■■■■■
■■■■■■■■■■■■■■■■■■■■■■■■■■■■■■■■■■■■■■■
■■■■■■■ このスレは他板・他スレ運営妨害の非常に悪質糞スレの為に ■■■■■■
■■■■■■■反感を買って終了しました。 皆様のご愛顧有難う御座いました■■■■■■
■■■■■■■■■■■■■■■■■■■■■■■■■■■■■■■■■■■■■■■
■■■■■■■■■■■■■■■■■■■■■■■■■■■■■■■■■■■■■■■


473132人目の素数さん:2011/01/05(水) 12:28:56
>>471
何故>>463にアンカーを付けて他の問題を解いているのか?
474132人目の素数さん:2011/01/05(水) 14:46:30
>>473
それは>>467となんか関係あるのか?
475470:2011/01/05(水) 16:08:13
>>471
>>467>>470

>>474
>>463>>467には何の関係もない
476132人目の素数さん:2011/01/05(水) 17:16:46
>>467=>>470=>>473=>>475
1+x-(2/π)x^2≧1+x-x^2(等号成立はx=0のとき)もわからんのか。
数学勉強しろ。
477470:2011/01/05(水) 18:42:45
>>476
だから>>467ではないと書いているが。
そんな式は誰でも分かる。主張しているのは>>473
478132人目の素数さん:2011/01/05(水) 18:53:56
>>477
何故>>466に聞かん?
f(x)≧1+x-(2/π)x^2≧1+x-x^2がわからん?
>>467>>468の流れに>>470を書いた意味は何?
479132人目の素数さん:2011/01/05(水) 19:06:22
>>468に数学勉強しろ。と書いてるということは>>470>>467とは別人だけど
>>467と同じく>>466の最後の行が間違ってると思ってるってことでいいの?
480132人目の素数さん:2011/01/05(水) 20:32:16
>>478
>>476の式を書かなければ、違う問題を解いていることになるから>>470

>>479
間違っているとは思っていない。
481466:2011/01/05(水) 21:13:57

何も聞かれてないのに答えてスマソだが、

最後の行は、零点から遠ざかるほど大きくなるから…
482132人目の素数さん:2011/01/06(木) 09:27:48
馬鹿どもが騒いでたようだが正月くらいもちっと心に余裕を持てや
483132人目の素数さん:2011/01/06(木) 09:33:03
>>480
何故>>466に言わん?
484132人目の素数さん:2011/01/06(木) 09:44:38
数学勉強すれば>>467の意味がわかるんじゃないの
485132人目の素数さん:2011/01/07(金) 20:14:12
>>467の意味が分からないとは一言も書いてないのだが?
486132人目の素数さん:2011/01/09(日) 04:17:14
>>470の意味が分からない
487132人目の素数さん:2011/01/23(日) 02:23:27
あげ
488132人目の素数さん:2011/02/03(木) 19:02:29
機械Xと互い重さの違う重り1〜nがあるとする
また「1&2」「1&3」…「1&n」「2&3」…「2&n」…「n-1&n」とn(n-1)/2個の項目がある
パンチカードが何枚かあるとする
機械Xは例えば「1&2」「2&5」「2&3」の項目に穴を開けられたカードを読み取れば
まず重り1と2を比べ、次に2と5を比べ、最後に2と3を比べその結果を
「1<2」「2>5」「2<3」という風に出力することが出来るとする

パンチカードがm枚しか無かったとき
重りを比べる操作をk回以上行わせないと、どの重りが一番重いか分からないとき
k=f(n,m)とおくことで関数f:N×N→Nを定義する

例えばn=3の場合を考える
1枚目のパンチカードは「1&2」の項目にだけ穴をあけ機械に読み取らせ
出力が「1<2」なら「2&3」、出力が「1>2」なら「1&3」と
2枚目のパンチカードの項目に穴をあけて読み取らせば
機械に2回重りを比べる操作をさせるだけで重り1,2,3のどれが一番重いか分かる
しかしパンチカードが1枚だけしか無いなら「1&2」「1&3」「2&3」に穴をあけて読み取らせ
機械に3回重りを比べる操作をさせないとどれが一番重いか分からない
このように考えてけばf(3,1)=3 f(3,2)=f(3,3)=f(3,4)...=2だと分かる

(1) f(n,1)は幾つ?
(2) mが十分大きいならf(n,m)は幾つ?
(3) f(n,2)は幾つ?
489132人目の素数さん:2011/02/04(金) 01:55:45
>>488
(2) は直感としておそらく n-1 だな。 トーナメントの試合数と同じだろう。
m が log_2(n) を下回らない最小の整数(切り上げ)以上であればk = n-1。

(1)と(3)は考え中 
490132人目の素数さん:2011/02/04(金) 15:33:26
>>488
とりあえず(1)はn * (n - 1)
重りが4つだった場合、重い順に番号をつけておく

1 2 3 4

1が一番重いという事を知るには最低でも2番目と比較しないといけない
(1と2を直接比較しないと、他の全ての比較を行ったとしても1が一番重い確証を得られない)

どの重りも1番重い重りと2番目に重い重りに成り得るわけだから
いかなる場合でも一番重い重りを知るためには、全部の比較をしないといけない

だから、n * (n - 1)
491132人目の素数さん:2011/02/04(金) 15:38:46
>>488
間違えた n * (n - 1) / 2 だった

あとは、グループに分けて比較すれば(2)もできそう

(1 2 3) (4 5 6) (7 8 9) みたいにして、2段階のトーナメント
1段階目と2段階目の同時対戦人数をどうするかが問題だけど、
あとは機械的に解けそう
492132人目の素数さん:2011/02/06(日) 00:18:32
二段階トーナメント式で一回戦のグル−プ数をX個
それぞれのグループ内の重りの数をY個orY+1個ずつになるようにする
ある自然数aについて
a^2(a-1)/2≦n≦a^3/2のときX=-[-n/a]
a^3/2<n≦a^2(a+1)/2のときX=[n/a]
a^2(a+1)/2<n<a(a+1)^2/2のときX=a(a+1)/2
Y=[n/X]であり、計算するといずれの場合もY=aになる
一回戦は計 (X-(n-aX))*a(a-1)/2 + (n-aX)*a(a+1)/2
二回戦は X(X-1)/2 この二つを足したものがたぶん答え
ちゃんと計算したわけじゃないので間違ってる可能性は高い
493132人目の素数さん:2011/02/06(日) 22:38:25
>>489-492
(1)(2)は正解です
(1)は490のやり方でnC2未満にはならない事を示せますし
(2)はトーナメントと同様に考えればn-1未満にならない事を示せる
(3)は491のやり方で機械の重り比較回数を最小に出来るか
実は自分にも分かってないので正解だと保障出来ません
すいません
494132人目の素数さん:2011/02/07(月) 18:02:11
>>492の方法では、
n=14の時(a=2の第二分類)の一回戦のグループ数=4
n=35の時(a=4の第二分類)の一回戦のグループ数=8
となっていると思います。
a≦4で調べたところ、ほとんどで、私の結果と一致しますが、上では異なり、
それぞれ、もう1グループ多い方が、比較回数を少なくできると思いますがどうでしょう?
495132人目の素数さん:2011/02/07(月) 23:03:15
│7│7│7│7│7│7│7│7│7│7│7│7│7│7│7│7│7│7│7│ 
├─┼─┼─┼─┼─┼─┼─┼─┼─┼─┼─┼─┼─┼─┼─┼─┼─┼─┼─┼ 
│6│6│6│6│6│6│6│6│6│6│6│6│6│6│6│6│6│6│6│ 
┝━┿━┿━┿━┿━┿━┿━┿━┿━┿━┿━┿━┿━┿━┿━╋─┼─┼─┼─┼ 
│5│5│5│5│5│5│5│5│5│5│5│5│5│5│5┃5│5│5│5│ 
├─┼─┼─┼─┼─┼─┼─┼─┼─┼─┼─┼─┼─┼─┼─╋━┿━╋─┼─┼ 
│4│4│4│4│4│4│4│4│4│4│4│4│4│4│4│4│4┃4│4│ 
├─┼─┼─┼─┼─┼─┼─┼─┼─┼─┼─┼─┼─┼─┼─┼─┼─╂─┼─┼ 
│3│3│3│3│3│3│3│3│3│3│3│3│3│3│3│3│3┃3│3│ 
├─┼─┼─┼─┼─┼─┼─┼─┼─┼─┼─┼─┼─┼─┼─┼─┼─╂─┼─┼ 
│2│2│2│2│2│2│2│2│2│2│2│2│2│2│2│2│2┃2│2│ 
├─┼─┼─┼─┼─┼─┼─┼─┼─┼─┼─┼─┼─┼─┼─┼─┼─╂─┼─┼ 
│1│1│1│1│1│1│1│1│1│1│1│1│1│1│1│1│1┃1│1│ 
├─┼─┼─┼─┼─┼─┼─┼─┼─┼─┼─┼─┼─┼─┼─┼─┼─╂─┼─┼ 
│0│1│2│3│4│5│6│7│8│9│10│11│12│13│14│15│16┃17│18│ 
└─┴─┴─┴─┴─┴─┴─┴─┴─┴─┴─┴─┴─┴─┴─┴─┴─┸─┴─┴ 
碁盤状のマスを書き、上のように数字を書く。(1段目k列には k-1、m段目には全てm-1)
下寄せ、かつ、左寄せ の原則に従って、碁石をn個置く。その時、碁石によって隠された数字の合計が最小になるようにする。その最小値がf(n,2)。
上はn=100のときの碁石が置かれる場所を太い罫線で囲っている。
n=104では、右上の二つの5と左上の二つの6を覆うか、5を一つ除き、隣の列の「432117」を覆う。
496132人目の素数さん:2011/02/07(月) 23:08:34
ビジュアルに訴える直感的方法です。ずれてしまったので、もう一度、表をアップします。

│07│07│07│07│07│07│07│07│07│07│07│07│07│07│07│07│07│07│07│ 
├─┼─┼─┼─┼─┼─┼─┼─┼─┼─┼─┼─┼─┼─┼─┼─┼─┼─┼─┼ 
│06│06│06│06│06│06│06│06│06│06│06│06│06│06│06│06│06│06│06│ 
┝━┿━┿━┿━┿━┿━┿━┿━┿━┿━┿━┿━┿━┿━┿━╋─┼─┼─┼─┼ 
│05│05│05│05│05│05│05│05│05│05│05│05│05│05│05┃05│05│05│05│ 
├─┼─┼─┼─┼─┼─┼─┼─┼─┼─┼─┼─┼─┼─┼─╋━┿━╋─┼─┼ 
│04│04│04│04│04│04│04│04│04│04│04│04│04│04│04│04│04┃04│04│ 
├─┼─┼─┼─┼─┼─┼─┼─┼─┼─┼─┼─┼─┼─┼─┼─┼─╂─┼─┼ 
│03│03│03│03│03│03│03│03│03│03│03│03│03│03│03│03│03┃03│03│ 
├─┼─┼─┼─┼─┼─┼─┼─┼─┼─┼─┼─┼─┼─┼─┼─┼─╂─┼─┼ 
│02│02│02│02│02│02│02│02│02│02│02│02│02│02│02│02│02┃02│02│ 
├─┼─┼─┼─┼─┼─┼─┼─┼─┼─┼─┼─┼─┼─┼─┼─┼─╂─┼─┼ 
│01│01│01│01│01│01│01│01│01│01│01│01│01│01│01│01│01┃01│01│ 
├─┼─┼─┼─┼─┼─┼─┼─┼─┼─┼─┼─┼─┼─┼─┼─┼─╂─┼─┼ 
│00│01│02│03│04│05│06│07│8│9│10│11│12│13│14│15│16┃17│18│ 
└─┴─┴─┴─┴─┴─┴─┴─┴─┴─┴─┴─┴─┴─┴─┴─┴─┸─┴─┴ 
497132人目の素数さん:2011/02/08(火) 03:27:49
ごちゃごちゃした計算を繰り返しましたが、一つの形になりました。
一回戦で m(m≧2) のグループに分けるのが最適である n のうち、
最小の n は、(m-1)(p+1)-p(p-1)/2     ただし、p=[{1+√(8m-15)}/2]
最大の n は、   m (q+1)-q(q-1)/2     ただし、q=[{1+√(8m+ 1)}/2]

で与えられそうです。(調査した結果と例外なく一致しております。)
これを使って、m=2,3,4,...,30に対応する最小と最大のnを計算すると、
___ m:02,03,04,05,06,07,08,09,10,11,12,13,14,15,16,017,018,019,020,021,022,023,024,025,026,027,028,029,030
min n:02,05,08,13,17,21,29,34,39,44,56,62,68,74,80,097,104,111,118,125,132,155,163,171,179,187,195,203,233
max n:05,09,13,17,24,29,34,39,50,56,62,68,74,90,97,104,111,118,125,147,155,163,171,179,187,195,224,233,242
となります。

496でもずれてしまいましたが、だいたいokだったので、気持ちで見てください。
498132人目の素数さん:2011/02/08(火) 04:05:55
またずれたので、再掲&補足
分割数 m:02,03,04,05,06,07,08,09,10,11,12,13,14,15,16,017,018,019,020,021,022,023,024,025,026,027,028,029,030
最小の n:02,05,08,13,17,21,29,34,39,44,56,62,68,74,80,097,104,111,118,125,132,155,163,171,179,187,195,203,233
最大の n:05,09,13,17,24,29,34,39,50,56,62,68,74,90,97,104,111,118,125,147,155,163,171,179,187,195,224,233,242

多くの場合 m=kでの「最小のn」とm=k-1での「最大のn」が一致しているが、後者が大きくなっている場合もある。
小さいところでの好例は、n=21〜24で、21 = 4+4+4+3+3+3 = 3+3+3+3+3+3+3など。
7列目の3段を追加するときに必要なコストは 06+01+02 = 9 で、これは、4段目の3つの03のコストと一致している
│04│04│04│04│04│04│04 │04│04│04│04│04│04│04 │04│04│04│04│04│04│04 
├─┼─┼─┼─┼─┼─┼─ ├─┼─┼─┼─┼─┼─┼─ ├─┼─┼─┼─┼─┼─┼─ 
│03│03│03│03│03│03│03 │xx│xx│xx│03│03│03│03 │03│03│03│03│03│03│03 
├─┼─┼─┼─┼─┼─┼─ ├─┼─┼─┼─┼─┼─┼─ ├─┼─┼─┼─┼─┼─┼─ 
│02│02│02│02│02│02│02 │xx│xx│xx│xx│xx│xx│02 │xx│xx│xx│xx│xx│xx│xx 
├─┼─┼─┼─┼─┼─┼─ ├─┼─┼─┼─┼─┼─┼─ ├─┼─┼─┼─┼─┼─┼─ 
│01│01│01│01│01│01│01 │xx│xx│xx│xx│xx│xx│01 │xx│xx│xx│xx│xx│xx│xx 
├─┼─┼─┼─┼─┼─┼─ ├─┼─┼─┼─┼─┼─┼─ ├─┼─┼─┼─┼─┼─┼─ 
│00│01│02│03│04│05│06 │xx│xx│xx│xx│xx│xx│06 │xx│xx│xx│xx│xx│xx│xx 
└─┴─┴─┴─┴─┴─┴─ └─┴─┴─┴─┴─┴─┴─ └─┴─┴─┴─┴─┴─┴─ 
真ん中のn=21の状態に対し、静的に石を追加するのか、右側のn=21に対し石を追加するか、どちらも可能。
そのような状態が4回連続するため。
499132人目の素数さん:2011/02/11(金) 05:35:12
>>496-497
計算結果と碁盤表示の仕方は分かりました
こういう風に計算出来ると…
こりゃf(n,3)の計算は更に複雑でどうしようもなくなりそうですな
問題に出さなくて良かった
500132人目の素数さん:2011/02/25(金) 08:04:19.54
あれ?
100人が1〜100の数字のうちひとつをそれぞれ選ぶ
最大の数字を出した人が勝ち
ただし、同じ数字を複数の人が出したらダメ
幾つを選ぶと、勝つ確率が高いか?
みたいな問題が出たなかったっけ? 
オレの夢か? それとも 別のスレの記憶間違いなのか?
501132人目の素数さん:2011/02/25(金) 20:08:54.65
age
502132人目の素数さん:2011/02/25(金) 22:33:49.72
>>500
分からない問題はここに書いてね350
http://kamome.2ch.net/test/read.cgi/math/1296285009/675
503500:2011/02/26(土) 00:51:26.45
それそれ。
ルールを追加したら、他人の思考を定量化できるかなと思った。

自分以外の人は以下の作戦で数字を決めるとする。

1) 最大の数字(100)を書こうとする
2) 確率aで (0<a<1) 今書こうとした数字を実際に書く → 終わり(書く数字が決定)
3) それ以外は、「いやいやこの数字は他の人が書きそうだ」と思いとどまり
  思い浮かべていた数字よりもひとつ小さな数字を書こうとする
4) → 2)へ

とりあえずは確率aではなく 簡便のため1/2とかでもいいかもしれない。
 
504132人目の素数さん:2011/02/26(土) 00:55:38.15
ただの確率じゃ扱えんだろ
505132人目の素数さん:2011/02/26(土) 01:13:58.63
なんで?
506132人目の素数さん:2011/02/26(土) 11:40:02.99
平面上に、「O」を実数と同じ個数だけ、どの2つも交わらないという条件で書くには、原点中心で半径rの円を、rを正の実数全体で動かして書けばいい。(書いた結果平面が真っ黒になって円を認識できないとかいうのは、この際関係無し。)
では、「T」を同じ条件で書くことは可能か?
507132人目の素数さん:2011/02/26(土) 12:33:28.74
>>503
途中まで、nを選ぶ確率をP(n)とすると
P(n) = a(1-a)^(100-n) (n >= 2)
P(1) = 1-a(1-a)^98
この後はかなり厄介。
508132人目の素数さん:2011/02/26(土) 13:15:35.43
>>506
> (書いた結果平面が真っ黒になって円を認識できないとかいうのは、この際関係無し。) 

半径2rの円を書けば隙間もできると言えるか?
509132人目の素数さん:2011/02/26(土) 14:21:11.23
>>506
可能。
Tの右半分、左半分の余白に入れ子状にTを配置するようにすれば、
無限大の二分木の構造になるので、
実数の2進表記と対応させればいい

510132人目の素数さん:2011/02/26(土) 14:23:36.63
>>509
それだと実数が加算濃度にならんか?
511132人目の素数さん:2011/02/26(土) 19:10:53.95
>>506
その場合、負の実数に対応するものはどうなりますか
512132人目の素数さん:2011/02/26(土) 19:42:48.62
>実数の2進表記
513132人目の素数さん:2011/02/26(土) 19:43:59.84
>>509
その場合、非整数に対応するものはどうなりますか
514132人目の素数さん:2011/02/26(土) 19:52:07.61
>>508
rも2rも、正の実数全体を渡らせれば同じだよ。

>>509
それだと有限小数しか対応しないので可算無限になる。

>>511
正の実数全体と実数全体を対応させるのは簡単。
たとえば f(x) = log x で対応付けられる。
515132人目の素数さん:2011/02/26(土) 20:00:06.98
>>514
実際に円を重ならないように描くことについてです
516132人目の素数さん:2011/02/26(土) 20:17:02.29
「T」を真ん中の点から三つの線分が飛び出てる図形と考えて
真ん中の点を「T」の中心、三つの線分のうち一番短い奴の長さを「T」のサイズと呼ぶことにする
そして>>506のように「T」を連続濃度だけ平面に詰めれたとする
このとき実数a>0があってサイズがa以上の「T」が平面に連続濃度だけ入ってると考えられる
次にaよりずっと小さい実数b>0を適当にとって平面を可算個の直径bの円で覆えば
そのうちの一つの円には連続濃度のサイズa以上の「T」の中心が入ってることになる

後はa>>bならば直径bの円の中に点を一定個数おいて
それらの点を中心とするサイズaの「T」を考えると
「T」のうち2つは交わってることを証明すればいい
517132人目の素数さん:2011/02/26(土) 20:18:34.27
>>515
言いたいことがいまいち分からない。
518132人目の素数さん:2011/02/26(土) 21:02:04.46
>>506
平面上に、「O」を実数と同じ個数だけ、どの2つも交わらないという条件で書くには、原点中心で半径rの円を、rを正の実数全体で動かして書けばいい。

これだと負の実数を考慮してないと思ったので>>511で尋ねましたが
>>514でいただいた答えだと、実数の個数だけ円を描く問題で負の実数が考慮されてないことに対しての
答えになってないと思うんです。
519132人目の素数さん:2011/02/26(土) 21:18:34.08
>>518
実数xに対して半径e^xの円を描けばいい。>>514の最後はそういう意味だろう。
520132人目の素数さん:2011/02/26(土) 21:22:42.84
なるほど
わかりました。逆関数の対数なので何を言おうとしてるのかピンときませんでした
補足ありがとうございます
521132人目の素数さん:2011/02/26(土) 22:13:21.78
一般的に、0でない面積を持つ図形を、境界及び内部の重なり無く、
平面に負荷算無限個配置することができる例はあるの?
522132人目の素数さん:2011/02/26(土) 22:30:00.15
ない。
証明は重ならない区間が可算であることと同じ。
523132人目の素数さん:2011/02/26(土) 22:54:23.50
>>522
ありがと。
じゃあTをスキマ無く敷き詰められない時点で
ダメなんじゃ無いの?
524132人目の素数さん:2011/02/26(土) 23:17:42.69
>>523
何を言っているのかわからない
525132人目の素数さん:2011/02/27(日) 11:14:40.52
>>516で証明できてるのかな?
じゃあ、Tを連続的に変形した形は全部Tとみなすならば、どうだろうか。
526132人目の素数さん:2011/02/27(日) 13:50:35.61
>>500を見て思い出したんだけど、

・2人が同時に正の整数を書く。
・小さい数を書いた方に1点。
 ・ただし差が1の場合は大きい数を書いた方に2点。
 ・また差が0の場合はノーカウント。
このゲームの最善戦略はなにか。

という問題。
527132人目の素数さん:2011/02/27(日) 21:44:53.32
>>525
>連続的に変形した

とは何を指すの?
トポロジー的に一致していれば同形? 
(Tと F E G Y とか。 端点と分岐点の関係が同じなら)

あ、Gは書体によっては同一かどうか微妙だな。
528132人目の素数さん:2011/02/28(月) 00:08:09.85
>>525
ある図形Xに対して以下の条件を満たす
単射な連続関数f_1,f_2,f_3 : [0,1] → R^2があるとき、Xを「T」型の図形と呼ぶことにする
・1≦i<j≦3なら「f_i(x)=f_j(y)⇔x=y=0」が成り立つ
・f_1([0,1])∪f_2([0,1])∪f_3([0,1]) = X
またこのときf1(0)をXの中心、min{i=1,2,3 | d(f_i(0),f_i(1))} をXのサイズと呼ぶことにする

R^2内に連続濃度の「T」型の図形を互いに交わらずに埋め込めたとする
このとき>>516と同様に考えることで、ある実数a>0と直径aの円Yがあって
円Yの内部に中心を持つサイズa以上の「T」型の図形が連続濃度だけあることが分かる
それらの図形に対して角度を以下のように定める(円Yの中心はOとする)
「上のf_1,f_2,f_2をとってきてi=1,2,3に対しP_i=f(min{x∈[0,1] |f(x)は円Yの境界(円周)に含まれる})
 とするとき線分P_1P_2、P_1P_3、P_2P_3の中で一番短い物をP_iP_jとした時の∠P_iOP_j」
するとある実数b>0があって上記の「T」型の図形の中で
角度がb以上2b未満の物が連続濃度あることが分かる

あとは中心を円Yの内部に持つ角度がb以上2b未満の「T」型の図形が可算個あれば
そのうちの2つは交わってる事を証明すればいいが説明すんのめんどい
529132人目の素数さん:2011/02/28(月) 00:58:24.99
幅狭いTの下端が半径1の周上にあるように外側に放射状に並べれば
非可算濃度にならんか?
530132人目の素数さん:2011/02/28(月) 02:20:09.05
△ABCの∠ABCの二等分線上に点D 辺BC上に点Eをとったとき
AB=BC=BE、BD=DE=ECとなった ∠EABの大きさを求めよ
531132人目の素数さん:2011/02/28(月) 02:30:53.53
72度であってます?
532530:2011/02/28(月) 02:33:53.27
すいませんミスですorz
求めるのは∠EABじゃなくて∠EBAです
533132人目の素数さん:2011/02/28(月) 02:38:08.21
BC上に点Eをとったとき BC=BE てことは EとCは同一点てこと。
BD=DE=EC てことは BとDとEも同一点。
BとCは同一点なのでABCは三角形ではない。
∠EABは0度。

おそらくは何か間違っている
534530:2011/02/28(月) 02:39:16.19
訂正:△ABCの∠ABCの二等分線上に点D 辺BC上に点Eをとったとき
AB=AC=BE、BD=DE=ECとなった ∠EABの大きさを求めよ
535132人目の素数さん:2011/02/28(月) 22:07:05.19
>>534
EはBC上にあることと、AB=AC=BEであることから
△ABCは正三角形。
よってCとEは同一点。
∠EABは60度
536132人目の素数さん:2011/02/28(月) 22:14:04.70
>>535
アホ

>>534
記号変えただけのマルチ

38:132人目の素数さん :2011/02/27(日) 02:20:28.51
△ABCのBC上に点P、∠ABCの二等分線上に点Qをとった
AB=AC=BP、BQ=QP=PCのとき
∠ABCを求めよ

という問題なのですが 三角関数無しで解けると言われたのですが
どうすれば解けるかどなたか教えていただけないでしょうか。

小中学校範囲の算数・数学の問題のスレ Part 42
http://kamome.2ch.net/test/read.cgi/math/1298469455/
537132人目の素数さん:2011/02/28(月) 22:19:22.58
>>530
>>532
>>534

糞マルチ
記号の書き換えも上手く出来ないのかよw
538530:2011/03/01(火) 00:40:55.75
>>532 20°
539132人目の素数さん:2011/03/01(火) 13:09:29.82
>>536
小中学校範囲〜で丸一日レス付かなかったからこちらに書いたんだろ?
鬼の首でもとったようにマルチマルチと騒ぐバカはなんなんだ?

>三角関数無しで解けると
3角関数なんて使って解く事の方が難しいんでないかい。
てか、3角関数を使ってうまく解ける方法が有るなら教えて欲しいものだ。
ある程度正確に図を書いてにらめっこしたら中2範囲で解けるよ。
答えは40度
540132人目の素数さん:2011/03/02(水) 18:02:23.89
y=2x^2とy=6xのグラフで
造られる最小のめんせき
541132人目の素数さん:2011/03/02(水) 18:51:22.35
1日の睡眠時間がミリ秒単位で同じ人が日本に少なくとも2人いることを示せ。
542132人目の素数さん:2011/03/02(水) 19:41:11.99
鳩の巣原理。
543132人目の素数さん:2011/03/03(木) 00:38:30.28
>>541
徹夜している人が少なくとも2人いることを示せばよい
544132人目の素数さん:2011/03/03(木) 00:50:53.32
>>542
日本にいる人は
24×60×60×1000人より多い

ですね?
545132人目の素数さん:2011/03/03(木) 00:54:04.79
三年寝太郎
546132人目の素数さん:2011/03/05(土) 01:53:26.24
0からx(自然数)までの総和をW(x)と表すとする
座標平面上で
P(n)は(w(n),w(s-n))となる点を表すとする ※sは2以上の自然数で定数
そしてP(a)とP(a+1)を通る直線とX軸との交点をQ(a)とする

このときQ(x):Q(y)をx,yを用いて表せ
547132人目の素数さん:2011/03/05(土) 01:56:34.21

訂正:Q(a)は交点のxの値 
548132人目の素数さん:2011/03/05(土) 05:23:02.84
549132人目の素数さん:2011/03/05(土) 09:06:18.63
>>541
ミリ秒までの正確さで「睡眠時間」の定義はそもそもできない。
550132人目の素数さん:2011/03/05(土) 13:46:18.79
>>546

 P(a) = (w(a),w(s-a)) と P(a+1) = (w(a)+(a+1), w(s-a)-(s-a))
を通る直線は
 X = w(a) + {(a+1)/(s-a)}{w(s-a) - Y},
題意により (X,Y) = (Q(a),0) を通るから、
 Q(a) = w(a) + {(a+1)/(s-a)}w(s-a)
    = {(a+1)/2}{a + (s-a+1)}
    = {(a+1)/2}(s+1),
∴ Q(x):Q(y) = (x+1):(y+1),

蛇足だが、
 P = (X,Y)は 放物線 √(2X + 1/4) + √(2Y + 1/4) = s+1, 上にあるらしい・・・
551132人目の素数さん:2011/03/05(土) 16:12:56.85
>>549
ならば余計に被りやすくなるだけだ
552132人目の素数さん:2011/03/05(土) 18:41:16.67
>>551
逆だ。判定がつかない以上、そんな人はいない。
553132人目の素数さん:2011/03/05(土) 21:09:09.77
>>550
正解
答えが意外ときれいになるのが個人的に好き
554132人目の素数さん:2011/03/05(土) 22:41:49.27
〔問題〕
2011 は、引き続く11個の素数の和であり、
また、引き続く3個の素数の和でもあるという。

 2011 = p_a + p_(a+1) + p_(a+2) + ・・・・・ + p_(a+10)
    = p_b + p_(b+1) + p_(b+2),

このとき、素数 p_a と p_b を求む。

出典
 小川洋子「素数は私を裏切らない」文藝春秋, p.89 (2011/03)
555132人目の素数さん:2011/03/05(土) 23:07:10.36
556132人目の素数さん:2011/03/05(土) 23:07:30.35
157,661
557132人目の素数さん:2011/03/06(日) 21:19:00.63
2つのパーツから成る知恵の輪は、
迷路の探索に例えると何次元か?
558132人目の素数さん:2011/03/07(月) 14:06:45.49
3次元
559132人目の素数さん:2011/03/07(月) 14:11:37.51
1^2+(1^2+3^2)+(1^2+3^2+5^2)+・・・・・+{1^2+3^2+5^2+・・・・・+(2n-1)^2}を簡単にせよ。
560132人目の素数さん:2011/03/07(月) 14:23:05.60
n(n+1)(2n^2+2n-1)/6
561132人目の素数さん:2011/03/07(月) 15:23:06.68
>>560正解
562132人目の素数さん:2011/03/10(木) 02:23:54.39

p[b]について、
3つの連続する素数の和が2011に
なるというのだから、
2011÷3=670…1
つまり、こう考えればいい。
p[b]=670+s[b]
p[b+1]=670+s[b+1]
p[b+2]=670+s[b+2]
(s[n]は整数値をとる)

計算省略して、
s[b]+s[b+1]+s[b+2]=1

後は670と倍数かつ和が素数に
なるように計算すればいい。
よって、p[b]=667(おそらく)

p[a]に関しても同様。
但し、サイトから素数表探して
やるのが一番かと。

2011÷11=182…9

計算省略して、
Σ[n=a,a+10]s[n]=9

後は任せた。
563132人目の素数さん:2011/03/10(木) 02:36:45.24
>>556
正解
564132人目の素数さん:2011/03/10(木) 06:36:05.09
なんかユニークな問題見つけた。
キャスフィより(チャレンジ問題 -31)
半径r,中心O,中心角θ<πの扇形の重心をGとする。
OGを求めよ。
565132人目の素数さん:2011/03/10(木) 09:23:07.28
>>564
Oをxy平面上の原点に、OGがx軸の+方向に重なるように
x軸、y軸を設定する
OGを求めるには
{(x,y)|x^2+y^2≦r^2,tan(-θ/2)≦y/x≦tan(θ/2)}での重積分
(∬xdxdy)/(∬dxdy)を計算すればよい
(∬xdxdy)/(∬dxdy)=4r*sin(θ/2)/3θ
566猫は村八分 ◆MuKUnGPXAY :2011/03/10(木) 10:55:09.96
村八分やナ。


567132人目の素数さん:2011/03/10(木) 13:45:40.82
s
568じゅー:2011/03/10(木) 13:47:34.77
>>565
実はパップスギュルダンとガバリエリの原理で出来ます。
答え↓↓
http://www.casphy.com/bbs/test/r.cgi/highmath/1165998931/
569132人目の素数さん:2011/03/10(木) 17:37:55.62
AB=ACとなる二等辺三角形ABCの 辺AB上にBC=CDとなる点Dをおく。 また、辺AC上にAD=CEとなる点Eをおく。 その時∠CDE=50°となる。 ∠Aの角度を求めよ。
570132人目の素数さん:2011/03/10(木) 17:42:12.20
初等幾何で解く方法が見つかっていません。
解けた方いらっしゃいましたら是非解法を教えてください。
571132人目の素数さん:2011/03/10(木) 20:05:28.22
>>568
そんな定理があるんですか。
初めて知りました。
572じゅー:2011/03/10(木) 21:46:04.77
重心の移動距離*移動させる図形の面積=回転体の体積
というものです。
最近では中学受験とかの進学塾でも教えることがあるようです。
573132人目の素数さん:2011/03/11(金) 00:23:30.94
>>569
解けねえ
574132人目の素数さん:2011/03/11(金) 01:05:27.84
扇形の内心と外心はいかがだぜ?
575132人目の素数さん:2011/03/11(金) 01:45:38.18
>>571-572
プププ
576132人目の素数さん:2011/03/11(金) 06:33:26.92
AD=CE->BD=CE
2y+50=180
x=180-2y=50
577132人目の素数さん:2011/03/11(金) 09:23:41.06
>>576
不正解です。
一行目のようにはなりません。
578132人目の素数さん:2011/03/11(金) 10:25:06.35
>>569
三角関数使って40度と確認できた。まだ幾何では考え中
579132人目の素数さん:2011/03/11(金) 11:03:44.74
>>578
この程度もサクッと解けんのか雑魚が!
と思ったら、結構難しいじゃないか…
580132人目の素数さん:2011/03/11(金) 14:11:26.68
>>579
Yahoo知恵袋にこの問題が貼られてから、
同サイト内やその他のコミュニティサイトなどで話題になっていますが、
まだどなたも幾何で解けていないようです。
581132人目の素数さん:2011/03/11(金) 19:13:40.25
補助線で平行四辺形はいっぱいできるのに…
582132人目の素数さん:2011/03/11(金) 20:08:03.17
とりあえず∠Aが50°(DEが中点連結定理を満たすと仮定した場合)
というのが矛盾しているという証明

△ADC∽△DECより、AC:DC=DC:EC
これとEC=AC/2よりAC:DC=DC:AC/2 ∴(√2)CD=AC
CD=BC、AC=ABより(√2)BC=AC=AB

∠A=50°より余弦定理から
cos(A)=cos(50°)=(AB^2+AC^2-BC^2)/(2*AB*AC)
=(2BC^2+2BC^2-BC^2)/(2*2BC^2)=3/4

cosのグラフは[0,π]の範囲で単調減少であるが
cos(45°)=(√2)/2、cos(50°)=3/4
(√2)/2<3/4であるので矛盾■
583132人目の素数さん:2011/03/11(金) 21:13:04.52
初等幾何だから三角関数使えないんじゃないの
584132人目の素数さん:2011/03/11(金) 21:35:19.53
>>583
>>582
http://detail.chiebukuro.yahoo.co.jp/qa/question_detail/q1251895847に
>∠A=50°というのは矛盾のない一つの解であろうと思われます。
とあったので矛盾しているということを示しただけです。

そもそも問題に無い条件を仮定して導いた解答ですので
初等幾何を使う云々の前に証明として成立していないわけです。
585132人目の素数さん:2011/03/11(金) 21:55:59.35
>>584
あっ、すいません。
勘違いしてましたorz
586132人目の素数さん:2011/03/12(土) 15:19:20.13
>>569

BC=a, AB=AC=b とおく。
題意より CD=a, ∠B = 90゚ - (1/2)∠A, sin(A/2) = a/2b,
△ABC ∽ △BCD より BD = a^2 /b,
∴ CE = AD = b - (a^2)/b,
∠DCE = ∠B - ∠A = 90゚ - (3/2)∠A,

∠CDE = θ とおくと
∠DEC = 180゚ - {90゚ - (3/2)A} - θ = 90゚ + (3/2)A - θ,

正弦定理より
 sinθ/cos(θ-(3/2)A) = CE / CD
これと sin(A/2) = a/2b より
 tanθ = cos(A){1-2cos(A)}^2/{tan(A/2)cos(180゚-3A)},

A=40゚ のとき θ=50゚ になる。(終)
587132人目の素数さん:2011/03/12(土) 21:56:04.15
>>586 (補足)

 1-2cos(A) = 3-4cos(A/2)^2 = -cos(3A/2)/cos(A/2),
 {1-2cos(A)}^2 = {1+cos(3A)}/{2cos(A/2)^2},
を上式に代入して
 tanθ = {1+cos(3A)}/{tan(A)cos(180゚-3A)}

A=40゚ のとき 3A=120゚, tanθ = 1/tan(40゚) = tan(50゚),
588132人目の素数さん:2011/03/13(日) 00:57:16.67
大量の正方形のタイルを平面の上に隙間なく敷き詰める。
はじめ、1枚のタイルは赤色くほかのタイルは全て白い。
これらの白いタイルは赤いタイルに1秒間隣接すると赤く変色するという。
また、赤いタイルは1秒経過する毎に1/2の確率で黄色く変色するという。
nを自然数として、n秒後の黄色いタイルの枚数の期待値を求めよ。
589132人目の素数さん:2011/03/13(日) 01:09:57.34
黄色いタイルは変色せんの?
590132人目の素数さん:2011/03/13(日) 01:10:22.54
>>589
しない
591132人目の素数さん:2011/03/13(日) 05:06:58.53
そんなタイルは存在しない
592132人目の素数さん:2011/03/13(日) 12:00:31.61
最初のタイルが、n秒後に黄色に変色している確率は、1-(1/2)^n
k秒後に白→赤と変色したタイルは4k枚
そのタイルが、n-k回(n-k秒間)の 赤→黄 という変色機会を免れ、赤のままでいる
確率は(1/2)^(n-k)なので、黄色へ変色している確率は 1-(1/2)^(n-k)

1-(1/2)^n+Σ[k=1,n-1]4k*(1-(1/2)^(n-k))=2n^2-6n+9-9/2^n
593132人目の素数さん:2011/03/13(日) 21:16:42.33
>>587
条件からその角度だけが解となることを示されていないと思われます。

一応、途中まで、計算できない?
AB=l、AC=1とおく
△ABC∽△CBDから
BD=AE=1/l、AD=EC=l-1/l
∠ABC=αとおくと△ADEに正弦定理を用いて
1/sin(180°-2α) = l/sinα
∴l = 1/(2*cosα)・・・@
△CEDに正弦定理を用いて
(l-1/l)/sin50° = 1/sin(310°-3α)・・・A
@Aから
(4*(cosα)^2-1)(sin50°cos3α+cos50°sin3α) = 2cosαsin50°
この式はα=70°(∠A=40°)のとき、成立する。
594132人目の素数さん:2011/03/13(日) 21:29:22.89
>>593
上の一行は間違えましたので取り消します。
595132人目の素数さん:2011/03/14(月) 09:00:47.14
f(x) = (4*(cos(x))^2-1)*(sin50°*cos(3x)+cos50°*sin(3x))/(2*cos(x)) -sin50°、0<x<90
でこの解を求めると、2個の解が存在し
x=29.6103...°、x=70° ∴∠A = 40°, 120.7792...°
596132人目の素数さん:2011/03/14(月) 12:30:25.37
t=tan50°、y=tanx°とすると
yは以下の方程式を満たす
y^5+t*y^4-6*y^3-14*t*y^2+9*y+t=0
597132人目の素数さん:2011/03/16(水) 02:56:34.55
理想的な単三電池を何もない平らな机の上に任意の数だけ配置する。
立てるの禁止。重ねるの禁止。
どの方向に傾けても転がらない配置はあるか?
598132人目の素数さん:2011/03/16(水) 03:47:30.61
板違いでは?
599132人目の素数さん:2011/03/16(水) 09:35:57.02
「傾けても転がらない」と「傾けると転がる」をきちんと数学的に定義すればここでも扱える。
600132人目の素数さん:2011/03/16(水) 14:51:42.22
>>597
乾電池は滑らない?
長さと垂直方向にしか動かないってこと?
601132人目の素数さん:2011/03/16(水) 16:18:40.13
滑ることを無視するなら放射線状に並べればいけるんじゃないか?
602132人目の素数さん:2011/03/16(水) 17:03:20.85
>>601
頭いいな、最低5つでいけるってことか
603132人目の素数さん:2011/03/16(水) 18:07:40.95
>>602
うん
n<5はダメ
恐らく無理じゃないかなぁ
604132人目の素数さん:2011/03/17(木) 01:30:33.57
乾電池がじつは円柱ではないことに注目すれば4個でもいけると思うが。
605132人目の素数さん:2011/03/17(木) 01:31:44.34
すまん「3個でも」の間違い。
606132人目の素数さん:2011/03/17(木) 13:24:31.34
>>604
あの凸使っていいのかw
じゃあ3

2は…ダメだな
607132人目の素数さん:2011/03/18(金) 11:56:40.23
nを自然数とし、数列{a[n]}を以下のように定義する。
a[1]=1
(a[n+1])^2=(a[n]+1)^2

a[n]を全て求めよ
608132人目の素数さん:2011/03/18(金) 12:32:32.80
a[n]=nですかね
609132人目の素数さん:2011/03/18(金) 12:44:12.99
a[n]=n と a[n]=((3*(-1)^(n-1))-1)/2
610132人目の素数さん:2011/03/18(金) 13:06:41.33
>>607 数列が定義できてない。
611132人目の素数さん:2011/03/18(金) 14:37:28.84
a[0]=0とかにすると規則性が美しいんだがな
612132人目の素数さん:2011/03/18(金) 15:58:30.68
α>0 β>0のとき、

α^(2β)≧2α+β^αを満たす実数α、βのうち
βは有限であることを示せ
613132人目の素数さん:2011/03/18(金) 16:29:45.32
>>612
α=2とするといくらでもβをおっきくできないか?
614132人目の素数さん:2011/03/18(金) 17:07:36.28
>>613
なにっ
615132人目の素数さん:2011/03/18(金) 17:12:21.81
>>609
kwsk
616132人目の素数さん:2011/03/18(金) 17:39:37.93
a[2]^2=4だから、a[2]=2なのか、-2なのか、定まらない。
a[3]^2=9か、1だから、a[3]=±3、±1
a[4]^2=16,4,0 →a[4]=±4,±2,0
a[5]^2=25,9,1 →a[5]=±5,±3,±1
...
a[n]=±n、±(n-2)、...、±1か0(nの偶奇による)  (n≧2)の時
一応一言、これは数列とは言わない。
617132人目の素数さん:2011/03/18(金) 18:01:47.08
>>612
糞マルチ
618132人目の素数さん:2011/03/18(金) 18:42:41.01
>>617
>>612
>糞マルチ
619132人目の素数さん:2011/03/18(金) 19:48:10.10
>>618
アホくさい
620132人目の素数さん:2011/03/18(金) 20:32:30.73
1から15まで続けて書くと123456789101112131415となる。これを1つの整数と考えると、この数は21けたで,1が8回使われている。(中略)1から1000まで続けて書いてできる整数の桁数と、その整数に1が何回使われているか求めよ。(98灘中) 

これわからん おせーて
621132人目の素数さん:2011/03/18(金) 20:40:05.69
619 名前:132人目の素数さん :2011/03/18(金) 19:48:10.10
>>618
アホくさい

お前だよ(笑)
622132人目の素数さん:2011/03/18(金) 20:58:11.99
623620:2011/03/18(金) 21:43:39.55
>622
thx

しかし、まだ、1から1000まで続けて書いてできる整数の桁数がわからーん
624132人目の素数さん:2011/03/18(金) 21:59:08.06
>>623
1桁の数は1〜9の9個
2桁の数は10〜99の90個
3桁の数は100〜999の900個
プラス1000の4桁で
1*9+2*90+3*900+4桁じゃない?
625132人目の素数さん:2011/03/18(金) 22:00:22.17
>>615

前の方は a[n+1] = a[n] +1 の解。

後の方は a[n+1] = -(a[n] +1) の解。
 a[n] = 1,  (n:奇数)
 a[n] = -2, (n:偶数)
626132人目の素数さん:2011/03/18(金) 22:06:45.96
>>625
そうすると>>616のような現象に陥る
627132人目の素数さん:2011/03/19(土) 02:50:52.84
>>620
桁数について:
 1〜9の9個は各1桁分、10〜99の90個はは各2桁分、100〜999の900個はは各3桁分、1000は4桁分
これらを合計すればよい。

1の数について:
繋げる前の各数字の桁ごとに分けて考える。
各数字の下一桁(1の位)には1が使われているのは全体の10個に1個の割合である。
各数字の10の位には1が使われているのは全体の10個に1個の割合である。
各数字の100の位には1が使われているのは全体の10個に1個の割合である。
各数字の1000の位には1が使われているのは全体で1個ある。
これらを合計すればいい




628132人目の素数さん:2011/03/19(土) 03:16:06.15
>>621
お前、高校生の質問スレにいたキチガイだろ
629132人目の素数さん:2011/03/19(土) 11:02:35.28
おお。乾電池の答えが出てる。
めっさ適当に問題投げたのに。
>凸無し=5本
点と点で支え合えるルールの場合ね。
点対点の接触は必ずどちらかにずれてしまうルールにしたらどうなるだろう?
多分もう一本余計に要りそう
>凸有り=3本
うーん…分からん…
630132人目の素数さん:2011/03/19(土) 12:06:52.04
点+点接触なしでも5こでいけるだろ。
うまい言葉が見つからないんだが巴型というか
ひねった放射状にすれば。
631132人目の素数さん:2011/03/20(日) 12:39:23.34
>>609
符号ミスってね?
632132人目の素数さん:2011/03/20(日) 21:36:25.14
>>628
アホかw
633132人目の素数さん:2011/03/21(月) 05:13:58.21
>>632
お前の事だよクズ
634132人目の素数さん:2011/03/21(月) 05:27:11.89
>>633
勉強してろ
635132人目の素数さん:2011/03/22(火) 22:13:54.26
>>634
数学出来ないキチガイが
636132人目の素数さん:2011/03/22(火) 22:23:55.44
>>635
お前よりはできる自信あるw
637132人目の素数さん:2011/03/22(火) 23:11:46.42
面白くない問題は他所でやってくれよ
638132人目の素数さん:2011/03/23(水) 00:47:25.05
∫[0,1] x^4(1-x)^4/(1+x^2) dx
639132人目の素数さん:2011/03/25(金) 23:05:31.33
>>638

 x^4 (1-x)^4 /(1+x^2)
 = (x^8 -4x^7 +6x^6 -4x^5 +x^4)/(1+x^2)
 = x^6 -4x^5 +5x^4 -4x^2 +4 -4/(1+x^2),
より
 (与式) = [ (1/7)x^7 -(4/6)x^6 +x^5 -(4/3)x^3 +4x -4arctan(x) ](x=0,1)
 = 1/7 -2/3 +1 -4/3 +4 -π
 = 0.00126448926734961868021375957764
640132人目の素数さん:2011/03/26(土) 00:41:25.13
>>638
∫ [0,1] (x^4 (1-x)^4)/(1+x)^2 dx = 2329/35-96 log(2)~~0.0007278
641132人目の素数さん:2011/03/26(土) 01:39:12.23
>>638
これって大学入試レベルでは解けるの?
642132人目の素数さん:2011/03/26(土) 15:38:07.41
>>641
1/(1+x^2)の積分、tanで置換するヒントが無いとキツいだろうね。
643132人目の素数さん:2011/03/26(土) 16:56:55.00
a[1]=1/2
a[2]=1
a[3]=7/6
a[4]=5/4
a[5]=13/10
a[6]=4/3
a[n]=?
644132人目の素数さん:2011/03/26(土) 20:02:07.44
>>643
a[n]=(3n-2)/(2n)
645132人目の素数さん:2011/03/26(土) 20:46:54.48
>>644
うn
646132人目の素数さん:2011/03/27(日) 05:07:52.06
>>643-644

 a[n] = (1/2) + 2/(5 - 2a[n-1]),
647132人目の素数さん:2011/03/31(木) 18:06:09.28
nを正の整数とする。
地震が発生する間隔の期待値をn日としたとき、
1日あたりの地震が発生する確率を求めよ。
地震が発生する確率はどの日も同じであるものとする。


求める確率は近似値でも良いです。
648132人目の素数さん:2011/04/01(金) 02:21:45.42
>>647

求める確率をpとおく。
間隔がk日以上である確率は (1-p)^k,
間隔がk日である確率は p(1-p)^k,
したがって、間隔kの期待値は
 E{k} = Σ[k=1,∞) k・p(1-p)^k
   = Σ[k=1,∞) k・{(1-p)^k - (1-p)^(k+1)}
   = Σ[k=1,∞) (1-p)^k
   = (1-p)/p
   = n,   (← 題意より)
 p = 1/(n+1),
649132人目の素数さん:2011/04/01(金) 12:03:05.30
「サイコロを振り、偶数の目が出たら掛け金が2倍に、奇数の目が出たら掛け金が0になる」
というゲームを考える。
このゲームに以下のような戦略で挑む。

(1)1円の金を賭ける。
(2)サイコロをふり、偶数が出たらゲームをやめる。
(3)奇数が出たら掛け金の額を今賭けていた額の2倍にして(1)に戻る。

このとき、ゲーム開始前とゲーム終了後の
自分の所持金の変化量の期待値を求めよ。
なお、自分がゲームに使うことのできる金は
いくらでも用意できるものとする。
650132人目の素数さん:2011/04/01(金) 12:11:39.05
1円
651132人目の素数さん:2011/04/01(金) 12:20:49.95
>>649 の戦略の説明文に不備があったようなので訂正します

(1)1円のかけ金を用意する。
(2)用意したかけ金を賭ける。
(3)サイコロをふり、偶数が出たらゲームをやめる。
(4)奇数が出たら今賭けていた金の2倍の額のかけ金を用意して(2)に戻る。
652132人目の素数さん:2011/04/01(金) 14:56:08.55
>>649
奇数でやめる選択肢はナシで
無限にやりつづけることも想定するわけだな
653132人目の素数さん:2011/04/01(金) 14:59:12.81
1枚の紙がある
2本の直線で紙を5等分したい

どうする?
654132人目の素数さん:2011/04/01(金) 15:12:37.84
条件が不明確だなあ…
655132人目の素数さん:2011/04/01(金) 15:12:41.25
合同分割?等積分割?2次元内?
656132人目の素数さん:2011/04/01(金) 15:23:17.89
3次元ない
657132人目の素数さん:2011/04/01(金) 17:04:26.63
三次元内なら直線も三次元直線?
紙の形は?
658132人目の素数さん:2011/04/01(金) 17:05:40.21
適当に決めて
面白い解答求む
659132人目の素数さん:2011/04/01(金) 22:49:14.34
曲面上に直線がokならトーラスを考えれば終わるな。
っていうとこくらいでもうそれ以上に面白い要素はなさそうですが?
660132人目の素数さん:2011/04/01(金) 22:50:25.52
つーかトーラスに一直線で無限個に分割できてめでたし^2
661132人目の素数さん:2011/04/02(土) 22:43:41.64
xy平面上の直線y=xを時刻tにおける速度が(cost,sint)となるように動かしつづける。
この直線はy軸と平行な向きにどのような動きをしているように見えるか?
662132人目の素数さん:2011/04/02(土) 22:47:16.46
>>661
直線はt=0でy=xに一致しており、動かしている間はy=xに
一致しているとは限りません
663132人目の素数さん:2011/04/02(土) 22:51:53.38
(2.3)
(5.7)
(11.13)
(17.19)
を通る関数の存在の是非を示せ
664132人目の素数さん:2011/04/02(土) 22:55:53.11
>>663
(x-2)(x-5)(x-11)(x-17)+(y-3)(y-7)(y-13)(y-19)=0
665132人目の素数さん:2011/04/03(日) 00:50:35.86
>>663
是非をどう問うのだ?
666132人目の素数さん:2011/04/03(日) 01:50:49.39
>>663

 f(x) = (3x+1 - |x-3|) /2,
667132人目の素数さん:2011/04/03(日) 10:59:06.59
座標平面上に4点A(0.1)B(1.1)C(2.4)D(4.3)があります。
直線m:y=ax+bが2つの線分AB,CD(両端含む)の両方に交わります。
このときa,bはいろいろな値を取ります。次の値の最大値、最小値を求めなさい。
(1)a (2)b (3)a+b (4)3a+2b (5)a-2b
668132人目の素数さん:2011/04/03(日) 16:32:35.23
昔、(3)以降の解き方の発想を知った時には小さな感動があったなー
669132人目の素数さん:2011/04/03(日) 16:53:52.71
y=ax^3+bx^2+cx+d
(ただし、a.b.c.dは互いに異なる素数の定数)
この3次関数について、(ab.cd) (ac.bd) (ad.bc) この3点を通るとき、積abcdの最小値を求めよ。
670132人目の素数さん:2011/04/03(日) 16:58:21.88
面白味がなさそうだな…

ちゃんと答えや考え方を用意してから問題出してる?
671132人目の素数さん:2011/04/03(日) 17:22:11.64
Hi
672132人目の素数さん:2011/04/03(日) 17:23:27.66
n,rを自然数とし、
円x^2+y^2=r^2が通る格子点の個数をnとおく。
nは最大値を持たないことを示せ。
673132人目の素数さん:2011/04/03(日) 18:20:02.07
>>669
d|bc.
674132人目の素数さん:2011/04/03(日) 18:24:25.01
n組のピタゴラス数同士の組み合わせから
条件を満たすrを構成することができるため
675132人目の素数さん:2011/04/03(日) 18:35:05.99
a+b=c
a>b
cは素数
bは偶数
acはCM
この条件を満たすa.b.cのうち、積abcの最小値を求めよ
676132人目の素数さん:2011/04/03(日) 23:41:29.28
>>675
> acはCM

舐めてんのか、テメェ!
677132人目の素数さん:2011/04/04(月) 00:01:14.53
>>676
うるさいです
678132人目の素数さん:2011/04/04(月) 00:14:05.28
>>675
bが負でもいいなら、abcに下限は無いように思えるが
679132人目の素数さん:2011/04/04(月) 01:01:59.74
そういう心無い発言はacのCMの条件を満たしません
680132人目の素数さん:2011/04/04(月) 01:47:03.45
(元)公共広告機構のコマーシャルってダジャレはいいとして
CMってどういう意味よ?
なんか倍数関係のテクニカルタームか?
681132人目の素数さん:2011/04/04(月) 06:06:20.46
数学でCMいうたら虚数乗法ですわ
682132人目の素数さん:2011/04/04(月) 08:33:40.48
(1+x)^e/{e^(2x)+4x^e}dx
683132人目の素数さん:2011/04/04(月) 18:33:08.42
一人の人間にチェーンメールを送る。
チェーンメールが届いた人が他の人に転送する確率をpとしたとき、
このチェーンメールが届く人間の人数の期待値を求めよ。

・チェーンメールが届いた人は同時に二人以上に転送しない。
・チェーンメールが自分に届いたときも確率pでほかの人に転送する。
・チェーンメールが2回以上届いた人は同じ人に転送するとは
限らない。
・人々がメールの送ることのできる人の数は十分大きいものとする。
684132人目の素数さん:2011/04/04(月) 23:19:05.21
>>683
文章を推敲したまえ!
685132人目の素数さん:2011/04/04(月) 23:58:42.10
条件も推敲の必要があるな
686132人目の素数さん:2011/04/05(火) 00:32:25.68
気持ち悪い問題だな
>・人々がメールの送ることのできる人の数は十分大きいものとする。
って時点で
>・チェーンメールが自分に届いたときも確率pでほかの人に転送する。
>・チェーンメールが2回以上届いた人は同じ人に転送するとは限らない。
この2つは必要ないだろう
>・チェーンメールが届いた人は同時に二人以上に転送しない。
これもわざわざ書くほどというかなんて言うんだろうこの違和感は

Σk(1-p)(p^(k-1)) でいいんかな
687132人目の素数さん:2011/04/05(火) 00:37:49.76
>>683
修正

一人の人間にチェーンメールを送る。
チェーンメールが届いた人は届いたメールを確率pで自分とは異なる
他の一人に転送する。
全ての人がメールの転送先の対象となり得る。
また、転送先の対象となる確率はどの人に対しても同じである。

このとき、最初に送ったチェーンメールが最終的に届く
人の数の期待値を求めよ。
688132人目の素数さん:2011/04/05(火) 03:25:43.82
同じ人に複数回届いた場合
のべ人数なのか、何度届いても一人扱いなのか。

また、「全ての人がメールの転送先になりうる」というのは
自分以外の全員に等確率なのか、あるいは発信者ごとに確率分布があるのか
689132人目の素数さん:2011/04/05(火) 08:55:55.93
n人の人が丸くなり手を繋いだ(n≧2の整数)
そのときの、結び目を数えることとし、その数をkとする
例えばn=2のとき、k=2となる

ここで、円内の人1人を選択する
その人をAとする
Aの右手を握っているひとをBとする
また、Bの右手を握っている人を1人追加で足して、その人をCとする
また、Cの右手を握っている人を1人追加しその人をDとする
以下、その操作を続けていき、Aと円の中心を結び、A以外の交点(人)がZとなるとき、kを求めよ

ただし、アルファベット順にアルファベットは続くものとする
690132人目の素数さん:2011/04/05(火) 11:24:29.56
> ・人々がメールの送ることのできる人の数は十分大きいものとする。 

全体が十分大きいなら、既に受け取った人に再びメールが届く確率はほぼゼロでないか?
691132人目の素数さん:2011/04/05(火) 11:56:09.44
>>689 
「結び目」とか「追加する」とか、わざと分かりにくいというか読み間違いや誤解をうみそうな表現を用いてるの?

円周上に等間隔に点を配置する。 
ある点Aの真向かい(円心を中心とした対称点)にある点は
Aを1番とし右回りに数えて26番目であるという
円周上にある点は全部でいくつか?

これと同意?
692132人目の素数さん:2011/04/05(火) 12:35:13.74
>>691
まぁ、そう
693132人目の素数さん:2011/04/05(火) 13:22:48.58
>>688
メールが何度届いてもその人は一人として扱います。
どの発信者も自分以外の全員に対して同じ確率で転送します。
694132人目の素数さん:2011/04/05(火) 14:42:45.45
国語を勉強してから来いと言いたくなるような出題者がいるな…
695132人目の素数さん:2011/04/05(火) 15:28:03.16
y=ax^3-x^7-x^9(aは定数)

この関数について、原点から引ける接線の本数を求めよ

696132人目の素数さん:2011/04/05(火) 16:01:55.14
a>0のとき2本、a≦0のとき1本
697132人目の素数さん:2011/04/05(火) 16:07:16.53
一辺が10cmの立方体を紙で包む
紙は長方形である
紙の最小面積を求めよ

ただし、紙との空間は存在しないものとする
698132人目の素数さん:2011/04/05(火) 16:34:58.97
12x10cmx10cmまではできた
699132人目の素数さん:2011/04/05(火) 16:56:42.24
>>697
20cm×40cm?
700132人目の素数さん:2011/04/05(火) 16:58:38.54
>>698
展開図のおさまる長方形というアプローチか
なるほど

それだと


□□
  □
  □
これも立方体の展開図になるから
10cm×10cm×(2×5)まではいきそう

6面分の正方形を正方形のまま残さずにぶった切れば
ひょっとしたらもっと小さくなるのかもな
701132人目の素数さん:2011/04/05(火) 20:43:55.01
10*70を折り曲げる

      □
/□□□/
702132人目の素数さん:2011/04/05(火) 22:01:07.09
幅→0のテープのようなものなら折って重なる部分の面積→0になるんじゃなかろうか
703132人目の素数さん:2011/04/05(火) 22:20:31.25
>>702
一つの頂点から、リンゴの皮むきのような巻き方だな
704132人目の素数さん:2011/04/05(火) 22:23:24.88
このゲームは2人で行う
(仮に、A.Bと呼ぶことにする)
まずAはお金を賭ける
ただし、1円以上であり上限はない
その次にBとじゃんけんをする
○勝てばBから、今賭けた分だけお金を貰える
○負けたらBに今賭けた金額の100倍を支払う
○あいこであればBから、今賭けた金額の10倍を貰う
次にBの番になり今の操作をくりかえす

持ち金はそれぞれ
30000円
相手の金額がちょうど無くなった場合はそこでゲームは終了(勝敗が決まる)
相手が借金をする場合は考えないこととし、相手の負けとなる

一緒にゲームをしないか
705132人目の素数さん:2011/04/05(火) 22:35:42.89
ウホッ

阿部さんw



一円以上つーことはπー2円とかでもいいわけか
706132人目の素数さん:2011/04/05(火) 22:38:16.09
30000円かければ一発じゃん

借金ナシなら300万払うことになってもそれを考えずただ負けで済む
707132人目の素数さん:2011/04/05(火) 22:41:24.07
>>706
あなたが正解
708132人目の素数さん:2011/04/05(火) 23:03:50.97
>>700
□□□◇
  ◇□□□のような形にして考えてみた
紙が折り重なっている部分の傾きθについてtan(θ/2)=1/3
折り重なっている部分の面積は5/3(cm^2)
重なっていてかつはみ出ている部分が1/3(cm^2)
展開図で必要な面積の6(cm^2)と合わせて総面積8(cm^2)まで減った
709697:2011/04/05(火) 23:07:37.56
正解に近づいてきました
710132人目の素数さん:2011/04/05(火) 23:18:37.97
711132人目の素数さん:2011/04/05(火) 23:23:53.85
なるほど…7(cm^2)までは減ったんだけどそんな方法が…
712これは、GAMEだ:2011/04/05(火) 23:55:03.38
13人でじゃんけんをする
ルールは普通のじゃんけん同様
ただし
勝者→2人
敗者→11人 になるまで行う

1回目のじゃんけんで負けたのものは、自動的にそのまま敗者になる
そこで勝ち上がった人が数名いたとすると、その中でまたじゃんけんをし、負けたものが自動的にそのまま敗者となる
この操作を続ける

なお、勝者が1人になった場合はもう一度始めから全員参加となり、じゃんけん再開
すなわち、2回目のじゃんけんが始まる

13人の中の1人「X」が2回目で勝者になる確率を求めよ
713132人目の素数さん:2011/04/06(水) 00:13:27.01
>>712
高校生のための数学の質問スレPART293
http://kamome.2ch.net/test/read.cgi/math/1301297905/507
714132人目の素数さん:2011/04/06(水) 05:14:48.07
連続する3つの自然数n,n+1,n+2のそれぞれの平方の和をMとする。
千の位の数がa、百の位の数がa-1,十の位の数もa-1,一の位の数はaである4けたの数をNとする。
M=Nとなるときのnを求めよ
715132人目の素数さん:2011/04/06(水) 06:53:06.13
そんだけ揃えてしまったら
シラミつぶしでもたかが知れてて面白味がないのでは?

modで絞ったり素因数分解などの工夫の必要性がほとんどなくない?
716132人目の素数さん:2011/04/06(水) 07:02:00.42
>>714
m=n+1とすると、前者は3m^2+2.
すると後者は2を引いて3の倍数である数であり、
条件を満たす数は3種類に絞られる.
717132人目の素数さん:2011/04/07(木) 01:06:46.48
30√2×15√2 = 900 にはできた。
718132人目の素数さん:2011/04/07(木) 01:10:11.64
>>702
折り目で重なる部分のひとつひとつの面積は →0だろうが
折り目の数は →∞ になるので、
重なり全体の面積が→0だとは簡単には結論付けられないと思う。
719718:2011/04/07(木) 01:14:38.42
すまん勘違い
720132人目の素数さん:2011/04/07(木) 03:49:30.06
>>719
うん。細さ倍にすると重複部分の面積はちょうど半分になるね
721132人目の素数さん:2011/04/07(木) 05:10:42.57
a,b,c>=0とする。
√(3(a+b+c)) >= √a +√b+√c を示せ。
722132人目の素数さん:2011/04/07(木) 05:57:44.47
>>721
シュワちゃんの不等式そのまんま
芸がない
723132人目の素数さん:2011/04/07(木) 06:48:08.67
シュワちゃんでできるのか? 凸っちゃんなら、まんまだが
724132人目の素数さん:2011/04/07(木) 14:22:52.15
東大の数学と同じで、
灘高の数学はテクニックだけでは太刀打ちできない。
入試では地頭の善し悪しが問われる。
725132人目の素数さん:2011/04/07(木) 14:57:21.26
正四面体をある箱の中に20個、隙間なく詰めた
このとき、その箱は立方体で無いことを示せ
726132人目の素数さん:2011/04/07(木) 15:03:00.15
(1^2+1^2+1^2)(ra^2+rb^2+rc^2)>=(1ra+1rb+1rc)^2.
727132人目の素数さん:2011/04/07(木) 22:28:04.32
>>721-723

シュワちゃん不等式  >>726
 ラグランジュ恒等式 n*Σ[k=1,n] (r_k)^2 - (Σ[k=1,n] r_k)^2 = Σ[1≦i<j≦n] (r_i - r_j)^2 >= 0,

 凸ちゃん n*f((1/n)Σ r_k) >= Σ[k=1,n] f(r_k),
728132人目の素数さん:2011/04/07(木) 22:31:59.66
>>724
地頭(笑)
729132人目の素数さん:2011/04/07(木) 23:19:11.61
泣く子
730132人目の素数さん:2011/04/08(金) 16:46:48.92
球の外部に点光源を配置し球の表面全体に光を当てたい。
少なくともいくつの点光源が必要か?
球の半径を1、光の届く距離をrとする。(r:正の実数)
球以外に光をさえぎるものは存在しない。
731132人目の素数さん:2011/04/08(金) 17:08:51.15
光というよりプラズマボールを想像したほうがしっくりくるような問題に見えた
rが巨大なら4つあれば足りるけど少し小さくなると難しい
0に非常に近ければ6√3/r^2に近づくんだろうか
732132人目の素数さん:2011/04/08(金) 18:05:43.15
3つのコップがあります
そのうち2つに水が入っています
目隠しをした状態で水の入っていないコップを当ててもらうゲームです
ただし、触ることはできません

良い答え、面白い答え 待ってます!wwwwwwwwwwwww
733132人目の素数さん:2011/04/08(金) 18:37:28.07
体積1を持つ物体のうち、表面積が最大のものを理由とともに書け。
734132人目の素数さん:2011/04/08(金) 19:03:16.27
限りなく薄い物体なら無限の表面積になると思うんだけど
735132人目の素数さん:2011/04/08(金) 19:16:03.71
でかい紙

理由?
見ればわかる
736噂の統計力学:2011/04/08(金) 21:14:41.70
1日1回ある人が等確率で別の人に出会う。
ある噂を持っている人はそのときに他人に教えようか迷う。
教えた噂を相手が知らなければ優越感を+1味わう。
教えた噂を相手が知っていれば優越感を−1味わう。
人は自分たちの集団がN人だと知っており、最高に頭が良く、
相手に出会ったかどうかの記憶があり、自分の優越感を最大化するよう行動する。
全員に噂が拡がるには平均何日かかるか?
737132人目の素数さん:2011/04/08(金) 21:30:58.92
>>735
紙の形をお願いします
738132人目の素数さん:2011/04/08(金) 21:32:24.71
体積が存在するような
739132人目の素数さん:2011/04/08(金) 23:10:17.69
フラクタル
740132人目の素数さん:2011/04/08(金) 23:17:23.68
>>734
>>735
中が空洞で、無限の半径をもち、球面に一つの穴が開いた球の表面積は
無限の長さの辺をもつ紙の表面積より大きい。
741132人目の素数さん:2011/04/08(金) 23:18:10.25
なん…だと!
742132人目の素数さん:2011/04/08(金) 23:34:50.95
体積1を持つ正n面体のうち、表面積が最大となるnを理由とともに書け。
743132人目の素数さん:2011/04/08(金) 23:35:47.95
>>740 を書き込んだ者ですが、

表面積の大小を比較するにあたって、
辺の長さの比がx:x:1/x^2の直方体の表面積をA(x)、
半径x、空洞の球の表面積をB(x)とおき、
lim[x→∞]A(x)/B(x)を求めて直方体と球の表面積の大小を
比較したのですが、
間違ってるところがあったら指摘してください。
744132人目の素数さん:2011/04/09(土) 01:06:35.69
1x1x1 の三次元ペアノ曲線
まじこれ最強
745132人目の素数さん:2011/04/09(土) 01:07:55.87
しかもお前らのよりずっとコンパクト
胸ポケットに入れられる
746132人目の素数さん:2011/04/09(土) 10:45:59.31
一辺が1の正方形ABCDの内部に
 L1:ABを直径とする半円
 L2:BCを直径とする半円
 L3:Cを中心とする4分円
を描くとき、L1、L2、L3で囲まれる領域に内接する円の半径を求めよ
747132人目の素数さん:2011/04/09(土) 14:28:41.91
>>746
> L3:Cを中心とする4分円
D じゃなくて C?
あと、半径いくら?
748132人目の素数さん:2011/04/09(土) 14:51:39.92
>>746
 L3:Cを中心とする半径1の4分円

です、すまぬ
749132人目の素数さん:2011/04/10(日) 00:19:32.18
L3n何の意味が?
750132人目の素数さん:2011/04/10(日) 06:04:59.41
たぶん

L1とL2の重なる部分の一つ上にある領域のことなんじゃないかな
751132人目の素数さん:2011/04/11(月) 13:19:15.40
95%減ってすげえな
752132人目の素数さん:2011/04/11(月) 19:21:41.05
xy平面全体に正n角形を隙間なく敷き詰めたい。
これが可能であるnを全て求めよ。
753132人目の素数さん:2011/04/11(月) 19:39:30.83
n=6
754132人目の素数さん:2011/04/11(月) 19:44:31.79
abc=100
-7≦ab≦3
-ab≦4c≦25
b^2≦5ac≦4c^2

を満たす実数a、b、cの領域Dに収まる最大の円の半径を求めよ
755132人目の素数さん:2011/04/11(月) 23:00:24.31
求めよ!と言われてハイそうします、と答えるほど従順ではないのです
756132人目の素数さん:2011/04/11(月) 23:20:51.26
>>755
消えろ!
カーッ(゚Д゚≡゚д゚)、ペッ
757132人目の素数さん:2011/04/12(火) 01:09:03.21
>>754
実数a、b、cの領域、って何?
758132人目の素数さん:2011/04/12(火) 01:37:43.37
2000! と、1000^2000 はどっちがおおきい?
759132人目の素数さん:2011/04/12(火) 01:53:41.23
うーむ
後者の方が大きい気がするのう

1×2× … ×999×1000×1001×…×1999×2000
―――――――――――――――――――――――
1000×1000×   …            ×1000×1000
760132人目の素数さん:2011/04/12(火) 03:45:50.10
2000! =2000*1999*(1998*2)*(1997*3)*…*(1001*999)*1000
    =2*1999*(1998*2)*(1997*3)*…*(1001*999)*(1000*1000)
    ≦2*1999*(1000^2)^999
    ≦2*2000*1000^1998
    ≦4*1000^1999
    <1000^2000
761132人目の素数さん:2011/04/12(火) 05:00:37.13
a[n+1]=S[n]^2,a[1]=1の一般項求めてください
昔の大数の問題らしいです
762132人目の素数さん:2011/04/12(火) 05:35:34.57
[√44] = 6, [√4444]=66をよくみてこれを一般化し、証明せよ
763132人目の素数さん:2011/04/12(火) 05:51:23.12
>>761
がっこん?
764132人目の素数さん:2011/04/12(火) 05:54:53.10
任意の正の整数nについて、7と0の数字からのみからなるnの正の倍数が存在することを示せ。
765132人目の素数さん:2011/04/12(火) 06:10:27.48
>>761
√a[n+1] = √a[n](√a[n] + 1)から先にすすまぬ…
766132人目の素数さん:2011/04/12(火) 06:27:00.13
>>765
そこまではどうやって変形したの?
767132人目の素数さん:2011/04/12(火) 06:31:40.87
>>765
そこにすら辿りつかん…
差を取っていけばいいと思ったがa[n+2]まで出てきて…
768132人目の素数さん:2011/04/12(火) 07:14:43.36
√a[n+1] = S[n]
 = Σa[1..n] (ってことだよねS[n]って…違ったらすまん)
 = a[n] + Σa[1..n-1]
 = a[n] + √a[n]
769132人目の素数さん:2011/04/12(火) 08:16:06.78
>>759 相加平均が等しいのなら、無個性集団の方が、相乗平均は大きい。
770132人目の素数さん:2011/04/12(火) 12:29:32.83
771132人目の素数さん:2011/04/12(火) 13:47:03.91
>>764
n = p_1^(k_1) * ... * p_m^(k_m) と素因数分解する
ある正数 l が存在して、各iについて
10^l = 0 or 1 (mod p_i^(k_i))
M = 10^l + 10^(2l) + ... + 10^(nl)
とおくと M = 0 (mod n)
7Mは0と7のみからなる正のnの倍数
772132人目の素数さん:2011/04/12(火) 15:17:59.43
>>762
nを正の整数、aを実数とする。
√(Σ[k=0,n-1](11*10^k))=√(10^2n-1)/3
10^n-1<√(10^2n-1)<10^nより
√(10^2n-1)=10^n-1+a(0<a<1)
10^n-1≡0(mod 3)より
[√(10^2n-1)/3]=33…33(3がn個)
[√44…44](4が2n個)
=[2*√(10^2n-1)/3]
=66…66(6がn個)
773132人目の素数さん:2011/04/12(火) 15:20:33.26
>>772
訂正
誤:√(10^2n-1)=10^n-1+a(0<a<1)
正:[√(10^2n-1)]=10^n-1+a(0<a<1)
774132人目の素数さん:2011/04/12(火) 15:24:08.36
>>772
再度訂正
誤:√(Σ[k=0,n-1](11*10^k))=√(10^2n-1)/3
正:√(Σ[k=0,n-1](11*10^2k))=√(10^2n-1)/3
775132人目の素数さん:2011/04/12(火) 15:34:14.25
>>772
3回も連続で投稿してすまん。

誤:10^n-1≡0(mod 3)より
正:10^n-1=99…99(9がn個並んだ整数)より
776132人目の素数さん:2011/04/12(火) 16:05:21.66
>>736
面白い問題というか、一見して面白そうな問題来たな

777132人目の素数さん:2011/04/12(火) 16:57:38.64
mケタの正の整数a[n],a[n-1],・・・,a[0]を
この順に並べ、並べてできる整数をNとする。
Σ[k=0,n]a[k]*x^kが自然数Mで割り切れるとき、
NもMで割り切れる。
正の整数xを求めよ。
778132人目の素数さん:2011/04/12(火) 17:00:11.65
>>777
書き忘れましたがMの桁数はmです。
779132人目の素数さん:2011/04/12(火) 17:37:40.90
780132人目の素数さん:2011/04/12(火) 21:11:53.68
>>761
わからん
ヒントちょうだい
781132人目の素数さん:2011/04/12(火) 22:03:03.71
>>240
       / ̄ ̄ ̄ ̄ ̄ ̄ ̄ ̄
       | 通報しますた!
       \
          ̄∨ ̄ ̄ ̄ ̄ ̄ ̄
                   ∧_∧      / ̄ ̄ ̄ ̄ ̄ ̄ ̄
         ∧_∧     ( ´Д` )    <   通報しますた!
         ( ´Д` )   /⌒    ⌒ヽ    \_______
        /,  /   /_/|     へ \
       (ぃ9  |  (ぃ9 ./    /   \ \.∧_∧  / ̄ ̄ ̄ ̄ ̄ ̄ ̄
        /    /、    /    ./     ヽ ( ´Д` )<  通報しますた!
       /   ∧_二つ (    /      ∪ ,  /   \_______
       /   /      \ .\\     (ぃ9  |
      /    \       \ .\\    /    /  ,、    ((( )))  / ̄ ̄ ̄ ̄ ̄ ̄ ̄
     /  /~\ \        >  ) )  ./   ∧_二∃    ( ´Д` ) < 通報しますた!
     /  /   >  )      / //   ./     ̄ ̄ ヽ    (ぃ9  )  \_______
   / ノ    / /      / / /  ._/  /~ ̄ ̄/ /   /    ∧つ
  / /   .  / ./.      / / / )⌒ _ ノ     / ./    /    \   (゚д゚)シマスタ!
  / ./     ( ヽ、     ( ヽ ヽ | /       ( ヽ、   / /⌒>  )  ゚(  )−
(  _)      \__つ    \__つ).し          \__つ (_)  \_つ   / >
782132人目の素数さん:2011/04/12(火) 22:12:18.83
A、B、C、Dの4人で賭けをする

AはBからお金を貰う→1
BはCからお金を貰う→2
CはDからお金を貰う→3
DはAからお金を貰う→4

貰うお金の金額を示すのはそれぞれ貰う側である
また、このゲームはお金を貰う相手のお金が0になった時点でゲームは終了
それまで永遠に行うものとする

しかし、AとCでチームを組んでおりAとCの合計金額がBとDの合計金額より多くなるようにしている

それぞれの持ち金は100万円
ゲームは1、2、3、4と進んでいく
例えば、AがBから40万円貰うとすると、Bの残金、すなわち60万円が1が終わったときの状況である

ただし、相手から貰う金額は50万円以内とする

このとき、AとCはどのようなことをすればいいか
783132人目の素数さん:2011/04/12(火) 22:26:58.67
>>777
10^(m + 1) の倍数でいいの?
784132人目の素数さん:2011/04/12(火) 22:57:23.30
>>783
10^(m+1)以外にも存在する
785132人目の素数さん:2011/04/13(水) 05:50:26.91
>>779
それL2の外側にあるだろ。
786132人目の素数さん:2011/04/13(水) 08:03:14.10
L2の内部にあるなんて、問題文の何処に書いてあるんだね?
これだから、ゆとりっ子は・・・
787132人目の素数さん:2011/04/13(水) 18:00:48.42
問題
[]をガウス記号とする。
a = [10^m/M+1]
x = (1-a)*M+10^m
>>777 の題意を満たしますか?
788132人目の素数さん:2011/04/13(水) 18:32:06.35
>>786
囲まれたというのは 内部と言う意味ではないのか?
789132人目の素数さん:2011/04/13(水) 19:17:19.63
( 'ー`)
790132人目の素数さん:2011/04/13(水) 21:17:08.42
絶対大小決まってるけど絶対大小分からない二組の数字を WANTED
791132人目の素数さん:2011/04/13(水) 22:07:14.40
あ、日本語で大丈夫ですよ
792132人目の素数さん:2011/04/13(水) 22:33:25.58
2*a[n]+S[n]=a[n+1]-8*a[n]
793132人目の素数さん:2011/04/14(木) 01:47:20.90
>>788
>>746が弧L1、弧L2、弧L3で囲まれると書けばよかったんだろうな

しかし囲まれるという時にはたいてい線に注目することが多いのでは?
また、半円や四分円の領域に注目して、それに囲まれると見る場合は
むしろ図形の外にあることの方が多いと思うよ

  ●
 ●●
こんな感じで3つの円を外接させたとき
3つの円に囲まれる領域はどの円の内側にもない
794132人目の素数さん:2011/04/14(木) 11:17:32.36
とりあえず、おいらには解けない、とだけ言っておこう!
795じゅー:2011/04/14(木) 12:27:46.82
出題です。
以下を満たす整数a,b,cを一つ求めよ。
@a,b,cを三辺とする三角形が存在する。
A(a+b+c)(ab+bc+ca)-9abc<6(a-b)(b-c)(c-a)
796132人目の素数さん:2011/04/14(木) 15:54:54.37
>>793
その円が重なっている場合でも、 「囲まれた」が外側を指す場合があるかな?
797132人目の素数さん:2011/04/14(木) 18:05:02.25
全部の数学記号を漢字で訳す行為をなぜしないのパンパンの子孫という
白人崇拝人種たちは。
798132人目の素数さん:2011/04/14(木) 19:07:58.38
aが素数である確率は?
799132人目の素数さん:2011/04/14(木) 20:27:29.14
>>796
御託はいいから解いてみろ!チンカスめ!
800132人目の素数さん:2011/04/15(金) 02:22:23.07
>>795
 (a,b,c) = (60n, 113n, 173n-1) (n≧2)


キャスフィー 不等式 538 (2) によると

a,b,cが三角形の三辺のとき、
 (a+b+c)(ab+bc+ca) - 9abc ≧ 5.96887066353172*|(a-b)(b-c)(c-a)|,
801じゅー:2011/04/15(金) 02:47:39.97
正解です。
802132人目の素数さん:2011/04/15(金) 07:45:07.76
>>779
女子中学生でも解けるが、ここに書くには面倒すぎる
803132人目の素数さん:2011/04/15(金) 19:12:00.30
>>761
b[1] = b[2] = 3/2
n≧2のとき、b[n+1] = b[n] + 1/4

ただし、b[n] = √a[n] + 1/2 とする

ここまで行けた。
後は、b[n]の一般項を予想して帰納法か?
804132人目の素数さん:2011/04/15(金) 19:12:50.18
>>803
書き間違い
b[n+1] = (b[n])^2 + 1/4
805132人目の素数さん:2011/04/18(月) 19:35:49.91
1〜100の数が書かれたカードが1枚ずつある。
これを2人のプレイヤーが交互に取っていく。
小さい数のカードから順に取らなければならず、1度に取れるのは3枚まで。
全てのカードを取り終えた時、取ったカードの数の合計が多い方の勝ち。
双方が最善を尽くした場合、どちらが勝つか?
806132人目の素数さん:2011/04/18(月) 19:43:01.02
最小にカードをとったほう
807132人目の素数さん:2011/04/18(月) 23:06:12.35
正n角形の各辺を無造作に白か黒に塗り分ける。
ある角に隣り合う二辺が同じ色をしていた場合、
この角を「塊」とする。
「塊」の個数の期待値を求めよ。
808132人目の素数さん:2011/04/18(月) 23:10:16.76
>>807
最初にある辺を白か黒に塗り分ける。
そして、その隣の角を無造作に「塊」であるか「塊で無い」かを決める。
すると隣の辺の色が決まる。

これを繰り返すと無造作に白か黒か塗り分けたことになるから。多分、n/2個じゃないかな
809132人目の素数さん:2011/04/19(火) 06:07:01.86
最後の頂点は決まってしまうからそううまくいかんのじゃないか?

たとえばn=3の時の「塊」の数は1か3のみ
810132人目の素数さん:2011/04/19(火) 11:20:36.19
>>809
期待値は3/2になるじゃん
811132人目の素数さん:2011/04/19(火) 11:51:44.87
●●●3 ●●○1 ●○●1 ○●●1
○○○3 ○○●1 ○●○1 ●○○1

頂点24通りに対して塊は12通りだから期待値は0.5、という理解でいいのか?
812132人目の素数さん:2011/04/19(火) 11:55:01.10
>>811
> 頂点24通りに対して
なんだって?
813132人目の素数さん:2011/04/20(水) 05:39:48.96
(・3・)
814132人目の素数さん:2011/04/20(水) 09:11:03.58
>>813
アルェー
815132人目の素数さん:2011/04/22(金) 17:48:35.11
RA アールエー
816132人目の素数さん:2011/04/22(金) 18:01:10.08
乙警です!
817132人目の素数さん:2011/04/24(日) 17:42:25.05
光の粒子に加速度(0,0,-1)を与える特殊な空間中の点(0,0,1)に点光源を置く。
また、xy平面には鏡が張られており、衝突時、光のz方向の速度が
(-a)倍される。(0<a<1)
点光源から出た光が通過する領域の体積を求めよ。
点光源から出る光の速さを1とする。
818132人目の素数さん:2011/04/27(水) 08:38:27.41
光速の扱い方は特殊なのだがな
それに粒子とも言い切れない。同時に波動性を持つからな。
不備だらけ。

誤解を招きたくなければ光など用いずに単なる粒子を用いればすむ
819132人目の素数さん:2011/04/27(水) 11:21:52.14
では光ではなく架空の粒子で
820132人目の素数さん:2011/04/28(木) 20:36:47.51
ぶっちゃけ >>817 の体積に特に興味ない
821132人目の素数さん:2011/04/28(木) 20:55:29.56
わしもじゃ
822132人目の素数さん:2011/04/29(金) 01:10:43.36
>>817
解き方のヒントをくださいな
包絡線みたいなのを考えるの?
823あんでぃ:2011/04/29(金) 10:25:53.95

Andy
824132人目の素数さん:2011/05/04(水) 19:48:24.83
沖にある船に縄をつけ、水面から9メートルの高さの岸から毎秒2メートルずつ縄を巻き取る。
縄が15メートルになったときの船の速さを求めよ。
825132人目の素数さん:2011/05/04(水) 22:42:32.72
>>824
 x^2 + 9^2 = L^2,
 2x(dx/dt) = 2L(dL/dt),
題意より dL/dt = 2 (m/s)
これに L=15 (m) と x=12 (m) を入れて計算する。
826132人目の素数さん:2011/05/05(木) 12:23:09.78
x^2+y^2=t^2+(sint)^2
t = x/√2 + y/2

で囲まれる領域の面積を求めよ
827132人目の素数さん:2011/05/05(木) 12:34:55.15
>>824
なんかデジャブを感じるぞ
828132人目の素数さん:2011/05/06(金) 16:46:11.74
次のルールで行うゲームの必勝法として正しいのはどれか
なお、ゲーム開始に当たり、先手または後手のどちらを選択するか決定し得る立場にあるものとする

〇ゲームは二人で行う
〇四つの袋があり、その中にはそれぞれ1個、2個、3個、4個のボールが入っている
〇各プレイヤーは毎回任意にどれか一つの袋を選び、その中からボールを1〜3個取り出さなければならない
〇互いに相手が選んだ袋及び取り出したボールの個数は常に把握することができ
同時に、それぞれの袋の中に残っているボールの個数も常に把握することができる
〇この作業を交互に繰り返していき、最後にボールを取り出したプレイヤーを勝ちとする

1.先手を選択し、初手は1個のボールが入っている袋を選択する
2.先手を選択し、初手は2個のボールが入っている袋を選択する
3.先手を選択し、初手は3個のボールが入っている袋を選択する
4.先手を選択し、初手は4個のボールが入っている袋を選択する
5.後手を選択する
829132人目の素数さん:2011/05/06(金) 17:42:37.30
後手勝ち
(),(2,2),(3,3),(1,2,3),それらに1を偶数個追加した形などが後手勝ちなので
その形に持ち込めばいいが、先手がどういう手を選んでもどれかには持ち込める
830132人目の素数さん:2011/05/07(土) 11:26:37.43

数列{F_n}=1,1,2,3,5,8,…の一般項を2011で割っていきます。
2011で割り切れる項は第何項ですか、1つ挙げてください。

理論的な解法付きで解答して下さい。
831132人目の素数さん:2011/05/07(土) 11:41:49.20
>>830
マルチすんな
832132人目の素数さん:2011/05/07(土) 11:47:51.99
マルチという悪さを知りませんでした、すみません。
こっちの問題は無しでお願いします
833132人目の素数さん:2011/05/08(日) 03:49:33.61
>>826
半径 r の円と原点から距離 d の直線 で囲まれる領域の面積は
 S1 = (r^2)arccos(d/r) - d√(r^2 -d^2),
 S2 = (r^2){π-arccos(d/r)} + d√(r^2 -d^2),
 ただし、 0 < d <r,

本問では
 r = √{t^2 + sin(t)^2},
 (x√2 + y)/√3 = (2/√3)t,
 d = (2/√3)t,
834132人目の素数さん:2011/05/10(火) 00:17:04.00
x軸上にn個の点P[h]を左から順にとる。(h=1,2,・・・,n)
このときP[1]は点(0,0)に、P[n]は点(1,0)に一致している。
P[k]とP[k+1]の中点を通り、x軸に垂直に交わる直線と
x^2+y^2=1(x>0,y>0)の交点をR[k]とする。(k=1,2,・・・,n-1)
三角形P[k]R[k]P[k+1]が全て相似な三角形となる場合が存在する
nを全て求めよ。
835 ◆hx3Ob8DAdU :2011/05/12(木) 19:05:52.14
x^5=1解けますか?
836132人目の素数さん:2011/05/12(木) 19:42:40.44
解けますんn
837132人目の素数さん:2011/05/12(木) 20:31:15.41
x=e^(2nπi/5)
838132人目の素数さん:2011/05/13(金) 02:47:45.88
>>835

 x^5 -1 = (x-1)(x^4 +x^3 +x^2 +x+1)
  = (x-1){(x^2 +x/2 +1)^2 -(5/4)x^2}
  = (x-1)(x^2 -x/φ +1)(x^2 +φx+1)
  = (x-1){x^2 -2cos(2π/5)x +1}{x^2 -2cos(4π/5)x +1}
  = (x-1){x-e^(i2π/5)}{x-e^(-i2π/5)}{x-e^(i4π/5)}{x-e^(-i4π/5)},

ここに φ = (√5 +1)/2, 1/φ = (√5 -1)/2,
839132人目の素数さん:2011/05/13(金) 05:57:49.08
>>838
そんな変形の仕方あるんだな。でも>>837がシンプルでいいじゃん
840132人目の素数さん:2011/05/13(金) 21:20:07.30
複素平面上で丁度正五角形描く感じ
841132人目の素数さん:2011/05/14(土) 02:02:51.42
>>837 の補足

 |x|^5 = 1,
 |x| = 1,
 x = e^(iθ), とおける。
 0 = x^5 -1 = e^(i5θ) - 1,
 5θ = 2nπ,
 θ = 2nπ/5,
842 ◆rcig2FSqeE :2011/05/14(土) 13:16:21.01
>>836
>>837
>>838
>>839
>>840
>>841
ありがとうございます
843 ◆ywz7iI8RHQ :2011/05/15(日) 09:04:19.18
sin3x=1
844132人目の素数さん:2011/05/15(日) 09:29:27.35
3x=π/4+2nπ
x=π/12+2nπ/3
845132人目の素数さん:2011/05/15(日) 19:38:40.00
面白いという他はないな....
846132人目の素数さん:2011/05/18(水) 18:51:24.94
次の方程式が表す図形を座標平面に図示せよ。(ただしひとつの平面に書き込むこと)

x^2+y^2=1

x^2+y^2=4

y=±x (−4≦x≦−3,3≦x≦4)

y=0 (−4≦x≦−3,3≦x≦4)

x=0
847 忍法帖【Lv=11,xxxPT】 :2011/05/18(水) 22:27:24.10
簡単過ぎ
848132人目の素数さん:2011/05/18(水) 23:55:39.13
変域指定がチキンだなw
849132人目の素数さん:2011/05/19(木) 07:46:40.78
>>847-848
お前ら>>845を書いた奴の意図が分かってないのか?
小学生が喜ぶような低俗な内容だけどな
850132人目の素数さん:2011/05/19(木) 07:48:41.32
>>849 アンカー訂正
>>847-848
お前ら>>846を書いた奴の意図が分かってないのか?
小学生が喜ぶような低俗な内容だけどな
851132人目の素数さん:2011/05/19(木) 11:58:45.40
a△bを1/(a-b)を表す演算とする。(a△b)△0=a-bとなり、aとbの減法とする。
この演算を使って加法(a+b)、乗法(ab)、除法(a/b)を構成せよ。
852132人目の素数さん:2011/05/19(木) 12:56:04.61
(0△b)△0=-b
(a△((0△b)△0))△0=a+b
853132人目の素数さん:2011/05/19(木) 18:33:01.77
>>850
分かってるわ ボケ
854132人目の素数さん:2011/05/19(木) 19:45:29.55
>>851
積がつくれないんだけど?
855132人目の素数さん:2011/05/19(木) 19:49:35.10
(a△1)△(a△-1) = (a^2 - 1)/2
と加法を利用すれば

(a^2 - 1)/2 + (b^2 - 1)/2 + ((a+b)^2 - 1)/2 + 1/2 = ab

bの代わりに
b△0 = 1/b
とすれば、a/bともなる。
よって

ab=
(((a△1)△(a△-1))△((0△((((b△1)△(b△-1))△((0△((((((a△((0△b)△0))△0)△1)△(((a△((0△b)△0))△0)△-1))△((0△1/2)△0))△0))△0))△0))△0))△0

a/b=
(((a△1)△(a△-1))△((0△(((((b△0)△1)△((b△0)△-1))△((0△((((((a△((0△(b△0))△0))△0)△1)△(((a△((0△(b△0))△0))△0)△-1))△((0△1/2)△0))△0))△0))△0))△0))△0


もっとスマートな方法があるかどうかは知らん。
856132人目の素数さん:2011/05/19(木) 19:55:14.31
すまん。
(a^2 - 1)/2 + (b^2 - 1)/2 - ((a-b)^2 - 1)/2 + 1/2 = a
だから

ab=
((((a△1)△(a△-1))△((0△((((b△1)△(b△-1))△((0△((((((a△((0△b)△0))△0)△1)△(((a△((0△b)△0))△0)△-1))△1/2)△0))△0))△0))△0))△0)

a/b=
((((a△1)△(a△-1))△((0△(((((b△0)△1)△((b△0)△-1))△((0△((((((a△((0△(b△0))△0))△0)△1)△(((a△((0△(b△0))△0))△0)△-1))△1/2)△0))△0))△0))△0))△0)

だった。
857132人目の素数さん:2011/05/19(木) 22:42:15.04
>>856
(a^2 - 1)/2 + (b^2 - 1)/2 + ((a+b)^2 - 1)/2 + 1/2 = ab じゃんか!
858132人目の素数さん:2011/05/20(金) 01:57:31.09
>>856
(a^2 - 1)/2 + (b^2 - 1)/2 - ((a-b)^2 - 1)/2 + 1/2 = ab
ね。度々すまん。

下の式は、(一番外側の括弧を外しわすれたけど)合ってるはず。
誰も(俺も)確認したくはないと思うが
859132人目の素数さん:2011/05/20(金) 03:13:11.67
>>850

分かった上で冷やかされてるのに
それに気づかず改めて分からないだろうと強がってるところが恥の二重の上塗りだな
冷やかされてることにも気づいてないことを表明する結果にもなってるから三重の上塗りか。

>>848の意味を考えてみた?
860132人目の素数さん:2011/05/20(金) 16:47:44.14
>>858
凄いな
つかこの演算って一意に定まるんだろうか
861132人目の素数さん:2011/05/20(金) 18:14:21.43
(a△0)△0=1/(1/a)=a
(0△((0△b)△0))△0=-(-b)=b
と変形はできる。これ以外の自明でないものがあるかはわからん。


>>856
(a△1)△(a△-1) = (a^2 - 1)/2
だと、aが+1や-1の時は駄目だね。

>>856のabの式は
a≠±1、b≠±1、a-b≠±1
の時しか使えない。

同様にa/bの式は
a≠±1、b≠0,±1、a-(1/b)≠±1
の時しか使えない。


c≠0,±aとして
(a△c)△(a△-c) = (a^2 - c)/(2c)
だから

任意のa(特に、a=±1のとき)に対し
c≠0,±a,±1,±(a-1)の時
(a^2 - c)/(2c) + (1^2 - c)/(2c) - ((a-1)^2 - c)/(2c) + 1/2 = a/c
が利用できる。
862132人目の素数さん:2011/05/20(金) 20:18:02.13
そんなこといったら減法の(a△b)△0=a-bもa=bのときは駄目じゃないか
863132人目の素数さん:2011/05/20(金) 20:57:04.66
a=bはもともと定義されてないじゃないか
864132人目の素数さん:2011/05/21(土) 02:41:00.38
wwwwwwwwwwww
865132人目の素数さん:2011/05/21(土) 06:18:05.27

【サッカー】「ビッチを具現化した女と一緒に来てる」アディダス女性社員がハーフナー・マイクをツイッターで中傷し炎上→厳正処分へ★30

1 :ドクターDφ ★:2011/05/20(金) 23:46:35.66 ID:???0

ヴァンフォーレ甲府の長身FW=ハーフナー・マイク。
オランダ出身で、94年に家族で日本国籍を取得した父=ディド・ハーフナー
(GK/名古屋グランパスエイト、ジュビロ磐田などで活躍)の息子であり、
日本初の親子Jリーガーとして複数のクラブを渡り歩いたが、
昨シーズンは、J2得点王にも輝き、甲府のJ1昇格に大きく貢献する目覚しい活躍を遂げた。

今シーズンが楽しみな23歳のマイクは、すでに6試合で3得点。
18日には一般女性と入籍を発表したばかり(入籍日は5月16日)と順風満帆だったが、
そんなマイクが、入籍したお相手とみられる女性と共にアディダス銀座店を訪れた際、
ある問題が起こり、ネット上は大騒ぎとなっている。

なんと、店員の女性が自身のツイッターでマイクに悪口雑言の限りを尽くしたのだ。
掲示板上では、店員の女性も特定され、すでにその女性はツイッターもmixi も退会しているが、
該当するツイッターでは、来店したマイクに、
「そいえば今日マイクハーフナーが来た。ビッチを具現化したような女と一緒に来てて、
何かお腹大っきい気がしたけど結婚してんの(^ω^)??」、
「帰化したからハーフナーマイクかwアシュトンカッチャー劣化版みたいな男が
沢尻劣化版みたいな女連れてきたよwとりあえずデカイね、ホントにwww」などと、
とても店員とは思えないツイートを行っていた。

866 忍法帖【Lv=13,xxxPT】 :2011/05/21(土) 10:40:12.27
867132人目の素数さん:2011/05/21(土) 14:06:24.65
>>863
例えば>>856で上の式がa=1で駄目な理由は式中の(a△1)がa=1で定義されてないからであって
>>862と本質的には変わらないと思うんだが
868132人目の素数さん:2011/05/21(土) 23:14:53.49
某スレが落ちちゃったので…
y^2=x^3+1の整数解(x,y)を全て求めよ
869猫は海賊 ◆4c5pft6zx. :2011/05/21(土) 23:31:09.28
870132人目の素数さん:2011/05/22(日) 00:29:00.90
スレなんて落ちてない。
871132人目の素数さん:2011/05/22(日) 02:43:15.22
>>868
(x,y) = (-1, 0) (0,±1) (2,±3) 

カタラン予想(ミハイレスクの定理)
 http://mathworld.wolfram.com/CatalansConjecture.html
 数セミ増刊「数学100の問題」p.104-105 (1984/Sep)
 Preda Mihuailescu (Paderbon大学): J.reine angew. Math., 572, p.167-195 (2004)
 "Primary Cyclotomic Units and a Proof of Catalan's Conjecture",
872132人目の素数さん:2011/05/22(日) 23:23:18.82
頂点が6個の完全グラフをユークリッド平面上にコンパクトに書きたい。
一番短い辺に対する一番長い辺の比が最も小さくなるような頂点の配置はどんな形か?
873132人目の素数さん:2011/05/24(火) 00:11:39.61
Aさんは1000万人に1人しかかからない病気の検査を受けたところ、結果は陽性だった。
検査の精度が99%であるとすると、Aさんがこの病気にかかっている確率を求めよ。
874132人目の素数さん:2011/05/24(火) 12:49:32.27
0.0000001 × 0.99 + 0.9999999 × 0.01
= 0.010000098 = 1.0000098 %
875132人目の素数さん:2011/05/24(火) 22:59:32.19
>>873
検査の精度が99% とは 
「罹患しているにもかかわらず陰性とでるひとが1%」なのか
「罹患していないにもかかわらず陽性とでるひとが1%」なのか
またはなにか両者の平均のよな物なのかによって変わる。

876132人目の素数さん:2011/05/24(火) 23:05:04.37
100人にひとりしかかからないような病気があり
100人にその病気の罹患を検査したところ、全員が陰性であったが
実際には1人が罹患していた。

1) この検査の制度は 何%だと考えるのが妥当か?

2) あとから実際にはいつでも陰性と結果を出す検査であったことがわかった。
  この検査の制度は 何%だと考えるのが妥当か?

877132人目の素数さん:2011/05/25(水) 01:14:22.04
>>876
また背伸びしておるね

出題や問いかけより
まずは基礎的な勉強が先ですよ
878132人目の素数さん:2011/05/25(水) 01:53:51.14
>>875
確かにそこに非対称性が出るのがより現実的だが、
論理的に解釈すると、「どちらであれ検査器の出力が真である確率が99%」と対称的に解釈するべきだし、
パズル的にも言及がないときには両方のことである解釈するのが一般的だろう。
879132人目の素数さん:2011/05/25(水) 01:58:21.74
>>876
どちらも独立な100サンプル中99サンプル当たったのだから99%だろう
880132人目の素数さん:2011/05/25(水) 02:16:33.45
亀レスだが知恵の輪の問題。
>>558
片方に対してもう片方を回転させないのであれば3次元迷路だと言えるが、回転も考えるのが普通。

パーツA上のある一点に対するパーツB上のある2点の3次元位置が状態のパラメータとなる。
よって6次元だが、パーツB上の2点は距離が不変という拘束条件を持っているので、
5次元の非ユークリッド平面上の迷路であると言える。
881132人目の素数さん:2011/05/27(金) 13:40:34.54
n個の数字を横一列に並べる。(n≧4)
この列に「2011」が含まれる確率を求めよ。
列に使う数字は0〜9のいずれかである。
882132人目の素数さん:2011/05/27(金) 15:28:29.14
類題
f(x) = 
  xを10進数表記したときに小数点以下(どの桁で始まってもよい)に
   2 0 1 1 と4つの数字が連続でならぶことがあるならば 1
  ないならば 0

∫_[x=0〜1] f(x) を求めよ。
 
883132人目の素数さん:2011/05/27(金) 19:16:55.52
>>881
(5-n)/10000
勘だけど
884132人目の素数さん:2011/05/27(金) 20:58:34.41
>>883
nが5で0かよ
885132人目の素数さん:2011/05/27(金) 23:12:44.14
>>884
マジだ(n-3)だった
あってるかは知らんが
886132人目の素数さん:2011/05/27(金) 23:31:45.01
>>885
n=10003で1かよ
887132人目の素数さん:2011/05/27(金) 23:53:59.01
>>886
しかも、n>10003のときは1を超える。確率なのに1より大きいwwww
888132人目の素数さん:2011/05/28(土) 01:34:48.42
>>882 dx が抜けてるかな?
∫[x=0〜1] f(x)dx
= ∫[0.2011≦x<0.2012] f(x)dx + Σ[m=1,…,∞]Σ[k=0,…,9]∫[(k+0.2011)/(10^-m)≦x<(k+0.2012)/(10^-m)] f(x)dx
= 0.0001 + Σ[m=1,…,∞]Σ[k=0,…,9] 10^(-m-4)
= 0.0001 + Σ[m=1,…,∞]10^(-m-3)
= 0.00011111…
= 1/9000

細かい間違いがまだあるかも知れんが、指針はあってるはず。
テーマはフラクタルだな。
1 か 0 かどっちかと思っていたらまさか中途半端な有理数になるなんてね。ひさびさに面白い問題だったよ。
889132人目の素数さん:2011/05/28(土) 01:51:33.06
>>888
(k+0.2011)/(10^-m)>1でないの?
890132人目の素数さん:2011/05/28(土) 01:54:25.66
>>889
あ、(k+0.2011)/(10^m)
もしくは (k+0.2011)(10^-m) だった。
馬から落ちて落馬してた
891132人目の素数さん:2011/05/28(土) 01:55:21.58
バガヤロー!
892132人目の素数さん:2011/05/28(土) 02:13:20.81
>>890
0.2011…
0.k2011…
0.0k2011…
0.00k2011…
の系列だけで、0.5552011… のようなものは含んでいないように見えるが。
893132人目の素数さん:2011/05/28(土) 10:59:50.69
0.20112011…と0.55552011…みたいなやつ二回カウントしてね
894132人目の素数さん:2011/05/28(土) 12:09:46.24
>>882
 1
 有限桁で終わるようなxは可算個だから、実数全体の0%に過ぎない。
 桁数が無限の実数は、0%の場合を除き、2011 を含むだろう。(終)
895132人目の素数さん:2011/05/28(土) 12:23:38.83
>>894
 有限小数、右から見れば自然数

という川柳があるぐらいだから、可算(可付番)
896132人目の素数さん:2011/05/28(土) 12:43:21.82
>>888

小数点以下(m+1)桁目以後がある条件を満たす確率が p(mによらない)とすると、

 Σ[m=0,∞) p(1-p)^m = Σ[m=0,∞) {(1-p)^m - (1-p)^(m+1)} = 1,
897132人目の素数さん:2011/05/28(土) 13:14:50.98
実数を10進数小数に現れる 2011 という文字を11番目の文字Eに置き換えて
左詰めにした11進数の小数が表す実数に対応つける。
一対一対応になる。

0.E... の面積は 1/11
0.?E... の面積は 10/121
0.??E... の面積は 100/1331...
つまり答えは Σ[k=1,2,...,∞] (1/11)(10/11)^k = 1
898132人目の素数さん:2011/05/28(土) 13:50:41.05
>>897
0.2011E…のようなのと0.E…と重複してないか?
899132人目の素数さん:2011/05/28(土) 13:59:20.46
>>897
[0,1]と[0,1]はx->x^2で一対一対応になるから[0,1/4]に入る確率は1/2
900132人目の素数さん:2011/05/28(土) 14:30:15.55
0.EE…は0.????E…に含むよな
901895:2011/05/28(土) 15:34:40.06
改良しますた。

 有限小数、逆さに読めば自然数。
902132人目の素数さん:2011/05/28(土) 16:53:08.86
0.20112011を2度数えたいやつ大杉
903あんでぃは馬鹿 ◆AdkZFxa49I :2011/05/28(土) 17:01:51.50
あんでぃ
904132人目の素数さん:2011/05/28(土) 18:40:32.59
>>887
この時の俺はどうかしていたよ
1-(9999/10000)^(n-3)ですね?
905132人目の素数さん:2011/05/28(土) 18:47:00.56
>>904
a/10^n(aは整数)になってないから間違い。
906132人目の素数さん:2011/05/28(土) 18:53:45.06
>>905
そ、そんなぁ…
907132人目の素数さん:2011/05/28(土) 19:02:01.22
>>904
n=4で1/10000
n=5で20/100000
n=6で300/1000000
n=7で4000/10000000
n=8で(50000-1)/100000000
n=9で(600000-30)/1000000000
908132人目の素数さん:2011/05/28(土) 19:32:11.07
>>902
二度数えたい訳ではない
909132人目の素数さん:2011/05/28(土) 20:27:41.37
ならば数えない式にするほうがいい
910132人目の素数さん:2011/05/28(土) 22:06:20.25
がん細胞を攻撃する免疫細胞の活動を抑制する抗体を、a個の細菌の中で
b個の細菌を組み合わせた時に、減少させる事ができる。全ての組み合わせを
試験した時に、この効果が得られる確率を求めよ。
また、a=115、b=46の時を計算せよ。
911あんでぃは馬鹿 ◆AdkZFxa49I :2011/05/28(土) 22:19:17.14
あんでぃ
912132人目の素数さん:2011/05/28(土) 22:42:28.13
>>901
反歌

 自然数、逆さに読めば有限小数。
>>911

914あんでぃは賛同 ◆AdkZFxa49I :2011/05/28(土) 23:56:37.06
あんでぃ

916あんでぃは賛同 ◆AdkZFxa49I :2011/05/29(日) 00:11:05.21
あんでぃ

918あんでぃは賛同 ◆AdkZFxa49I :2011/05/29(日) 00:18:23.80
あんでぃ

920あんでぃは賛同 ◆AdkZFxa49I :2011/05/29(日) 00:39:26.60
あんでぃ

922あんでぃは賛同 ◆AdkZFxa49I :2011/05/29(日) 00:49:36.60
あんでぃ

924132人目の素数さん:2011/05/29(日) 01:59:15.73
正規分布に従って生起する値が、
観測史上の最大値を更新する確率を時間の関数で表すとどうなる?

質問っぽくなっちゃったけど、
単に興味本位で疑問に思ったので
925132人目の素数さん:2011/05/29(日) 02:01:38.06
>>910
全ての組み合わせって無限にありそうだけど、意図してるのはどういう意味
926132人目の素数さん:2011/05/29(日) 03:28:14.22
>>924
それは、値は連続量で、時間(時刻)は離散時間を考えるということでいいの?
で、その確率は時刻0から見た確率ということでいいんだよね。

ならば、まず考えるのは時刻nまでの最大値
(=正規分布に従うn個のサンプルの最大値)の確率分布。
927132人目の素数さん:2011/05/29(日) 03:30:38.92
時刻nまでの最大値の確率密度関数をg_n(X)とし、
正規分布自体の累積分布関数をF(X)とすると、
時刻n+1で最大値を更新する確率は
∫(1-F(X))g_n(X)dX (積分区間は-∞〜∞)
928132人目の素数さん:2011/05/29(日) 06:34:43.15
>>925
a個の中からb個(0<=b<=a)を選択する組み合わせだから、その組み合わせの総数は有限
929132人目の素数さん:2011/05/29(日) 09:30:45.13
そこかよ。
んじゃ、1だろ。
930132人目の素数さん:2011/05/31(火) 21:53:51.62
n個の数字(0〜9)を横一列に並べてできる文字列を
全て1つ以上含む文字列を作りたい。
この文字列に必要な数字の個数の最小値とその作り方を求めよ。
931132人目の素数さん:2011/05/31(火) 22:03:58.12
>>910
Σ[k=0, a-b](C[a-b, k]/C[a, b+k])
a=115、b=46のとき
Σ[k=0, 69](C[69, k]/C[115, 46+k])
= 590295810358705651712/41023433264885418154367916094212808
= 1.438923472219419558090714758372489854340760324323379... × 10^-14
932132人目の素数さん:2011/06/01(水) 02:26:31.49
>>931

 C[a-b,k]/C[a,b+k] = C[b+k,b]/C[a,b] = {C[b+k+1,b+1] - C[b+k,b+1]}/C[a,b],
k=0,1,・・・・・,a-b の和をとると、
 C[a+1,b+1]/C[a,b] = (a+1)/(b+1) = 116/47,
となるんぢゃね?
933132人目の素数さん:2011/06/01(水) 03:33:00.30
>>910

すべての組合せ: 2^a
b個の細菌を含む組合せ: 2^(a-b)
なので (1/2)^b = 1.4210854×10^(-14)
かな?
934132人目の素数さん:2011/06/01(水) 07:07:55.83
>>932
間違えた、訂正
(Σ[k=0, a-b]C[a-b, k])/(Σ[k=0, a-b]C[a, b+k])
以下同文
935あんでぃはアホ ◆AdkZFxa49I :2011/06/01(水) 07:13:19.32
>>923
あんでぃ
936132人目の素数さん:2011/06/02(木) 01:54:14.18
>>934

Σ[k=0,a-b] C[a-b,k] = 2^(a-b),

Σ[k=0,a-b] C[a,b+k] = 2^a - Σ[L=0,b-1] C[a,L]
  = 2^a - 5.1494160339320×10^32
  = 2^a / 1.01255237707844
937132人目の素数さん:2011/06/03(金) 23:35:24.15
cosx*tanx=cos2x (-π≦x≦π)
938132人目の素数さん:2011/06/04(土) 02:20:55.39
>>930
n=3 のとき最小値 1002, n=4 のとき最小値 10003、一般の n ではたぶん 10^n + n - 1

n=3
・最初の文字列を 12 とする
・12 の後ろに 312 を付け加えて 12312 とすると、この文字列は 123,231,312 を含む
・23 の後ろに 423 を付け加えて 12342312 とすると、この文字列は 234,342,423 を新たに含む
・同様の操作を続けて相異なる3文字から成る長さ3の全ての文字列を含む文字列を構成できる
・一箇所の 12 の後ろに 112 を付け加えると、文字列は 121,211,112 を新た含む
・同様の操作を続けて aab, aba, baa という形の文字列も全て含む文字列を構成できる
・一箇所の 00 の後ろに 0 を付け加えると、文字列は 000 を新たに含む
・同様にして 111〜999 も含む文字列を構成できる

以上で構成された文字列は条件を満たす
各段階で付け加えた文字数と新たに含まれる長さ3の文字列の数は同じなので、
文字列全体の長さは 1002、最小性は明らか


n=4
・最初の文字列を 123 とし、abc の後ろに dabc を付け加える操作を続けて、
 相異なる4文字から成る長さ4の全ての文字列を含む文字列を構成する
・abc の後ろに aabc を付け加えて、abca, bcaa, caab, aabc を新たに含む文字列を構成する
・bac の後ろに abac を付け加えて、baca, acab, caba, abac を…
・aba の後ろに ba を付け加えて、abab, baba を…
・aab の後ろに baab を付け加えて、aabb, abba, bbaa, baab を…
・aab の後ろに aaab を付け加えて、aaba, abaa, baaa, aaab を…
・aaa の後ろに a を付け加えて、aaaa を…
939132人目の素数さん:2011/06/04(土) 15:59:43.59
>>930
n-1 個の数字の列を頂点とし、頂点 a_1a_2...a_n から a_2...a_na_{n+1} への有向線分を持つ有向グラフを作る.
この有向線分に a_{n+1} という数字を割り振っておく。
各頂点に出入りする有向線分は 10 本づつで、
有向グラフは連結なので、一筆書きが可能である.
940132人目の素数さん:2011/06/04(土) 16:04:17.09
(続き)この一筆書きに現れる有向線分に割り振られた数字を順に並べ
その後に最初の n-1 個を追加したものが求める文字列となる。
有向線分の数は 10^n なので 10^n+n-1.
941132人目の素数さん:2011/06/05(日) 00:11:38.37
>>930
最小値n
元の文字列を1種類の数字にすればよい

カンタンすぎるねw
あるいは出題の不備
942132人目の素数さん:2011/06/05(日) 00:18:55.37
>>941
10^n 個の文字列を全て内部に持つような文字列という意味じゃないの。
943132人目の素数さん:2011/06/05(日) 00:24:41.42
>>941
例えばn=2だったら
00 01 02 03 04 05 … 99を全て含む文字列を作れってことでしょ
0123だったら01 12 23を含んでいる事になる
944132人目の素数さん:2011/06/05(日) 00:27:14.36
>>942
それだと問題文の(0〜9)という部分が不備。

>n個の数字(0〜9)を横一列に並べてできる文字列を
こういう言い方はしない。

その10^n通りの文字列全てを内部にもつなら
普通の文字列なら10^n+n-1.、環状の配列なら10^nが最小値だろうな
945132人目の素数さん:2011/06/05(日) 00:37:50.00
>>944
数字(0〜9) で意味は取れるが、はて。
946132人目の素数さん:2011/06/05(日) 00:46:13.65
>>944
> >n個の数字(0〜9)を横一列に並べてできる文字列を
> こういう言い方はしない。

そこで切るからだよ。

n個の数字(0〜9)を横一列に並べてできる文字列を全て1つ以上含む文字列

意味の通る表現だと思うが。
947132人目の素数さん:2011/06/05(日) 00:55:39.11
今まで普通に通じてたんだから通じるってことだろうが。
948132人目の素数さん:2011/06/05(日) 12:12:16.56
意味を補えば通じるから普通に解けるけど、数学的厳密さはないってことだと思うよ。
数学の問題として、「○○さんは解けているからこれは意味が通じる問題です」というのはちょっと。
もっとも、問題集や参考書じゃないんだから、「普通に意味が通じる」程度で十分なんだけど。割とどうでもいい議論。
949132人目の素数さん:2011/06/05(日) 17:46:56.44
全員分かっとるから脱線から戻れハゲ
950132人目の素数さん:2011/06/05(日) 17:52:57.89
ドヤ顔で、カンタンすぎるねw、と書いてたから
どれだけ考えてるんだか、なんてね。
951132人目の素数さん:2011/06/06(月) 02:09:43.51
>>948
おまえ、周りから、話が長い上に何言ってるんだかワカラナ〜イ、言われているだろ?(ゲラゲラ!)
952132人目の素数さん:2011/06/06(月) 03:18:33.09
>>951
的はずれな返しだな
「論理性」と「スレの流れや感情論」を区別できないと
その程度しかできないんだろうけど
953132人目の素数さん:2011/06/06(月) 23:02:44.77
全員分かっとるから脱線から戻れハゲ
954132人目の素数さん:2011/06/07(火) 01:25:55.27
>>953
お前が脱線を続けてるわけだがw
955132人目の素数さん:2011/06/07(火) 01:52:05.95
ゲラゲラ
956132人目の素数さん:2011/06/07(火) 12:48:31.34
「1から6の目が書いてあるサイコロを振って〜」みたいな問題に
サイコロの目が均等に出るか問題文からはわからないから
解は出せないみたいなこと言っちゃう人いたなぁ
957132人目の素数さん:2011/06/07(火) 13:34:55.89
ユークリッド平面とは限らないから
10進数とは限らないから

どこで折り合いつけるんだろうね
そういうひとは
958132人目の素数さん:2011/06/07(火) 18:06:32.94
143 名前: 名無しさん@お腹いっぱい。 投稿日: 2011/06/07(火) 03:16:03.04 ID:zBVuRq1Z
宇宙のことについては天才的知識をお持ちの皆さんに小学生でも
分かるように教えていただければさいわいです。
「曲がったコンパクト化」という言葉に偶然突き当たりましたが
意味を教えてください。自分で調べてみましたが頭がシュークリームです。
>宇宙スレから誘導されました。
959132人目の素数さん:2011/06/07(火) 22:18:43.60
とにかく面白い問題っていう議題なんだし
文章が微妙でも面白さが伝わって本題であれこれいうのが理想なわけで
面白くもないつっこみはスレ違い
960132人目の素数さん:2011/06/08(水) 01:15:13.51
>>959
スレ違いもはなはだしい
しかもくどい
961132人目の素数さん:2011/06/08(水) 13:57:37.84
いいから面白い問題持って来いよ
962132人目の素数さん:2011/06/08(水) 15:10:01.46
a^b=b^a(ただしa=b)を満たす整数a,bの組を求めよ。
963132人目の素数さん:2011/06/08(水) 16:22:56.48
>>962
>(ただしa=b)
ワロタ

≠のつもりかな?
≠より不等号のほうがスマートじゃないか?
964132人目の素数さん:2011/06/08(水) 16:33:08.50
>>963
ミスったwwwwwwwww
a>bってことにしよう!
965132人目の素数さん:2011/06/08(水) 16:45:22.40
答えが求めた組以外にない証明とかも必要かな
966132人目の素数さん:2011/06/08(水) 16:49:33.52
>>965
うん。
967132人目の素数さん:2011/06/09(木) 19:17:05.73
誰か解いてくれよ〜。
スレチだったか?
968132人目の素数さん:2011/06/09(木) 23:47:39.04
a>b≧2であることがわかる。
自然数nを用いてa=nb(n≧1)とおき、a^b=b^aに代入。
(nb)^b=b^(nb)
n^b*b^b=b^(nb)
n^b=b^b(n-1)
n=b^(n-1)

b<nを仮定すると
n = b^(n-1)>bとなり、これは成立しない。
よってb≧n
自然数xを用いて
b =xn(x≧1)とおく。
n=(xn)^(n-1)
x^(n-1)*n*(n^(n-2)-1)=0
n^(n-2)=1

n=1のときa=bとなり条件a>bに反する。
n=2のときn^(n-2)=1
b=2
a=4
n>2のときn^(n-2)=1は成立しない

以上よりa^b=b^a(a>b)を満たす整数組(a,b)は
(a,b)=(4,2)のみである
969132人目の素数さん:2011/06/10(金) 16:33:44.85
既出問題ですが

高速道路で30分間に自動車が存在する確率が0.95である場合、
10分間では確率はどれぐらいになりますか?
(確率は一定であると仮定します)

<Googleの面接試験問題より>

ググらなくても解けるよね
970132人目の素数さん:2011/06/10(金) 19:55:05.63
>>968
すげええええええええ!!正解!!
971132人目の素数さん:2011/06/10(金) 20:06:04.15
>>969
1-0.05^(1/3)じゃね
972132人目の素数さん:2011/06/10(金) 20:46:11.88
「30分間に自動車が存在する確率」とか糞訳だと思う
973132人目の素数さん:2011/06/10(金) 21:16:11.02
>>971
正解、60分での確率が1-0.05^2に気が付けば後は早いよね

974132人目の素数さん:2011/06/10(金) 21:22:48.56
>>973
全く同じ発想だったwww
975132人目の素数さん:2011/06/11(土) 00:26:48.66
>>969
>>971-974

これはなかなか面白かった
976132人目の素数さん:2011/06/11(土) 08:43:28.42
>自然数nを用いてa=nb(n≧1)とおき

aはbの整数倍なのはどうして?
977132人目の素数さん:2011/06/11(土) 09:03:47.96
>>976
a,bが整数だから。
978132人目の素数さん:2011/06/11(土) 09:10:28.81
>>962
a,b が自然数なら,昔の大学入試にあった。
自然対数をとって変形すると,(log a)/a=(log b)/b
f(x)=(log x)/x とおいて増減を調べると,f'(x)=(1-log x)/x^2 より x=e で極大値をとる。
よって,b<e<a なので,b=1,2
b=1 のとき,a^1=1^a より a=1 で不適。
b=2 のとき,a^2=2^a これを満たす a は x>e には1つだけで,a=4 のとき成立するので,a=4
よって,(a,b)=(4,2)

ただし,(1) f(x)=(log x)/x の増減を調べよ。 がヒントだった気がする。

a,b が整数なら,他にも (a,b)=(-4,-2) がある。
979132人目の素数さん:2011/06/11(土) 09:46:39.86
>>978
あ、ほんとだ自然数だった。すまん。
入試にもあったんだね。
980132人目の素数さん:2011/06/11(土) 16:13:33.62
面白い問題おしえて〜な 十七問目
http://kamome.2ch.net/test/read.cgi/math/1284253640


13人でじゃんけんをする
ルールは普通のじゃんけん同様
ただし
勝者→2人
敗者→11人 になるまで行う

1回目のじゃんけんで負けたのものは、自動的にそのまま敗者になる
そこで勝ち上がった人が数名いたとすると、その中でまたじゃんけんをし、負けたものが自動的にそのまま敗者となる
この操作を続ける

なお、勝者が1人になった場合はもう一度始めから全員参加となり、じゃんけん再開
すなわち、2回目のじゃんけんが始まる

13人の中の1人「X」が2回目で勝者になる確率を求めよ
981132人目の素数さん:2011/06/11(土) 16:54:10.59
>>977
それじゃ例えばa=3,b=2のときnは自然数にならないから説明になってないんじゃない?
982132人目の素数さん:2011/06/11(土) 18:04:38.90
二回目で終わる確率×2/13
983132人目の素数さん:2011/06/11(土) 18:05:25.62
>>981
そもそも>>976の質問がおかしいのか?
984132人目の素数さん:2011/06/11(土) 19:00:09.55
>>980
もうちょっと隅々までちゃんと詰めたルール説明がほしいな…
985132人目の素数さん:2011/06/11(土) 20:08:30.28
y^2=x^3+1の整数解を全て求めて!
986132人目の素数さん:2011/06/11(土) 21:57:21.79
>>977
精密に議論していくと「a,bが自然数でa>bならa=nbと置ける」ことが言えるが、
このことは自明ではない。君がどういう考えでこれを導いたのか知らないが、もし

「a^b=b^aだから、a^bはbで割り切れる。よってaはbで割り切れる」

こういう考えでa=nbと置いたのなら、それは間違い。
この考え方が成り立つのは、bが素数の場合のみ。
a^b=b^aという方程式でa=nbを言いたければ、もっと工夫して議論しなければならない。
987132人目の素数さん:2011/06/11(土) 22:33:17.78
>>985
既出
988132人目の素数さん:2011/06/11(土) 22:36:34.49
3^(x-1)≧(x-1)^3
実数xが変化するとき、実数xの動く軌跡を求めよ
989132人目の素数さん:2011/06/11(土) 23:05:38.22
>>985
既出 >>871
990132人目の素数さん:2011/06/11(土) 23:48:02.78
問題の「2回目で勝者になる確率」の「回目」というのは
あいこでもインクリメントされる数なのか、
勝者が3人以上になった時にもインクリメントされる数なのか、
勝者が1人になってしまってリセットされる時に初めてインクリメントされる数なのか
三通り解釈できてしまってまだ問題に取りかかれないし
どれだったとしても行き着くところは平凡な総当たりの確率問題で面白くない
991132人目の素数さん:2011/06/12(日) 01:06:34.29
>>980
答えや解き方、問題の条件が適切かどうかなどを
出題者が吟味した上で出しましょう

出来そこないの問題のコピペとはw
992132人目の素数さん:2011/06/12(日) 01:25:03.29
>>986
だな。
aやbを指数表記した上で、等式の指数部分に注目するのが手っ取り早い

もっとも、ピックアップしてる部分だけで論じるなら
>bが素数の場合のみ
これも間違いだけど。
993132人目の素数さん:2011/06/12(日) 12:09:07.26
>>987>>989
ありがとう。
994132人目の素数さん:2011/06/13(月) 09:07:03.69
面白い問題おしえて〜な 十八問目
http://kamome.2ch.net/test/read.cgi/math/1307923546/
次スレ立てたった
995132人目の素数さん:2011/06/13(月) 14:41:56.40
996132人目の素数さん:2011/06/14(火) 02:27:10.22
>>994
ごくろう、さがってよい
997132人目の素数さん:2011/06/14(火) 10:07:21.38
二百七十五日。
998132人目の素数さん:2011/06/14(火) 10:08:20.10
二百七十五日一分。
999132人目の素数さん:2011/06/14(火) 10:09:20.13
二百七十五日二分。
1000132人目の素数さん:2011/06/14(火) 10:10:20.17
二百七十五日三分。
10011001
このスレッドは1000を超えました。
もう書けないので、新しいスレッドを立ててくださいです。。。